■ちょっとした疑問や質問はここに書いてね125■

このエントリーをはてなブックマークに追加
1ご冗談でしょう?名無しさん
前スレ
■ちょっとした疑問や質問はここに書いてね124■
http://science6.2ch.net/test/read.cgi/sci/1267183170/

質問する前に
教科書や参考書をよく読もう
http://www.google.com/ などの検索サイトを利用し、各自で調べること
宿題を聞くときは、どこまでやってみてどこが分からないのかを書くこと。
丸投げはダメだからね
(丸投げ君は完全無視。答えるだけ無駄。)

質問に対する返答には、何かしらの返答ちょうだいね
★書き込む前に>>2の注意事項を読んでね
★数式の書き方(参考)はこちら>>3
(予備リンク:>>2-10
荒らし厳禁、煽りは黙殺、忘れないうちに定期age
単発質問スレを発見したらこのスレッドへの誘導をよろしくね。

定番FAQ
英語最新版
http://math.ucr.edu/home/baez/physics/
旧版日本語訳
http://research.kek.jp/people/morita/phys-faq/
他にも参考にあるサイトなどあればぜひ。
例)http://ja.wikipedia.org/
※wikipedia内の説明はすべてが正確なわけではありません。
このスレでの受け答えもそうですが。相互に補完しつつ精度を高めましょう。
2ご冗談でしょう?名無しさん:2010/03/16(火) 22:48:40 ID:??? BE:378684858-S★(517072)
書き込む際の注意

1.)
板の性格上、UNIX端末からの閲覧も多いと推察されます。
機種依存文字(ローマ数字、丸数字等)は避けて下さい。

2.)
以下のような質問に物理板住人は飽き飽きしているので、たぶん無視されます。
しないで下さい。
「相対性理論は間違っています」「量子力学は間違っています」
「宇宙論は間違っています」「シュレディンガーの猫は変です」
「永久機関を作りました」「タイムマシンについて教えて」
「どうして〜?」関連(→「どのようにして〜?」と質問すること)
「なぜ〜?」関連(たいてい、物理学の対象ではないため)

「どうして・なぜ」:http://academy6.2ch.net/philo/
(哲学板・雑談板のほうがむいている場合が多いです。)
雑談は雑談スレで:http://science6.2ch.net/test/read.cgi/sci/1181386663/
3ご冗談でしょう?名無しさん:2010/03/16(火) 22:49:03 ID:??? BE:75737524-S★(517072)
数式の書き方(参考)
●スカラー:a,b,...,z, A,...,Z, α,β,...,ω, Α,Β,...,Ω,...(「ぎりしゃ」「あるふぁ〜おめが」で変換)
●ベクトル:V=[v1,v2,...], |V>,V↑,vector(V) (混同しないならスカラーの記号でいい。通常は縦ベクトル)
●テンソル:T^[i,j,k...]_[p,q,r,...], T[i,j,k,...;p,q,r,...]  (上下付き1成分表示)
●行列  M[i,j], I[i,j]=δ_[i,j]  M=[[M[1,1],M[2,1],...],[M[1,2],M[2,2],...],...], I=[[1,0,0,...],[0,1,0,...],...]
(右は全成分表示。行または列ごとに表示する。例:M=[[1,-1],[3,2]])
●転置行列・随伴行列:M ',tM, M†("†"は「きごう」で変換可) ●行列式・トレース:|A|=det(A), tr(A)
●複号:a±b("±"は「きごう」で変換可)
●内積・外積・3重積:a・b, a×b, a・(b×c)=(a×b)・c=det([a,b,c]), a×(b×c)
●関数・数列:f(x), f[x] a(n), a[n], a_n
●平方根:√(a+b)=(a+b)^(1/2)=sqrt(a+b) ("√"は「るーと」で変換可)
●指数関数・対数関数:exp(x+y)=e^(x+y) ln(x/2)=log[e](x/2)(exp(x)はeのx乗、lnは自然対数)
●三角比:sin(a), cos(x+y), tan(x/2)
●絶対値:|x|  ●共役複素数:z~ ●ガウス記号:[x] (関数の変数表示と混同しないよう注意)
●階乗:n!=n*(n-1)*(n-2)*...*2*1, n!!=n*(n-2)*(n-4)*...
●順列・組合せ:P[n,k]=nPk, C[n,k]=nCk, Π[n,k]=nΠk, H[n,k]=nHk ("Π"は「ぱい」で変換可)
4ご冗談でしょう?名無しさん:2010/03/16(火) 22:49:14 ID:??? BE:596428079-S★(517072)
数式の書き方続き
●微分・偏微分:dy/dx=y', ∂y/∂x=y_x ("∂"は「きごう」で変換可)
●ベクトル微分:∇f=grad(f), ∇・A=div(A),∇xA=rot(A), (∇^2)f=Δf ("∇"は「きごう」,"Δ"は「でるた」で変換可.)
●積分:∫[0,1]f(x)dx=F(x)|_[x=0,1], ∫[y=0,x]f(x,y)dy, ∬[D]f(x,y)dxdy, ∬[C]f(r)dl ("∫"は「いんてぐらる」,"∬"は「きご
う」で変換可)
●数列和・数列積:Σ[k=1,n]a(k), Π[k=1,n]a(k) ("Σ"は「しぐま」,"Π"は「ぱい」で変換可)
●極限:lim[x→∞]f(x) ("∞"は「むげんだい」で変換可)
●図形:"△"は「さんかく」 "∠"は「かく」 "⊥"は「すいちょく」 "≡"は「ごうどう」 "∽"は「きごう」
●論理・集合:"⇔⇒∀∃∧∨¬∈∋⊆⊇⊂⊃∪∩"は「きごう」で変換
●等号・不等号:"≠≒<>≦≧≪≫"は「きごう」で変換
5ご冗談でしょう?名無しさん:2010/03/17(水) 00:56:28 ID:???
werwerw3erw3e4
6ご冗談でしょう?名無しさん:2010/03/17(水) 00:57:32 ID:???
dfddffoidfiodfoidfoidfoidfiodfiodfiodfiodioffiidddiiffffffififififififiif
ffdjgjvbjjjjjvjvjvjvjvjvjvjvjvjvfjvbjcjkivjcjcjcjcjcjcjvcjvb jcfjvbjfc
vbdfvdf
vcfvf
7ご冗談でしょう?名無しさん:2010/03/17(水) 00:58:21 ID:???
azazazazaaazazaaazzaazzaaazzaazzaazazaazzaazazzaazza
bkj
8ご冗談でしょう?名無しさん:2010/03/17(水) 00:59:11 ID:???
vggvvggvvggvvggvvgvbuuhhbuuhbuuhguhuhuuhhuuhuhuuhuuhuhbh
9ご冗談でしょう?名無しさん:2010/03/17(水) 01:00:04 ID:???
gygyygygygygyggygyg
10ご冗談でしょう?名無しさん:2010/03/17(水) 01:00:49 ID:???
bkjbkjbkjkjbkjbjkkjbbjkkjbbkkjnbbkkbkkjbkjbkbkjkkjbbjkkjbbbjkkjkjbbkj
11ご冗談でしょう?名無しさん:2010/03/17(水) 01:01:39 ID:???
vbnnbbhjhhjjjhjjhjjhjjjhhjjhjhjhhjh
12ご冗談でしょう?名無しさん:2010/03/17(水) 01:02:22 ID:???
vbnnbvvbnnbvvbnnbvvbnnbvvbnnbvnvvbnnbvnnbvvbnnbnbvvbnnbvvbnnbv
13ご冗談でしょう?名無しさん:2010/03/17(水) 01:37:44 ID:???
dsds
14ご冗談でしょう?名無しさん:2010/03/17(水) 01:38:42 ID:???
vbnnbvvbnnbvvbnnbvvbn
15ご冗談でしょう?名無しさん:2010/03/17(水) 01:39:52 ID:???
bn
16ご冗談でしょう?名無しさん:2010/03/17(水) 01:40:56 ID:???
bhbhhhbhhgghgh
17ご冗談でしょう?名無しさん:2010/03/17(水) 01:41:49 ID:???
kj
18ご冗談でしょう?名無しさん:2010/03/17(水) 01:42:48 ID:???
vbnnbvvbnnbvvbnnbvvbnnbvvbnnbvvbnnbvvbn
19ご冗談でしょう?名無しさん:2010/03/17(水) 01:43:32 ID:???
vbnnnbvvbnnbvbnnbvvbnnbvvbnnbvvbnnbvvbnnbvvbnnbvvbn
20ご冗談でしょう?名無しさん:2010/03/17(水) 01:44:21 ID:???
vnbvbnvnbvbnvnvnbvbnnbvnbnbnbvnbvbnvbn
21ご冗談でしょう?名無しさん:2010/03/17(水) 02:17:27 ID:???
dsds
22ご冗談でしょう?名無しさん:2010/03/17(水) 02:18:27 ID:???
qwertyuiopasdfgfhghhbnccvxcxcxcxcxcxxxxcxcxcxcx
23ご冗談でしょう?名無しさん:2010/03/17(水) 02:19:30 ID:???
zazazaza
24ご冗談でしょう?名無しさん:2010/03/17(水) 02:20:15 ID:???
lklklklk
25ご冗談でしょう?名無しさん:2010/03/17(水) 02:21:11 ID:???
nvbbbbbbbbbbbbb
26ご冗談でしょう?名無しさん:2010/03/17(水) 02:22:33 ID:???
vnnbnbnbnnnbvnbb
27ご冗談でしょう?名無しさん:2010/03/17(水) 02:23:31 ID:???
hjjhvhjvhhjvhjkjb/kbkjv
28ご冗談でしょう?名無しさん:2010/03/17(水) 02:24:30 ID:???
vbn
29ご冗談でしょう?名無しさん:2010/03/17(水) 02:38:13 ID:???
■ちょっとした疑悶や膣悶はめこすじに掻いてね69■
30ご冗談でしょう?名無しさん:2010/03/17(水) 03:46:23 ID:???
bnmbmjkk
31ご冗談でしょう?名無しさん:2010/03/17(水) 03:47:23 ID:???
bhbbjkbkjbjjkbjbjbkj
32ご冗談でしょう?名無しさん:2010/03/17(水) 03:49:34 ID:???
bk
33ご冗談でしょう?名無しさん:2010/03/17(水) 03:50:25 ID:???
vbnnbvvbnnbvvbnnbvvbnnbvvbnnbvvbn
34ご冗談でしょう?名無しさん:2010/03/17(水) 03:51:28 ID:???
bhbhbhbhbhbhbhbhbhbh
35ご冗談でしょう?名無しさん:2010/03/17(水) 03:52:38 ID:???
lvhvhljhv;piuvpliuyvluyvl
36ご冗談でしょう?名無しさん:2010/03/17(水) 03:53:30 ID:???
vjhvjhvjhvjhvjhvjvjhjh
37ご冗談でしょう?名無しさん:2010/03/17(水) 03:54:23 ID:???
bkbkbk;bkjb;kjbbj;l;kjbbjk;;;kjbbj;;kjbbjk;;kjb
38ご冗談でしょう?名無しさん:2010/03/17(水) 05:15:14 ID:lgfDNzhR
この宇宙で一番小さい物質は1つですか?
39ご冗談でしょう?名無しさん:2010/03/17(水) 07:12:55 ID:???
sasaaas
40ご冗談でしょう?名無しさん:2010/03/17(水) 07:13:52 ID:???
qazxswedcvfrtgnhhgyyhghghghgffdfdfdffdfdffdffd
41ご冗談でしょう?名無しさん:2010/03/17(水) 07:14:43 ID:???
hjjhjhhjhjjhhjjhhjjhhjhjhjjh
42ご冗談でしょう?名無しさん:2010/03/17(水) 07:15:26 ID:???
kjbkjbbjkkjbbjkkjbbkkjbbjkkjbbjkkjbbjk
43ご冗談でしょう?名無しさん:2010/03/17(水) 07:16:13 ID:???
bmnnbbnmmnbbnmmnbnmmnbbnmmnmnbbnmmnbbnmmnbbmmnbbnmmnb
44ご冗談でしょう?名無しさん:2010/03/17(水) 07:17:10 ID:???
aaaaaaqqqqwwwwssssssssddddffffffffffgghh
45ご冗談でしょう?名無しさん:2010/03/17(水) 08:42:02 ID:NBC7s9Ot
在日朝鮮人帰れ!!
46ご冗談でしょう?名無しさん:2010/03/17(水) 10:27:24 ID:???
在膣メコス人帰れ!!
47ご冗談でしょう?名無しさん:2010/03/17(水) 16:53:37 ID:cB8eyabD
四角の中に、丸いボールを入れるとき、どんな物理理論を使いますか。大学2年レベルの問題・。
48ご冗談でしょう?名無しさん:2010/03/17(水) 19:15:29 ID:???
メコスジの中に、マラい棒を入れるとき
49ご冗談でしょう?名無しさん:2010/03/17(水) 21:43:47 ID:???
srfmxkc
50ご冗談でしょう?名無しさん:2010/03/18(木) 00:34:11 ID:???
dadsadada
51ご冗談でしょう?名無しさん:2010/03/18(木) 00:35:07 ID:???
sssssssssasa
52ご冗談でしょう?名無しさん:2010/03/18(木) 01:06:34 ID:???
vg
53ご冗談でしょう?名無しさん:2010/03/18(木) 01:31:14 ID:???
uuuy
54ご冗談でしょう?名無しさん:2010/03/18(木) 01:32:03 ID:???
giiugguiiugguiiugguiiuggui
55ご冗談でしょう?名無しさん:2010/03/18(木) 01:33:13 ID:???
vbnbvvbnnbvvbn
56ご冗談でしょう?名無しさん:2010/03/18(木) 01:34:10 ID:???
vggvvggvvggvvggvvggvvggvvggvvg
57ご冗談でしょう?名無しさん:2010/03/18(木) 01:35:10 ID:???
vbn
58ご冗談でしょう?名無しさん:2010/03/18(木) 01:52:11 ID:???
荒らしてるやつ1人なのか?
59ご冗談でしょう?名無しさん:2010/03/18(木) 03:29:47 ID:???
eeeeeeeeeeee
60ご冗談でしょう?名無しさん:2010/03/18(木) 03:30:46 ID:???
ddd
61ご冗談でしょう?名無しさん:2010/03/18(木) 03:31:38 ID:???
zaazzaazzazazazazazazazazxsxssda
62ご冗談でしょう?名無しさん:2010/03/18(木) 03:32:25 ID:???
21212121212121322435456647568
63ご冗談でしょう?名無しさん:2010/03/18(木) 03:33:13 ID:???
cccccxxxxxxxxxzzzzzyyyyywwwwjjjjj/////:::::;;;;l
64ご冗談でしょう?名無しさん:2010/03/18(木) 03:34:03 ID:???
chghcfffcggfcccfggfccfggfccfggfc
65ご冗談でしょう?名無しさん:2010/03/18(木) 03:34:54 ID:???
li
66ご冗談でしょう?名無しさん:2010/03/18(木) 03:36:17 ID:???
jjj
67ご冗談でしょう?名無しさん:2010/03/18(木) 09:33:17 ID:???
kku
68ご冗談でしょう?名無しさん:2010/03/18(木) 09:34:13 ID:???
hgvhgvhgvhghhghhghghgxc
69ご冗談でしょう?名無しさん:2010/03/18(木) 09:35:13 ID:???
jhv
70ご冗談でしょう?名無しさん:2010/03/18(木) 09:36:04 ID:???
hhh
71ご冗談でしょう?名無しさん:2010/03/18(木) 10:44:26 ID:???
rere
72ご冗談でしょう?名無しさん:2010/03/18(木) 10:45:08 ID:???
aaaasasasasasasadddddsdsdsdsdsffffdfdfdf
73ご冗談でしょう?名無しさん:2010/03/18(木) 10:48:23 ID:???
ffff
74ご冗談でしょう?名無しさん:2010/03/18(木) 10:49:10 ID:???
12345678910
75ご冗談でしょう?名無しさん:2010/03/18(木) 10:50:35 ID:???
fff
76ご冗談でしょう?名無しさん:2010/03/18(木) 10:51:19 ID:???
zzzz
77ご冗談でしょう?名無しさん:2010/03/18(木) 10:53:43 ID:???
fff
78ご冗談でしょう?名無しさん:2010/03/18(木) 10:54:48 ID:???
zz
79ご冗談でしょう?名無しさん:2010/03/18(木) 12:05:21 ID:???
cv
80ご冗談でしょう?名無しさん:2010/03/18(木) 12:58:39 ID:???
zzzddsds
81ご冗談でしょう?名無しさん:2010/03/18(木) 13:00:15 ID:???
dds
82ご冗談でしょう?名無しさん:2010/03/18(木) 14:33:13 ID:lw4mRhDK
ddddd
83ご冗談でしょう?名無しさん:2010/03/18(木) 14:34:09 ID:???
fg;dhg;gdoihgodifg
84ご冗談でしょう?名無しさん:2010/03/18(木) 14:35:11 ID:???
bn
85ご冗談でしょう?名無しさん:2010/03/19(金) 00:49:02 ID:IVd+dtr3
地球の大きさの水の塊が宇宙に浮かんでたら、その中心はどうなるんだろうね?
予想できる人がいたら教えてね。
86ご冗談でしょう?名無しさん:2010/03/19(金) 01:55:34 ID:???
aaaaaaaaaaaaaaaaaaaaaaaaaaaa
87ご冗談でしょう?名無しさん:2010/03/19(金) 01:56:21 ID:???
zzzzzzzzzzzzzzzzzzzzzzzzzzzzzzzzzzzzzzzzzzzzzzzzzzzzzzzzzzz
88ご冗談でしょう?名無しさん:2010/03/19(金) 01:59:51 ID:???
g
89ご冗談でしょう?名無しさん:2010/03/19(金) 02:00:53 ID:???
ds
90ご冗談でしょう?名無しさん:2010/03/19(金) 02:01:31 ID:???
ewaewaewaweaweaweewaaewweaweewaaweewaaweweaaweewaawe
91ご冗談でしょう?名無しさん:2010/03/19(金) 03:28:21 ID:???
mnb
92ご冗談でしょう?名無しさん:2010/03/19(金) 04:45:57 ID:???
ccx
93ご冗談でしょう?名無しさん:2010/03/19(金) 12:21:27 ID:0UKYPJ/U
幾学を、方程式にしてみよ
94ご冗談でしょう?名無しさん:2010/03/19(金) 14:42:59 ID:???
luyv
95ご冗談でしょう?名無しさん:2010/03/19(金) 15:03:40 ID:???
k
96ご冗談でしょう?名無しさん:2010/03/19(金) 15:15:43 ID:???
iuiu
97ご冗談でしょう?名無しさん:2010/03/19(金) 15:16:28 ID:???
fdfd
98ご冗談でしょう?名無しさん:2010/03/19(金) 15:17:41 ID:???
gh
99ご冗談でしょう?名無しさん:2010/03/19(金) 15:18:44 ID:???
vb
100ご冗談でしょう?名無しさん:2010/03/19(金) 15:19:47 ID:???
nm
101ご冗談でしょう?名無しさん:2010/03/19(金) 15:21:36 ID:???
bn
102ご冗談でしょう?名無しさん:2010/03/19(金) 15:59:07 ID:???
/./
103ご冗談でしょう?名無しさん:2010/03/19(金) 16:02:06 ID:???
gf
104ご冗談でしょう?名無しさん:2010/03/20(土) 03:19:40 ID:???
sssssssssssっさあああああああああああああああ
105ご冗談でしょう?名無しさん:2010/03/20(土) 04:12:30 ID:???
mn
106ご冗談でしょう?名無しさん:2010/03/20(土) 04:17:19 ID:???
zzxzxxzxxzxxzzxxzxzzxxxzzxxzzxzx
107ご冗談でしょう?名無しさん:2010/03/20(土) 04:58:56 ID:???
vjhjhvhj
108ご冗談でしょう?名無しさん:2010/03/20(土) 05:08:58 ID:???
nm
109ご冗談でしょう?名無しさん:2010/03/20(土) 06:09:25 ID:???
vbn
110ご冗談でしょう?名無しさん:2010/03/20(土) 07:15:58 ID:???
fdfd
111ご冗談でしょう?名無しさん:2010/03/20(土) 07:37:30 ID:???
gg
112ご冗談でしょう?名無しさん:2010/03/20(土) 12:47:54 ID:???
dd
113ご冗談でしょう?名無しさん:2010/03/20(土) 14:41:22 ID:EeqHa5mm
”フランク・ウィルチェック”の「物質のすべては光」に
    クオークの質量の起源は未だ判らない。
とあったのですが、クオークの質量が判らないということは、
その上部構造である核子の質量もわかっていないと思っていいのですか?
114ご冗談でしょう?名無しさん:2010/03/20(土) 14:55:07 ID:???
質量はわかっているが、質量の起源がわからないと言っている。
素粒子について基本的な知識を断片的に説明しておく。

原子核は基本的に中性子と陽子で構成されていて、
どちらの質量もあまり変わらず、3つのクォークから成り立っている。
しかし、クォークの質量を単純に3倍しても中性子や陽子の質量よりも圧倒的に足りない。
つまり、質量は足りないはずなのに、3つのクォークが合わさることで質量がかさましで発生する。
そのかさましされた質量の起源とはなにか?それがわからない。
115ご冗談でしょう?名無しさん:2010/03/20(土) 15:04:30 ID:???
>>114
QCDから計算できる程度には分かってるだろ
116ご冗談でしょう?名無しさん:2010/03/20(土) 15:39:17 ID:???
fdhgfdhfhdfhfdhdfhdf
117ご冗談でしょう?名無しさん:2010/03/20(土) 15:42:30 ID:???
ここで一つ質問です。

全知全能の神と全知全能の悪魔と全知全能の幽霊が戦ったら、誰が勝つのでしょうか?
118ご冗談でしょう?名無しさん:2010/03/20(土) 16:21:44 ID:???
>>113-114
かなり分かってるよ
u、d、sクォークからできるハドロンはカイラル対称性の自発的破れから来る寄与が主要になる
3フレーバー格子QCDでよい精度で計算できてる
2年前くらいに出た論文では確か誤差5%も無いくらいの結果が出てた
119ご冗談でしょう?名無しさん:2010/03/20(土) 16:59:57 ID:EeqHa5mm
>>114-115,>>118
ありがとうございます。
それでは電子の質量に関してはどうなのですか?
120ご冗談でしょう?名無しさん:2010/03/20(土) 17:02:12 ID:???
誰か>>117の質問に答えてくれる方はいないでしょうか?
121ご冗談でしょう?名無しさん:2010/03/20(土) 17:05:42 ID:???
>>120
板違い
122ご冗談でしょう?名無しさん:2010/03/20(土) 17:32:08 ID:???
>>119
クォークの質量の起源が分かっていないのと同程度に分かっていない。
123ご冗談でしょう?名無しさん:2010/03/20(土) 17:44:12 ID:EeqHa5mm
>>122
ありがとうございます。
とくべき問題はもう残っていないのではないかと思っていたのですが、
素粒子論はまだまだ豊穣な大地なのですね。
124ご冗談でしょう?名無しさん:2010/03/20(土) 17:56:20 ID:???
素粒子論は難題は沢山残っている

しかし、実験の大きな進展は望めないからかなり厳しい
LHCでTeVスケールには手が届くようになってそこらへんは進展があると思うが、
プランクスケールはさらに10^16倍くらい、10ケタ以上高いからな
理論側だけで色んな可能性を探るだけの状況がこのまま半永久的に続く
公算は高い
125ご冗談でしょう?名無しさん:2010/03/20(土) 18:09:01 ID:???
ペレルマンによるポアンカレ予想の証明は、物理的な考え方を数学に導入したそうですが、
具体的にはどのような概念を導入してどのように解決したんでしょうか?
126ご冗談でしょう?名無しさん:2010/03/20(土) 19:40:47 ID:???
http://iiyu.asablo.jp/blog/2007/10/29/1877162
この解釈が正しいかどうかはわからない
127ご冗談でしょう?名無しさん:2010/03/20(土) 21:18:56 ID:???
場の量子論あたりで1/√2Eの規格化を導入する理由が分かりません
いつもの(2S+1)Vd^3p/h^3等だけでは問題があるのでしょうか
128ご冗談でしょう?名無しさん:2010/03/20(土) 21:22:32 ID:???
>>127
ローレンツ対称性
129ご冗談でしょう?名無しさん:2010/03/20(土) 22:32:07 ID:3HETM4lD
人類は宇宙の熱的死をどのように解決するのでしょうか?
130ご冗談でしょう?名無しさん:2010/03/20(土) 23:59:48 ID:???
>>129
宇宙が膨張している限り、熱的死の心配はない。
131ご冗談でしょう?名無しさん:2010/03/21(日) 00:34:32 ID:???
太陽が無くなる方が先だろうし、さらにそれより先に人類が滅亡するだろう
まあ、今から心配することではない
132ご冗談でしょう?名無しさん:2010/03/21(日) 16:27:58 ID:6wS0iCzg
戦艦宇宙船ミラージューナイトに、酸素箱、酸素循環(昔、酸素循環装置)を付けること
133ご冗談でしょう?名無しさん:2010/03/21(日) 18:55:22 ID:MEtFwnfD
話題が違って申し訳ありませんが、この場を借りてお話させていただきます。
民主党は極めて危険な集団です。
小沢、鳩山のお金の問題なんか吹っ飛ぶ程の大問題です。
民主党は結党以来の目標として在日外国人への参政権付与や
外国人住民基本法という悪法を推進しています。
これらが成立すれば、日本はめちゃくちゃになります。
彼らは日本なんか好きでは無いのです。中国、韓国の工作員です。
子供手当てとか普天間基地とか細かいことはどうでもいいのです。
民主党は日本国家、日本国民をおとしいれようとしているのです。
彼らは犯罪集団です。
民主党に危険な正体を皆さんに知ってもらいたいのです。
これはマスコミでは放映されません。在日資本が入っているからです。
民主党は日本人を不幸にします。それを喜んでやる犯罪集団なのです。
下の法案はその一例です。

やばい!もっとひどい法案を民主党は進めている!
おかしい!よくもこんな法案を考えることが出来るな!
正真正銘の犯罪法案だ!

外国人住民基本法
1. 日本にどんな方法であっても 3年以上住めば 日本人と同じ権利を与える。
2. 二重国籍であろうが 本国の選挙権があろうが 犯罪者であろうが関係は無い。
3. 選挙権・永住権・戦争賠償 その他諸々全ての権利が保証される。
4. 尚且つ 外国人としての不当な扱いに対し 撤廃させる権利を持つ。

円より子が提出した。この人、民主党の中の民主党、国賊中の国賊、工作員中の工作員だ
これこそ日本を殺害する行為だ。こいつのウィキも見るべし。犯罪者だ、完全に。
134ご冗談でしょう?名無しさん:2010/03/21(日) 18:58:27 ID:oGvvhSdM
競馬で差しとか追い込みが決まるのってなんでなんでしょうね?
135ご冗談でしょう?名無しさん:2010/03/21(日) 19:08:30 ID:oGvvhSdM
先行馬は疲れたぶん前にいって
道中は同じくらいのスピードで走ってるわけじゃん?
136ご冗談でしょう?名無しさん:2010/03/21(日) 19:14:14 ID:???
>>130
はあ?
閉じた空間(孤立系)のエントロピーが増大し、空間が広がれば広がるほどに熱量が減っていくのが熱的死なんだがw
まあ熱的死も空想SFだし何とでも言えるかw
137ご冗談でしょう?名無しさん:2010/03/21(日) 20:01:26 ID:sXSyaDgD
牛乳でつくったやつの上澄みの精製を3、4回繰り返してつくった超回復水と、最上級キニー二の最上級分部を合わせて、完全に反応させたものは、切傷を接着する効果。
138ご冗談でしょう?名無しさん:2010/03/21(日) 20:46:54 ID:tNyYmU35
子供のおもちゃで、海外も含めて、サンドイッチなど、大量生産する器具ありませんか。工業生産ラインに使える。
登記
139koh:2010/03/21(日) 20:50:11 ID:NXKPORCd
複素数論における対数関数の多価関数性から、たとえ対数関数の引数が実数だろうと、実数解析学の対数関数とまったく同じ値は返しません。
(もちろん、実数解析学の対数関数にその実数の引数を放りこんだ値も、複素数論の対数関数が返す無数の値のなかの一つにはなっているのですが)
この事情は、複素関数論の累乗関数の引数が実数のときにも発生しますよね。
こう考えると、実数に限った対数関数と、複素数論における対数関数は、明確に区別すべきな気がするのですが、ぼくの持っている教科書では、どちらも
ln(z)
と表記してあり、非常にややこしいです。
明確に区別するような表記法ってないんですかね。

たとえば、[2^1/2を答えよ]とかって問題が出題されると
「え?複素数論なの?実数に限るの?」って悩んでしまいます。
長文ですいません。
140ご冗談でしょう?名無しさん:2010/03/21(日) 22:07:48 ID:???
>>139
「分枝は○○を取る」とただし書きを付ければいい。

ところで 2^1/2 と書いたら通常の演算規則では (2^1)/2 だとは思わないか?
141ご冗談でしょう?名無しさん:2010/03/21(日) 23:37:49 ID:ukE0Hxan
googleで検索して、静止衛星軌道が35786kmで有ることは解ったのですが、

そこまで行って帰るのに、どれぐらい掛かるのでしょうか?
142koh:2010/03/21(日) 23:56:32 ID:NXKPORCd
>140
表記の件はややこしくてすいません
なるほど、分枝を明言するのですか
143ご冗談でしょう?名無しさん:2010/03/22(月) 00:05:36 ID:???
>>141
ひまわり7号は打ち上げから静止化完了まで7日
アポロ11号の月旅行は発射から帰還まで約200時間
144ご冗談でしょう?名無しさん:2010/03/22(月) 00:35:56 ID:PDVTPBoN
スレチな質問でしたら申し訳ありません
高速増殖炉もんじゅが稼動すると、核兵器用プルトニウムが精製されるという話を聞きますが、
実際に核兵器に使用できるほどの高純度のプルトニウム239の精製は可能なんでしょうか?
145ご冗談でしょう?名無しさん:2010/03/22(月) 01:11:55 ID:???
プロパゲータの定義で、虚数係数つけるけど
あれって何の意味があるの?
146ご冗談でしょう?名無しさん:2010/03/22(月) 01:14:04 ID:???
ウィキペディア見ればいいんじゃね?
147ご冗談でしょう?名無しさん:2010/03/22(月) 04:23:16 ID:???
>>144
高速増殖炉のプランケットではPu239が効率的に生成されるのに対し、核兵器にとって邪魔となるPu240の生成は抑えられます。
実際「もんじゅ」のブランケットではPu239 96%以上のPuが生成されており、これは十分核兵器に使えるレベルです。
ちなみに「精製」するのは再処理過程においてです。
148ご冗談でしょう?名無しさん:2010/03/22(月) 08:45:02 ID:spj4CN8Y
下のスレの202(202、229、230)は物理学で言うと
何という現象(法則)に当て嵌まるのでしょうか?

ミクロ化して考えていて、ふと、これは物理学なのでは?
と思いググってもみましたが、結局解らなかったので
ここに質問させてもらいました。

http://yutori.2ch.net/test/read.cgi/fashion/1268322485/
149ご冗談でしょう?名無しさん:2010/03/22(月) 09:07:16 ID:???
物理学とは何か?がよく分からないけど、ちらっと見た感じでは
どんな良い物でも宣伝しなきゃ人に売れないし、
例えクソでも立派に宣伝されていたら良い物だと人は錯覚して買うってことですよね。
心理学とか認知とかそっちの話なんじゃあないの?
150ご冗談でしょう?名無しさん:2010/03/22(月) 09:14:07 ID:8K/WCemI
宇宙の中心は、地球だと思うのですがどう思いますか?

もし宇宙の中心がどこか他の遠くにあるとすれば
地球から見てそっちの方角の銀河より、その反対側の銀河の方が遠く離れていくはずですよね?
間隔がより大きくなっていくというか。
でもそんな事実はなくて、どちらも同じように離れて行ってるのだとしたら
やっぱり地球かその近くからビックバンが広がったと思うんですが。
151ご冗談でしょう?名無しさん:2010/03/22(月) 09:27:54 ID:???
下の黒い点を星としよう。
1) ・・・・・・・・・・・・・・・
2) ・ ・ ・ ・ ・ ・ ・ ・ ・ ・ ・ ・ ・ ・ ・
3) ・ ・ ・ ・ ・ ・ ・ ・ ・ ・ ・ ・ ・ ・
1)→2)→3)ではどの星から見ても左右同じようにはなれて行くのだが、
これだとどこを中心と言いたいのかな、君は。

152ご冗談でしょう?名無しさん:2010/03/22(月) 09:28:36 ID:???
しまった。ミスった。まぁ分かるか。
153ご冗談でしょう?名無しさん:2010/03/22(月) 09:31:23 ID:???
>>150
その場合、中心といえるような特別な点が無いという答えはありえる
現状の標準的な宇宙論はそういう認識
154ご冗談でしょう?名無しさん:2010/03/22(月) 09:36:06 ID:spj4CN8Y
>>149
物理学とは何か?と言われても…
量子力学になるんでしょうか…?
無学ですみません。

劣化した物(事)がピラミッド型に増える現象に
当て嵌まる法則(名前)はありますか?という質問です。
宣伝云々は別に気にしなくていいです。
言葉足らずでした。すみません。
155ご冗談でしょう?名無しさん:2010/03/22(月) 09:41:18 ID:???
>>145
Green関数的な定義だと、係数を自由に定義しても良さそうだけど
T積使った定義とちょうど一致するようにしてるんじゃない
Wickの定理でも後者が出てくるし、摂動展開、ファインマンルールで
上手く合うような定義に調整している
156ご冗談でしょう?名無しさん:2010/03/22(月) 10:04:07 ID:ZHkkj+Ab
>>143

ありがとうございます。
157ご冗談でしょう?名無しさん:2010/03/22(月) 10:36:39 ID:8K/WCemI
地球が中心の場合
1)・・・・・・・・・・・・1234●6789・・・・・・・・・・・・
2)・・・・・・・・1・2・3・4・●・6・7・8・9・・・・・・・・
3)1・・・2・・・3・・・4・・・●・・・6・・・7・・・8・・・9

地球が外れにある場合
1)・・・・・・・・・・・・123456●89・・・・・・・・・・・・
2)・・・・・・・・1・2・3・4・5・6・●・8・9・・・・・・・・
3)1・・・2・・・3・・・4・・・5・・・6・・・●・・・8・・・9

地球から見て、中心方向の向こう端が速く離れていきます。
158ご冗談でしょう?名無しさん:2010/03/22(月) 10:42:46 ID:???
>>157
違うだろバカ。

地球が中心の場合
1)・・・・・・・・・・・・4321●1234・・・・・・・・・・・・
2)・・・・・・・・4・3・2・1・●・1・2・3・4・・・・・・・・
3)4・・・3・・・2・・・1・・・●・・・1・・・2・・・3・・・4

地球が外れにある場合
1)・・・・・・・・・・・・654321●12・・・・・・・・・・・・
2)・・・・・・・・6・5・4・3・2・1・●・1・2・・・・・・・・
3)6・・・5・・・4・・・3・・・2・・・1・・・●・・・1・・・2

3)4・・・3・・・2・・・1・・・●・・・1・・・2・・・3・・・4
3)6・・・5・・・4・・・3・・・2・・・1・・・●・・・1・・・2
等しい。
159ご冗談でしょう?名無しさん:2010/03/22(月) 10:52:12 ID:8K/WCemI
まったく等しくないじゃん。
上のは、左右ともに16マス離れていったけど
下のは、左24マス右8マスだろ!!
160ご冗談でしょう?名無しさん:2010/03/22(月) 11:05:06 ID:???
無限という概念が理解できないようだ。
161ご冗談でしょう?名無しさん:2010/03/22(月) 11:09:14 ID:???
「無限」とはどういうことなのでしょうか?
「存在」とはどういうことなのでしょうか?
「無」とはどういうことなのでしょうか?
162ご冗談でしょう?名無しさん:2010/03/22(月) 11:10:52 ID:???
>>159
お前の宇宙、どんだけ小さいんだよ。
163ご冗談でしょう?名無しさん:2010/03/22(月) 11:11:57 ID:???
>>157
まあそういう一次元の例えでもいいけど、例えば、円状にそういう風に並べて
円の半径が広がっていくような状況を考えてごらん
円周の点でどこが中心なんて概念は出ないよね
例えば、そういうこと
非等方性が観測されないからといって、その点が中心と呼べるような
特別な点とは限らない
164ご冗談でしょう?名無しさん:2010/03/22(月) 11:16:17 ID:???
誰か>>161の質問に答えろカス。
165ご冗談でしょう?名無しさん:2010/03/22(月) 11:27:36 ID:???
同じ時計をばら撒いて一番遅く進んでるところが宇宙の中心でしょ
166ご冗談でしょう?名無しさん:2010/03/22(月) 11:30:52 ID:???
>>164
ゴーグルで調べてください
167ご冗談でしょう?名無しさん:2010/03/22(月) 11:31:07 ID:8K/WCemI
>>163
なぜ円周上だけに限定するのですか?
それこそ円の中点というものはちゃんと存在してるじゃないですか。

それにビックバンをでっかい爆発に例えるなら、やっぱり範囲は有限ですよ。
違いますか?
168ご冗談でしょう?名無しさん:2010/03/22(月) 11:44:43 ID:???
光速が有限であるため、地球と因果関係のある(観測可能の)宇宙は、地球を中心とした有限半径の球体だって分かる?
実際には宇宙は、もっと広く広がっている。

闇夜に包まれた100m四方のグラウンドで、灯りの届き見える半径1メートルが綺麗に自分を中心にしてるからって、
自分がグラウンドの中心に立ってると考えるとか、井戸の中の蛙レベルの脳だな。
169ご冗談でしょう?名無しさん:2010/03/22(月) 11:51:56 ID:???
dgtdgtjhhjtje
170ご冗談でしょう?名無しさん:2010/03/22(月) 11:52:47 ID:???
erregerSDc
171ご冗談でしょう?名無しさん:2010/03/22(月) 11:53:36 ID:???
awawwaawwaawwaawwaawwa
172ご冗談でしょう?名無しさん:2010/03/22(月) 11:55:02 ID:???
vbn
173ご冗談でしょう?名無しさん:2010/03/22(月) 11:55:20 ID:8K/WCemI
ぼくには分からない。
観測可能な外側にももっと広がっているなんて、そんなの確かめようがない。
有限半径ギリギリの銀河が、その外側の何らかの影響を受けてるのが分かったよ、だとかいうのならともかくね。

グラウンドの灯りが半径1メートルを照らしているのなら、そのグラウンドは1m四方の広さなのかもしれない。
だって、グラウンドの外まで照らす必要ないから。
174ご冗談でしょう?名無しさん:2010/03/22(月) 11:55:54 ID:???
vhvjhvjhvhjhvhjhvjhvhhjvhjhjvjh
175ご冗談でしょう?名無しさん:2010/03/22(月) 11:56:50 ID:???
cfgfccfgfgfcccfggggfccccfggggfccfg
176ご冗談でしょう?名無しさん:2010/03/22(月) 11:57:41 ID:???
hgchgcghgcch
177ご冗談でしょう?名無しさん:2010/03/22(月) 12:12:51 ID:???
zxzczx
178ご冗談でしょう?名無しさん:2010/03/22(月) 12:15:54 ID:D2LKzCfA
なぁちょっと聞きたいんだけど

最大種タイセイヨウクロマグロ
は全長4.5m・体重680kg
そして水中速度は魚雷並みの90km/h

カツオ
大型のものは全長1m・体重18kg
そして水中速度はクロマグロを凌ぐ最高100km/h

、が衝突した時に何が起こるかを物理板っぽく教えてくれ
179ご冗談でしょう?名無しさん:2010/03/22(月) 12:18:58 ID:???
>宇宙の中心は、地球だと思うのですがどう思いますか?

もうそれでいいよ。宇宙は地球から始まったのさ。
180ご冗談でしょう?名無しさん:2010/03/22(月) 12:49:14 ID:???
>>167
> ビックバンをでっかい爆発に例えるなら

まあ、このへんが啓蒙書の類で安易にアナロジーを使うことの弊害なんだけどな
そのアナロジーからのイメージだけで受け取ってしまう
アナロジーってのは大抵、不正確だよ
無理やり似たような現象持ってきて「分かって気にさせてる」だけだから

本当に理論を理解したかったら、専門書読んで、実際どういう方程式をどう解いて導かれるというようなことを学ぶしかない
それが嫌なら、ふうん?分かったような分からんような?位で満足しておいた方が良い
専門的な理解に到達するのはそう楽じゃない
181ご冗談でしょう?名無しさん:2010/03/22(月) 12:58:40 ID:???
円周宇宙の中心の定義があやふららしいね。円の中心(相当する点)が実は宇宙の
外部にある事に注意するとでは宇宙の中心をどう定義し直そうかという事になる。
「中心」という言葉の定義次第では、宇宙には中心は無いとも言えるし別な言い方を
すれば円周上の全ての点が宇宙の中心だったとも言える(宇宙初期の円周はとても
半径が小さくて事実上一点とみなしても差し支えない程だったという視点から)
実際、質問者の書き込み内容から察しても掲示板での説明で正しく理解出来るように
なるレベルじゃないだろうから仕方ないだろう。啓蒙書読んでここが分からないとかいう
のが出てきたらもう一度来るといいよ。
182ご冗談でしょう?名無しさん:2010/03/22(月) 12:59:38 ID:???
長文でしかも日本語ちょとおかしかったスマソ
183ご冗談でしょう?名無しさん:2010/03/22(月) 13:02:08 ID:???
>>181
円周や球面のような多様体は高次元ユークリッド空間への埋め込みを考えなければ
定義不能というわけではないということは、理解しておく必要があるな
184ご冗談でしょう?名無しさん:2010/03/22(月) 14:58:46 ID:???
無は「無限」ですか?
185ご冗談でしょう?名無しさん:2010/03/22(月) 15:07:03 ID:???
そうだっていってんだろ
186ご冗談でしょう?名無しさん:2010/03/22(月) 15:43:03 ID:???
>>185
死ねカス
187ご冗談でしょう?名無しさん:2010/03/22(月) 15:51:28 ID:???
ここで質問をさせてもらいます。

1.なぜ、マイナス×マイナスはプラスになるのでしょうか?
2.なぜ、プラス(マイナス)×0は0になるのでしょうか?

物理板の皆さん、お願いですから、この質問に詳しく答えてください。
188ご冗談でしょう?名無しさん:2010/03/22(月) 16:11:38 ID:???
数学板逝けカス
189ご冗談でしょう?名無しさん:2010/03/22(月) 16:12:55 ID:???
やっぱり物理板はカスしか居ないのか・・・。
190ご冗談でしょう?名無しさん:2010/03/22(月) 16:13:00 ID:???
1.なぜ、マイナス×マイナスはプラスになるのでしょうか?
複素数を極座標表示すると、マイナスの数は位相がπであり、
位相がπの二数を乗じれば位相部分は加算されてπ+πで2π、つまりプラスになる。

2.なぜ、プラス(マイナス)×0は0になるのでしょうか?
複素数を極座標表示すると、0は絶対値が0であり、
絶対値が0の数に任意の定数を乗じても絶対値が0のために結果も0になる。
191ご冗談でしょう?名無しさん:2010/03/22(月) 16:14:10 ID:???
>>190
もうちょっと分かりやすく詳しく教えてくれカス。
192ご冗談でしょう?名無しさん:2010/03/22(月) 16:26:18 ID:???
極めて数学的に答えるならば「そういう(演算)規則だから」だ
この規則を人間が覚えやすいように意味付けするなり解釈付けるなりするのは
あんたの自由だ。しかしそれは決して本質的ではない。単なる後付けの意味付
けに過ぎん
193ご冗談でしょう?名無しさん:2010/03/22(月) 16:42:26 ID:???
数学 >>> 物理

が、確定した瞬間・・・。
194ご冗談でしょう?名無しさん:2010/03/22(月) 17:31:39 ID:???
これ確か回答が前書いてあったじゃん
代数的に環の公理から導けるよ
195ご冗談でしょう?名無しさん:2010/03/22(月) 17:41:11 ID:UDS5xUi2
代数幾何のこと?
196ご冗談でしょう?名無しさん:2010/03/22(月) 17:47:11 ID:???
カオスの勉強を始めた学部生なのですが、
ロジスティック写像のリターンマップというものについて簡単に調べたいのですが
初学者に優しいサイトとかありますでしょうか?
197ご冗談でしょう?名無しさん:2010/03/22(月) 17:48:48 ID:???
カオスは初心者に優しくない。
198ご冗談でしょう?名無しさん:2010/03/22(月) 17:54:30 ID:???
199ご冗談でしょう?名無しさん:2010/03/22(月) 20:16:51 ID:8K/WCemI
結局、お前は分かってない、啓蒙書読めだとか言われただけで
「どこがおかしいのか」を指摘してもらえなかったのでケムに巻かれた気分です。

安易なアナロジーということなら
宇宙は円盤状ではなくリング状だから中心は円周上のどこかに行ってしまったよ
だとの事ですが、そもそもリングと見なすのを前提で話してますよね?
高次元ユークリッド空間が何のことだかわからないけど、3次元空間で考えたらいいじゃないか。
いけませんか?

はっきり言って、おれの理解力が問題なんじゃなくて
あんたの説明能力が稚拙だからなんじゃないのですか?
それはなぜだか分かります?回答者も結局理解できてないんだと思う・・・・・・
200ご冗談でしょう?名無しさん:2010/03/22(月) 20:19:41 ID:???
だってお前バカだから
201ご冗談でしょう?名無しさん:2010/03/22(月) 20:22:07 ID:???
>>199
ユークリッド空間ぐらい調べてこいよ。
wikipediaに定義書いてるだろ。
http://ja.wikipedia.org/wiki/%E3%83%A6%E3%83%BC%E3%82%AF%E3%83%AA%E3%83%83%E3%83%89%E7%A9%BA%E9%96%93
202ご冗談でしょう?名無しさん:2010/03/22(月) 20:27:18 ID:8K/WCemI
うるせ〜。
ユークリッド空間の言葉の意味を調べるのはおれの宿題かも知れないけど
なぜここでユークリッド空間の概念が出てきたのか、その流れを説明するのは
あんたの仕事だろーが!!
もういいよ。とりあえず、読んでくるけど。
203ご冗談でしょう?名無しさん:2010/03/22(月) 20:27:42 ID:???
>>199
直線の中点はどこか?
204ご冗談でしょう?名無しさん:2010/03/22(月) 20:35:39 ID:???
>>198
もっと説明のあるサイトはないですか?
205ご冗談でしょう?名無しさん:2010/03/22(月) 20:35:45 ID:8K/WCemI
>>203
左端からと、右端からの、ちょうど同じ距離にある真ん中だ。
206ご冗談でしょう?名無しさん:2010/03/22(月) 20:38:01 ID:???
>>205
直線に左端も右端もない。
207ご冗談でしょう?名無しさん:2010/03/22(月) 20:38:51 ID:???
直線 y = x の中点の座標は?
208ご冗談でしょう?名無しさん:2010/03/22(月) 20:40:03 ID:???
209ご冗談でしょう?名無しさん:2010/03/22(月) 20:40:42 ID:???
>>205
では直線の端とは?
210ご冗談でしょう?名無しさん:2010/03/22(月) 20:42:36 ID:8K/WCemI
端のない直線なんて、おまえの頭の中にしかない。
数学屋は理想しか言わん。
おれは、現実の話がしたい。宇宙の話をしよう。
211ご冗談でしょう?名無しさん:2010/03/22(月) 20:45:40 ID:???
>>210
まさに宇宙がそうなのだが。端が無い。
212ご冗談でしょう?名無しさん:2010/03/22(月) 20:47:39 ID:???
>>210
現実の宇宙の端が見つかっているとでも?
213ご冗談でしょう?名無しさん:2010/03/22(月) 21:02:01 ID:8K/WCemI
ふと思ったんですが、宇宙はいま膨張してますよね。
それと共に、地球からの観測可能な球の半径も延びていってる。1秒間に30万kmで。

この星が離れていく速さが、光速を超えることってあるのですか?
あるとしたら、いま一番遠くに見えている境界線上の銀河って
明日には見えなくなるんじゃないですか?
それって、こわい。
214ご冗談でしょう?名無しさん:2010/03/22(月) 21:02:19 ID:???
他人に教えを乞うなら、自分の先入観は捨てろや、な?
215ご冗談でしょう?名無しさん:2010/03/22(月) 21:04:49 ID:8K/WCemI
>>214
・・・すいませんでした。
216ご冗談でしょう?名無しさん:2010/03/22(月) 21:08:50 ID:???
>>210
君の疑問点がいまいちつかめないんだが、

線はいつか終わるけど、考えてる範囲に比べて端が相当遠いとき、端はないものと見ても問題ない
というのが古代の直線の考え方。
視線(光路)というのは、どこまで行ってもまっすぐだと考える。
人が測量や測定を行うとき、便利なように、現実に用意できる棒やロープ(線分)を延長したものとして、
終わりのない線(=無限に長い線)という概念として、直線を生み出した。

もうひとつは、円周はどこから沿ってもいいし、どこまで沿ってもいい。始まりと終わりがなくて、端がない。
この端がないことを利用して、人類は周期的に同じことが起こっても問題ないものに限り、
直線の代替として、カレンダーや車輪や歯車を作り出した。

C1とR1は同相だよという話以前に、まずこの古代人の考え方は理解できるか?
217ご冗談でしょう?名無しさん:2010/03/22(月) 21:13:14 ID:???
>>214

偉そうな事言わずに教えてやれよ先生
218ご冗談でしょう?名無しさん:2010/03/22(月) 21:13:14 ID:???
身長150cm体重40kgくらいの女の子が10mくらいの高さから音速で地上に向かって
跳び蹴りをした場合、どのような被害が予想されますか?
219ご冗談でしょう?名無しさん:2010/03/22(月) 21:13:29 ID:8K/WCemI
>>216
理解できます。
カレンダーは12月31日が端じゃんなんていちゃもんも言いません。
220ご冗談でしょう?名無しさん:2010/03/22(月) 21:16:57 ID:8K/WCemI
>>218
足の骨がおれます。たぶん。
221ご冗談でしょう?名無しさん:2010/03/22(月) 21:21:56 ID:???
>>219
宇宙の中心というものを、
「膨張宇宙」としての考え方から見たときのことを考えよう。

カレンダーの考えが分かるなら、そのたとえで行く。
1年を円周上に目盛ったものと、線分上に目盛ったものは一緒だ。
当然、線分上のものは12/31の隣の目盛りが1/1となる。つまり、12/32と1/1は同じだ。
これは、時間しかない宇宙だ。

君の誕生日から前後182日の範囲を宇宙とすると、君は宇宙の中心で生まれたことになる。
もし、地球の自転と公転がどんどん遅くなっていったら(実際にそうなんだが)、
1日と1年はどんどん伸びていくので、宇宙は伸びる。
ただ、君が宇宙の中心であることには変わりない。

なお、私の誕生日は6/20だが、そこを宇宙の中心としても一緒だ。
長い時間経てば、君と私の間にある時間は伸びる。だが、お互い膨張の中心にいることは一緒だ。
222216:2010/03/22(月) 21:33:21 ID:???
>>219
ごめん、読み直して見当ハズレのたとえをしてたのを分かった。
S1とR1の同相の話じゃなくて、地球が宇宙の中心か、の話ね

君の主張「地球が宇宙の中心だと思う」だよね。
で、その気持ちは宇宙の膨張からの発想だと。
だが、宇宙が無限であるとはいえないから、地球が実質的にも宇宙の中心でないといけないと

すなわち、ビッグバンの爆心地が地球じゃないといけないのでは?というのがもともとの疑問?

それだと、
観測できる範囲は地球から光速×宇宙の年齢の距離までだということを前提に、
宇宙の年齢が進むごとに広がる地球から見た光路的な宇宙の果てと、
インフレーション宇宙論における膨張という意味での宇宙の膨張とをまず切り離して考えないといけない。
223ご冗談でしょう?名無しさん:2010/03/22(月) 21:37:39 ID:8K/WCemI
>>221
なんとなくわかります。
でも、6/20の半年前である1/20と、半年後の1/20は別物なんじゃないのですか?
年齢も一歳違うし。
それを言うためには、時間は月と日だけで決定されるという前提がなくちゃいけない気がする。
宇宙の範囲を182日だと勝手に言い切ってしまうのもなぜか分からないし。
224ご冗談でしょう?名無しさん:2010/03/22(月) 21:41:56 ID:???
>>223
> 宇宙の範囲を182日だと勝手に言い切ってしまう

地球から観測可能な範囲が制限されてるのは分かる? その理由も。分からないのなら調べろ。
225ご冗談でしょう?名無しさん:2010/03/22(月) 21:44:53 ID:8K/WCemI
地球から見た光路的な宇宙の果てと、インフレーション宇宙論における膨張の果てが
一致していると言うことはないんですか?

一番遠くにある星が130億光年くらいで、ビッグバンが起きたのも137億年前らしいですけど
これって、同じようなものなんじゃないのかな。
226ご冗談でしょう?名無しさん:2010/03/22(月) 21:50:25 ID:???
>>225
地球は太陽を中心に周回しているし、銀河系もまた回転してるし、銀河系が属する上位構造もグレート・アトラクターに向かって移動してる
って分かる?

それでも宇宙の中心は地球だと? ちなみにその中心は、地球のどの一点だと言いたいんだ?
227ご冗談でしょう?名無しさん:2010/03/22(月) 21:53:55 ID:???
盛り上がってるところ悪いが一つ質問させてくれ

ブラケットが状態ベクトルとそのエルミート共役を意味するって事はわかるんだが

<φ|ψ>が確率振幅を表すって理屈がどうにもわからん。
|ψ|^2dqと何か関係あるの?
228ご冗談でしょう?名無しさん:2010/03/22(月) 22:17:05 ID:???
>>227
周辺の理解度を確認させていただきたいが、

内積が積分を用いて定義されてるのは理解できてる?
229ご冗談でしょう?名無しさん:2010/03/22(月) 22:17:06 ID:???
>>227
そりゃもちろん関係ある。
必ず教科書に載ってると思うぞ。
230ご冗談でしょう?名無しさん:2010/03/22(月) 22:18:40 ID:???
状態をψに見出す確率の平方根の意味での確率振幅ではなくて
遷移振幅の意味 状態φ ← 状態ψ

用語は正確なところはわかんないけど
231ご冗談でしょう?名無しさん:2010/03/22(月) 22:18:47 ID:???
>>227
1.ブラケットはベクトルの内積
2.複素関数の内積は、共役複素数との積を取った時の定義空間全域での積分
3.共役複素数との積は、絶対値の二乗

という流れで理解できると思う。
普通は逆に習っていくんだが。
232ご冗談でしょう?名無しさん:2010/03/22(月) 22:30:43 ID:???
>>227
ケットベクトルは状態ベクトル
ブラベクトルはそのエルミート共役、つまり双対空間ベクトル、つまり内積が定義されてるなら、射影を意味する。

しかも、全空間の全射影で1になるように規格化されているから、そのまま確率になる。

うまくいえないけど、
エルミート共役を双対空間と見ることで、ものすごく簡単に書こうとしたのがディラック。
総和記号を省いちゃうアインシュタインくらい物ぐさなのが受けた。
しかも、どっちかというと行列力学。場の話にあってたのでもっと受けた。
233ご冗談でしょう?名無しさん:2010/03/22(月) 22:38:45 ID:???
ユニタリ変換と直交変換の違いは、
相手とするベクトル空間が、実数上か複素数上かの違いだけですか?
234ご冗談でしょう?名無しさん:2010/03/22(月) 22:44:43 ID:???
227です。
皆さん色々とありがとうございました。
シッフとサクライあたりでもう少し調べてみますね
235ご冗談でしょう?名無しさん:2010/03/22(月) 23:15:00 ID:8nEUiMF9
第2量子化について

生成演算子や消滅演算子は位置や運動量で構成された演算子なのでしょうか?

つまり運動エネルギーとしてハミルトニアンにでてくるラプラシアンが生成演算子に作用しないのでしょうか?

教科書では消滅演算子aや生成演算子a^†は普通に作用していないようなのですが、

調和振動子の消滅演算子aや生成演算子a^†は
http://ja.wikipedia.org/wiki/%E8%AA%BF%E5%92%8C%E6%8C%AF%E5%8B%95%E...
のように位置qや運動量pの関数になっているので…。

236ご冗談でしょう?名無しさん:2010/03/22(月) 23:29:00 ID:???
古典場から量子場への道とよむといいよ
237235:2010/03/22(月) 23:31:32 ID:8nEUiMF9
H=∫dr[h^2/(2m)∇Φ^*∇Φ+V(r)Φ^*Φ]
とおいて
Φ=Σ[i]φ_ia_i
とおいて
代入すると
H==∫drφ^*(-h^2/(2m)∇^2+V(r))φa^†a
となるらしいのですが、aやa^†は∇に関係してこないのかを教えてほしいです。
238ご冗談でしょう?名無しさん:2010/03/23(火) 00:29:56 ID:???
>>237
> Φ=Σ[i]φ_ia_i

はもう少し具体的に何で展開した?
239ご冗談でしょう?名無しさん:2010/03/23(火) 00:33:10 ID:Wilgc0wE
http://detail.chiebukuro.yahoo.co.jp/qa/question_detail/q1310536438

ここを見ると電子の移動速度はものすごく遅い(0.1mm毎秒)ようなのですが、
電流が電子の流れ(の反対方向)という言い方は不適切ではないですか。

電流は光子の移動であって、光子が移動する結果電子も移動するということではないのですか?
240ご冗談でしょう?名無しさん:2010/03/23(火) 00:43:29 ID:???
>>239
電線の中を光子が移動する?って何で急にそんな画期的な事思いついちゃったんだ?
リンク先には光子のこの字も出てないのに
241ご冗談でしょう?名無しさん:2010/03/23(火) 00:45:48 ID:???
>>239
電流は電荷の流れで、光子は電荷ないです

流れそのものは遅くても流れ出す先頭位置が光速で移動してくれたらまあ速く見えるよね
242ご冗談でしょう?名無しさん:2010/03/23(火) 00:49:41 ID:???
ちょうどいいから便乗して質問

二準位系に光子を入れた場合、そこから放出される光は光子のものなの?
それともポンピングされた電子がエネルギーを放出してると解釈するべきなの?
243ご冗談でしょう?名無しさん:2010/03/23(火) 00:53:32 ID:???
何が放出しようとも電磁場にいったエネルギーは光子でしょ
244ご冗談でしょう?名無しさん:2010/03/23(火) 00:57:32 ID:???
>>237
場の振幅が調和振動子の例での位置qで
その時間変化が調和振動子の運動量pに対応してるので

この場合はaやa^†は∇に関係してこない
245239:2010/03/23(火) 01:23:40 ID:Wilgc0wE
>>241
全体の動きは遅くても、トコロテンみたいに、押せばすぐに出口からでると言うことですね。
よくわかりました。
246ご冗談でしょう?名無しさん:2010/03/23(火) 12:37:25 ID:32fd83FQ
経済指標、INDEXを代数幾何で解けるか?代数幾何の方程式で解けるか?
247ご冗談でしょう?名無しさん:2010/03/23(火) 14:27:27 ID:???
放送大学の、生井澤寛という人は、有名な人なんですか?
248ご冗談でしょう?名無しさん:2010/03/23(火) 20:25:12 ID:???
シュレーディンガー形式の波動関数が時間に依存して変化するというのはイメージがわくんですが
ハイゼンベルグ形式の演算子が変化するって言うのがいまいちイメージがわきません

<φ|A|ψ>と書いた時Aが変化すると言うことなんでしょうか。わけわからん。φとψはそのまま?
249ご冗談でしょう?名無しさん:2010/03/23(火) 20:34:26 ID:???
確率微分方程式とカオスというのは、どのように関係があるのでしょうか?
250ご冗談でしょう?名無しさん:2010/03/23(火) 21:04:24 ID:???
>>248
> <φ|A|ψ>と書いた時Aが変化すると言うことなんでしょうか。わけわからん。φとψはそのまま?

そう。
時間発展の因子をAに押し付ける形式。
2つの形式の対応の説明はそれなりの教科書にはきちんと書かれているから、読めば分かると思うけど
251ご冗談でしょう?名無しさん:2010/03/23(火) 21:04:56 ID:???
プラズマのHモードについて分りやすく説明して。
ググっても難しい話ばっかで良く分らん。
252ご冗談でしょう?名無しさん:2010/03/23(火) 21:19:46 ID:3TqN3MJA
↓科学技術政策に関する意見を募集しているようです。

http://www.mext.go.jp/b_menu/houdou/22/03/1291303.htm

科学技術政策に関するご意見募集について

平成22年3月10日

社会・国民とともに推進する科学技術政策の実現に向けて、皆様からのご意見を募集します。本意見募集の結果は科学技術週間中の4月17日に
行われるシンポジウムにおいて活用させていただくとともに、その成果とあわせて、今後、文部科学省として、より良い科学技術政策を推進していくために参考とさせていただきたいと考えております。
科学技術の力による輝きのある日本の実現に向けて皆様のご意見をお寄せ下さい


日本が科学技術を推進することの意義や必要性とは何であるとお考えになりますか。
日本や世界は、地球温暖化、資源・食料・エネルギー問題、経済危機、医療・福祉問題など様々な問題に直面していますが、科学技術を活用してどのような問題を解決してほしいとお考えになりますか?
科学技術によって、生命や宇宙の理解などの知的探究、宇宙の開発・利用、海洋探査など、人類にとって新たな挑戦が可能になると考えられますが、
これからの未来に向けて、どのようなことに挑戦してほしいとお考えになりますか?
科学技術を推進していくうえでは、大学における基礎的な研究活動の充実、小・中学校における理数教育の充実、研究者や政策担当者と
社会との間の相互理解など、必要なことがらはたくさんありますが、特に重点を置いて取り組む必要があるものは何だとお考えになりますか?
科学技術に関する国の予算や投資のあり方、目標・計画の立て方や評価のあり方、各省庁間の連携のあり方など、科学技術政策の進め方について、
改善すべきと考えられる点はどのようなことだとお考えになりますか?
その他、科学技術・学術審議会基本計画特別委員会がとりまとめた提言(我が国の中長期を展望した科学技術の総合戦略に向けて−ポスト第3期科学技術基本計画における
重要政策−中間報告)や科学技術政策に関することなど、ご意見・ご感想がありましたらお寄せください。
253ご冗談でしょう?名無しさん:2010/03/24(水) 00:46:28 ID:???
体重計に乗ると60kgって出るじゃないですか
あれは重量ですか?質量ですか?
単位がkgだし、高校物理で質量mに何か値を代入するときに
確か現実の天秤や測定器で測った値を代入した記憶があるので、たぶん質量でないかなと予想は付くのですが
月で同じ人が体重計に乗ったらたぶん10kgって表示されますよね
質量は変わらないはずだから60kg?なんでしょうが、月で表示されるのは10"kg"
そんなことを考えてたら頭の中がこんがらがってしまいました
どなたか頭のなかの整理を手伝って頂けないでしょうか
254ご冗談でしょう?名無しさん:2010/03/24(水) 01:22:05 ID:???
それは重量のこと
体重計の単位はkgとなっているが、これは質量のkgではなく、重量のkgwの意味
255ご冗談でしょう?名無しさん:2010/03/24(水) 10:12:04 ID:???
>>254
地球で測っても月で測っても重量なのですか?
となると、質量を測るにはどうすれば良いのでしょうか
256ご冗談でしょう?名無しさん:2010/03/24(水) 10:43:15 ID:ALVkt2Qi
それは意外に質量の定義にも関係する深いテーマ。
重さ(重量)とはある星の上でその星の重力によって受ける力のこと。
だから、質量が1kgでも星によっては1/5kgwになったり、逆に5kgwになったりする。
でも物体そのものに変わりはないわけで、その物体の”大きさ”の1つが質量と呼ばれる。
この質量の定義には2つあることになってる。
1つは慣性質量。もう1つは重力質量だったか。まぁ面白いテーマだわな。
257ご冗談でしょう?名無しさん:2010/03/24(水) 10:51:05 ID:???
>>255
力と加速度を測定してm=F/aから求めるのが定義通りの求め方。
しかしこれはめんどくさいので簡易的には天秤で測る。これなら
地球で測って1kgの分銅とつりあえば、月で測っても1kgの分銅とつりあう。
重力の違うところで資料と分銅は同じ割合で増減するので、重力の違いは
キャンセルされ、資料だけの量が引き出せる
258ご冗談でしょう?名無しさん:2010/03/24(水) 11:21:08 ID:???
>>256
まさに>>257さんの測定・計算をすることによって慣性質量が分かるわけですね
直接絶対的な質量を測ることはできない?ということは分かった気がします

>>257
1kgの分銅というのはどうやって測定されたものなのでしょうか
地球上で体重計に乗せて、1kgwと表示された 程度のことなんですかね
259ご冗談でしょう?名無しさん:2010/03/24(水) 11:25:06 ID:???
変動の大きい重量で計るバカがいるか?
260ご冗談でしょう?名無しさん:2010/03/24(水) 11:27:06 ID:???
>>258
キログラム原器 でぐぐれ
261ご冗談でしょう?名無しさん:2010/03/24(水) 11:59:06 ID:ALVkt2Qi
>>258
重力質量の方だろうけど、結構細かいんだよね。
重力質量は万有引力による引力の大小が質量によっていることから
質量を重力に基づき定義してるんだったかと思う。
ただ自転する天体上の物体は遠心力が働くから、その分を合わせた力になるのかな。

以下に結構詳しく書かれてるから、参考に。
深い話だというのがわかる。
http://ja.wikipedia.org/wiki/%E9%87%8D%E5%8A%9B
262ご冗談でしょう?名無しさん:2010/03/24(水) 12:03:23 ID:???
>>258
>直接絶対的な質量を測ることはできない?ということは分かった気がします
ちょ、おま。257の最初の方法で測れる。めんどくさいし精度も出ないというだけ。

>1kgの分銅というのはどうやって測定されたものなのでしょうか
この物体の質量を1kgとしましょう、というふうに取り決められた物体がある。
263ご冗談でしょう?名無しさん:2010/03/24(水) 12:07:57 ID:???
質問者は知ってて質問しているような
264ご冗談でしょう?名無しさん:2010/03/24(水) 13:21:37 ID:???
時間というのは、空間の要素なんですよね?
なら、時間というのは、空間のことを表していると考えても間違いではないのではないでしょうか?
265ご冗談でしょう?名無しさん:2010/03/24(水) 13:32:33 ID:???
>時間というのは、空間の要素なんですよね?
違います
266ご冗談でしょう?名無しさん:2010/03/24(水) 13:37:03 ID:???
>>265
じゃあ、何なの?
267ご冗談でしょう?名無しさん:2010/03/24(水) 15:46:30 ID:???
時間か・・・無定義語に近いものがあるんだよな
様々な系の状態は時系列的に変化していくわけだが、その瞬間と瞬間の間の尺度とでも言うか
まあ、相対性理論では時間と空間を合わせて4次元時空として扱うが、時間成分と空間成分が持つ意味や扱いは違う
268ご冗談でしょう?名無しさん:2010/03/24(水) 17:18:16 ID:???
 
吸血寄生虫 在日朝鮮人   【生活保護】 寄生虫!

在日(ざいにち)は 三日やったら やめられない!

   貴重な血税から 毎月二十数万円もらって

   遊んで暮らしている 吸血寄生虫 在日朝鮮人

在日朝鮮人なら 別に収入あっても 【生活保護】もらえる

【生活保護】もらいながら 豪邸に住み 高級外車に乗る

 / ̄ ̄ ̄ ̄ ̄ ̄ ̄ ̄ ̄ ̄ ̄ ̄ ̄ ̄ ̄ ̄ ̄ ̄ ̄ ̄ ̄
│        【生活保護】 もっと出せー!
│ 
│  早く選挙権よこせー!  【民主党】に投票しろー!
│ 
│ 【民主党】に投票しろー! 【民主党】に投票しろー!
 \_ ____________________
    V
  ∧_∧ 在日特権 るんるん〜♪
 <丶`∀´>      
 ( 貴 族 )       
 | | |       【社会保険料】
 〈_フ__フ      【税金税金】
【生活保護】       ( T_T )
【二十数万】       ( 奴 隷 ) 日本人は
【生活保護】      | | |  つらいよ〜
【二十数万】      〈_フ__フ 
■■■■■■■■■■■■■■■■■■
269ご冗談でしょう?名無しさん:2010/03/24(水) 18:34:29 ID:???
ミンコフスキー時空では時間のまがってんの?
270ご冗談でしょう?名無しさん:2010/03/24(水) 18:55:18 ID:jsD4vpla
まがってないよ
271ご冗談でしょう?名無しさん:2010/03/24(水) 20:55:18 ID:???
波動方程式のラグランジアンの形を調べているのですが、
どのような形になるのでしょうか?
272ご冗談でしょう?名無しさん:2010/03/24(水) 21:04:18 ID:???
ちなみに
(φの座標に関する2階微分)-定数(φの時間に関する2階微分)=0
という簡単な形式の波動関数を導くラグランジアンを調べています
273ご冗談でしょう?名無しさん:2010/03/24(水) 21:49:20 ID:???
光子って質量0なんですよね?
新しい宇宙船の概念でソーラーセイルって太陽から離れたら
レーザー光線をあてるとかいう話をディスカバリーで見たんですが
質量0なのに進むんですかね??
274ご冗談でしょう?名無しさん:2010/03/24(水) 21:51:01 ID:???
>>273
質量は0でも運動量はある
275ご冗談でしょう?名無しさん:2010/03/24(水) 21:56:11 ID:???
>>274
押せますかね?質量0で
276ご冗談でしょう?名無しさん:2010/03/24(水) 22:04:47 ID:???
電磁波の圧力はもう100年以上前に実験的に確認されている
ロシアのLebedevって人の実験
277ご冗談でしょう?名無しさん:2010/03/24(水) 22:13:43 ID:???
>>272
ラグランジアン密度が ((∂φ/∂t)^2 - (定数)(∂φ/∂x)^2) の定数倍
278ご冗談でしょう?名無しさん:2010/03/24(水) 22:20:54 ID:???
>>275
「押す」のは質量じゃなくて運動量のやり取り。
279ご冗談でしょう?名無しさん:2010/03/24(水) 22:24:27 ID:???
>>275
だから光子から運動量を受ければそれはdp/dtの力で押されるのと同じ。
光子が運動量を持っていさえすればよく、質量の有無は関係ない。
280ご冗談でしょう?名無しさん:2010/03/24(水) 23:22:04 ID:???
密度行列がさっぱりわからん

|ρ>A<ρ|とか書かれても何を意味しているのかさっぱり…
281ご冗談でしょう?名無しさん:2010/03/24(水) 23:37:07 ID:???
壁に穴があると光を通しますが光が通る最低の直径はいくらですか?
282ご冗談でしょう?名無しさん:2010/03/24(水) 23:51:53 ID:???
0。穴が無くても浸透する。
283ご冗談でしょう?名無しさん:2010/03/24(水) 23:55:00 ID:???
>>280
これは違うだろw
284ご冗談でしょう?名無しさん:2010/03/24(水) 23:58:41 ID:???
密度行列がさっぱりわからん

|ρ>A<ρ|
285ご冗談でしょう?名無しさん:2010/03/25(木) 00:14:37 ID:???
これはすっごく困ってます ってAAか?
286ご冗談でしょう?名無しさん:2010/03/25(木) 00:20:12 ID:???
もうAAにしか見えないw
287ご冗談でしょう?名無しさん:2010/03/25(木) 06:46:46 ID:KiVyDch9
放射圧って、マックスウェル電磁気学を理解できないと
理論的な理解は難しい感じだな。
288ご冗談でしょう?名無しさん:2010/03/25(木) 17:30:53 ID:oa1q2sz9
以前、知恵袋で質問した問題です(マルチポスト&未解決終い)

電荷Qが蓄えられたコンデンサー(電気容量C)に、電気抵抗(R)とコイル(インダクタンスL)を直列につなぐ。
この回路のその後の電流Iの時間変化を求めよ。
この問題について、最初の電圧の式(のみ)が良く分かりません。
参考書には
L(dI/dt)+RI+Q/C=0とありましたが、どう考えても
L(dI/dt)+RI-Q/C=0にしかなりません
(コンデンサー(放電状態I=-(dQ/dt))を、電池のようなものとして考えたのですが・・・・(V=L(dI/dt)+RI))
正しい考え方を教えて下さい
289ご冗談でしょう?名無しさん:2010/03/25(木) 18:19:42 ID:???
>>288
その参考書では
> 放電状態I=-(dQ/dt)
なの?
290ご冗談でしょう?名無しさん:2010/03/25(木) 18:19:51 ID:ITOKax0Z
そのぐらい自分で考えろよ>288

符号だけだろ
電流や電荷を適当にとれば式なんかどうにでも立つだろ
291ご冗談でしょう?名無しさん:2010/03/25(木) 18:45:18 ID:???
>>289
I=-(dQ/dt)は、私の考えです
参考書では、どうもI=(dQ/dt)としているようです
私の式を二階微分方程式にすると、参考書の二階微分方程式に一致します

>>290 sorry
292ご冗談でしょう?名無しさん:2010/03/25(木) 19:11:49 ID:???
>>280 本当にそんな式あった?それは密度行列とは関係ないよ、とマジレス
してみる。
293ご冗談でしょう?名無しさん:2010/03/25(木) 20:18:39 ID:???
>>291
電流の向きを放電時に電流が流れる向きにとればI=-(dQ/dt)だし、
充電時に流す向きにとればI=+(dQ/dt)。それだけのこと
294ご冗談でしょう?名無しさん:2010/03/25(木) 20:58:17 ID:???
>>293
なるほど!どうやら一方の考え方に固執していたみたいです
ありがとうございました
295ご冗談でしょう?名無しさん:2010/03/25(木) 21:45:59 ID:???
初心者ですみませんがお願いします
ヤング率とバネ乗数についてです。
SS400のヤング率は205800MPa(N/mm2)と思うのですが
これをバネ乗数(N/mm)に換算したいのですが、どういう公式を使えば良いのでしょうか?
断面積が分からないとバネ乗数を求める事はできませんよね?

宜しくお願いします
296ご冗談でしょう?名無しさん:2010/03/25(木) 21:53:47 ID:???
>>295
バネの形状と寸法を決めてから機械・工学板で質問しろ
297ご冗談でしょう?名無しさん:2010/03/25(木) 22:48:01 ID:???
>>292
ただの書き間違いだ、許してくれよいい加減w
298ご冗談でしょう?名無しさん:2010/03/25(木) 22:52:49 ID:???
電磁場テンソルをマクスウェル方程式に埋め込む方法がわかりません

というかアレは場を記述するテンソルであってマクスウェル方程式と直接の関係はないのでしょうか
299ご冗談でしょう?名無しさん:2010/03/25(木) 23:15:48 ID:???
>>298
電磁場テンソルて何?
300ご冗談でしょう?名無しさん:2010/03/26(金) 02:25:03 ID:vv6UZeqm
wikipedeaの「マクスウェルの方程式」の
「マクスウェルの方程式と特殊相対性理論」に書いてある。
301ご冗談でしょう?名無しさん:2010/03/26(金) 02:27:05 ID:???
おっとスペルミス
wikipedea→wikipedia
302ご冗談でしょう?名無しさん:2010/03/26(金) 02:46:08 ID:???
>>300
ありがとうございます。
ウィキペディアにしてはやたらと説明が詳しいですね
303ご冗談でしょう?名無しさん:2010/03/26(金) 05:50:41 ID:???
クォークの粒子だけで単独で存在できる原理はどんな
説明なんでしょうか?
304ご冗談でしょう?名無しさん:2010/03/26(金) 06:07:22 ID:???
305ご冗談でしょう?名無しさん:2010/03/26(金) 11:26:57 ID:???
ふーん、中性子星以上BH未満だからクォーク星か
じゃ本当にぎぎりぎりでBH未満の星なら超弦星もありうるのかな
306ご冗談でしょう?名無しさん:2010/03/26(金) 12:16:36 ID:???
電荷のない粒子は例えば
中性子=反中性子
なんでしょうか
307ご冗談でしょう?名無しさん:2010/03/26(金) 12:28:37 ID:???
>>306
> 中性子=反中性子
ではない。
308ご冗談でしょう?名無しさん:2010/03/26(金) 12:43:54 ID:???
>>307
何が違うのか教えてください
309ご冗談でしょう?名無しさん:2010/03/26(金) 12:49:50 ID:???
たぶん話の流れ的に中性子を構成するクオーク≠反クオークが変わってるということなんだろうが、
中性子の特性も何か変わるんだっけ?
310ご冗談でしょう?名無しさん:2010/03/26(金) 12:56:21 ID:???
過去スレにあるミルクでつくったやつを4日熟成して、をれを焼いて、牛乳で作ったやつ(1日熟成したやつ)に入れてよくかき混ぜ、定期的に食べると、永遠超えて生きられるよ
登記
311ご冗談でしょう?名無しさん:2010/03/26(金) 12:59:47 ID:???
登記さんは面白いと思ってやってるのかね?
312ご冗談でしょう?名無しさん:2010/03/26(金) 13:53:18 ID:???
>>306
粒子と反粒子が同じものもあれば違うものもある。例えば光子は粒子と反粒子が同じ。
中性子は違う。

>>309
何を指して特性と言っているかによる。質量などは変わらないけど、
当然何から何まで同じというわけにはいかない。
313ご冗談でしょう?名無しさん:2010/03/26(金) 14:12:04 ID:???
物体からは光が出ていて、それを観測することでそれの存在が分かるんですよね?
あ、この場合の”光”は車のライトのような物ではありませんよ。目で見えない、物体から発せられる光です

   物体から出てる光
A←----------------B

じゃあ、例えば直線上にA地点からB地点があるとしてA地点が観測者とする。当然AはBを観測する事ができる
     
       光
A←----------------B
       ↑
       自分

では、その線の垂直に居る自分はこの光を見てBが何であるかを観測できないのは何故ですか?
例えば、

       光
      鏡/----------B
       ↑
       自分

上記のような場合だとBを当然観測出来るわけなんですが
314ご冗談でしょう?名無しさん:2010/03/26(金) 14:36:31 ID:???
>あ、この場合の”光”は車のライトのような物ではありませんよ。目で見えない、物体から発せられる光です

根本から勉強し直したほうがいい。
315ご冗談でしょう?名無しさん:2010/03/26(金) 14:56:40 ID:???
部屋にある理科関係の本を全て廃棄して明日から小説や歴史のお勉強を沢山しましょうね(はぁと
316ご冗談でしょう?名無しさん:2010/03/26(金) 16:28:27 ID:???
ぶっちゃけ何を疑問に思っているかさえ理解できない>>313
317ご冗談でしょう?名無しさん:2010/03/26(金) 17:26:43 ID:???
>>312
光子が同じ事は分かりました
中性子は何が違うのでしょうか・・・
318ご冗談でしょう?名無しさん:2010/03/26(金) 17:36:08 ID:???
>>317
反中性子は陽子と反応すると最終的にレプトンだけの状態になりうるが、
中性子と陽子の反応ではそれは起こり得ない、など
319ご冗談でしょう?名無しさん:2010/03/26(金) 17:51:14 ID:???
>>317
例えばwikipediaには、反中性子の磁気モーメントは中性子の反対であり、中性子の-1.91 μNに対して1.91 μNである[2]。ここで、μNは核磁子を表す。とあります
320ご冗談でしょう?名無しさん:2010/03/26(金) 19:04:33 ID:???
>>318
それは何に起因するものなんですか?
中性子と反陽子なら対称的な反応になるのでしょうか

>>319
磁気モーメントが逆・・・スピンが逆になる感じですかね・・・
321ご冗談でしょう?名無しさん:2010/03/26(金) 19:37:07 ID:???
>>320
バリオン数という、知られている相互作用では保存する量があって、
陽子や中性子はそれが+1、反陽子や反中性子は-1
322ご冗談でしょう?名無しさん:2010/03/26(金) 19:56:57 ID:???
バリオン数とあとアイソスピンも逆だな
323ご冗談でしょう?名無しさん:2010/03/26(金) 20:12:58 ID:YamOWcqt
ブラジルナッツ効果って今初めて聞いた言葉だけど、
常識的に大きい面積の物より小さい面積の物が隙間を埋めるから
重力で下を埋めるために小さいのが下へ大きいのが上へっていう理屈だと思ってた
そしたら難しい学術で解明されてないとかあっちこっちに珍しそうに書いてる人ばっかりいるんだけど・・・
http://www.myeclipseide.jp/blog/2009/05/brazil-nut-effect.html

この理解じゃだめなの?
何か日本人頭悪くない?マジすげぇびっくりしたんだけど
324ご冗談でしょう?名無しさん:2010/03/26(金) 20:15:25 ID:YamOWcqt
っていうか、ピーナッツでも塩が下に残ってるじゃん?これだってブラジルナッツ効果の一つでしょ?
325ご冗談でしょう?名無しさん:2010/03/26(金) 20:25:42 ID:YamOWcqt
すげぇやっぱみんなアホだなすぐ教えて貰えると思ったら解答が来ない
俺高卒なのに常識だと思ってた
解答はしらないから間違ってたらののしっていいよwじゃさいなら
326ご冗談でしょう?名無しさん:2010/03/26(金) 20:27:38 ID:???
気の短い人だな
327ご冗談でしょう?名無しさん:2010/03/26(金) 20:29:49 ID:???
基本的にはエネルギー極小(に近い)状態ってことでそうなるんだと思うけど
328ご冗談でしょう?名無しさん:2010/03/26(金) 20:30:20 ID:???
書き込んでから捨て台詞で立ち去るまで13分って
329ご冗談でしょう?名無しさん:2010/03/26(金) 20:36:29 ID:???
場の古典論を読んでるんだけど、皆あの本読む時計算はどこまで確認するんだ?

俺はある程度までは暗算で済ませて暗算できない時は結果だけ眺めてるんだが…
330ご冗談でしょう?名無しさん:2010/03/26(金) 21:30:17 ID:jh0VQwA3
>>328 きっとかれは生粋の江戸っ子だんだ!これで万事説明がつく。
331ご冗談でしょう?名無しさん:2010/03/26(金) 21:36:09 ID:???
>>327
エントロピーだな。
332ご冗談でしょう?名無しさん:2010/03/26(金) 21:39:13 ID:???
犬だから既に出来レース
最初から豪快に燃えてるF35に金出せよって話だろう
日本国民やってりゃ誰も出わかるw
333ご冗談でしょう?名無しさん:2010/03/26(金) 22:37:54 ID:???
>>331
え、どうだろ
この話はエントロピー増大で理解できるのか?
揺さぶってる状態は孤立系でもないし、ちょっと違うような
俺はよく分からん
334ご冗談でしょう?名無しさん:2010/03/26(金) 22:43:48 ID:???
グランドカノニカル分布考えればいいんじゃないの?
335ご冗談でしょう?名無しさん:2010/03/26(金) 23:10:31 ID:???
>>334
うーん、この系はやるとすれば、カノニカルでいいと思うんだが
こういう有限な大きさを考えて、しかも大きさが多種の系は
複雑な拘束条件付きの問題になる
数値とかでやってやれないことは無いだろうけど相当大変だ

そもそも熱的な系って、マクロには(準)熱平衡的な意味で同じでも
常にミクロには違う状態間で経巡るような系だからな
熱的効果が本質的に効いてるようには思えない

小さいナッツが下に集まっているような状態の方がエントロピーが
大きいのか、俺は直感的によく分からない
まあ、俺は>>327程度の回答しか思いつかないので、もっと
理解してる識者が居たら補足願う
336ご冗談でしょう?名無しさん:2010/03/27(土) 00:12:57 ID:???
たかが数十個のナッツの配置が統計力学で議論できるわきゃーない。
337ご冗談でしょう?名無しさん:2010/03/27(土) 03:29:15 ID:???
粒子の対消滅対生成で保存されるもの・されないものってそれぞれ何でしょうか?
例えば電子陽電子ならスピンはどちらも1/2なので保存されないですよね?
338ご冗談でしょう?名無しさん:2010/03/27(土) 03:53:10 ID:???
>>337
スピンというか、全角運動量は保存される
あと代表的なものを挙げると、エネルギー、運動量、電荷、カラー、レプトン数、バリオン数など
339ご冗談でしょう?名無しさん:2010/03/27(土) 06:58:56 ID:Vzs0+MVM
>>327が正しい。みんな熱力学の教科書ちゃんと読もう。e.g. プリゴジン。
340ご冗談でしょう?名無しさん:2010/03/27(土) 07:21:55 ID:uYyUSgjZ
>>313
実際にそういうのをはっきり体験できるのってあるのかわからないが
何か関連する発明が最近ニュースか何かになってなかったっけ。。。
ある方向からは見えないって奴だったような。
まぁ規模がでかいので言えば、ブラックホールが一例だろう。
一番いいのは宇宙でレーザー放って、レーザーを側面から見ることだろうけど。
地上だと大気で散乱するから、よくないしね。

身近にあるもので液晶画面も関係あるかな。
あれは光の強さを確保するために光の方向を偏向膜みたいので制御してるみたいだから
向きを変えると、一定の波長の光が見えなくなる。
だから、液晶を斜めに見ると本来どおり見えなくなる。
これも明暗っていう単純な事例ではないものの、
ある色光が目に到達しないことで見えない例の1つといえるんじゃないかな。
341ご冗談でしょう?名無しさん:2010/03/27(土) 10:50:52 ID:???
オイラーラグランジュの方程式に、運動エネルギーとポテンシャル
エネルギーであらわされるラグランジアンを代入すると、
運動方程式がでてきますが、これは、運動方程式がエネルギー
という概念を使うことによって、導出されたと考えていいのでしょうか?
それとも、運動方程式の記述の仕方をちょっと変えただけと
考えるべきでしょうか?
342ご冗談でしょう?名無しさん:2010/03/27(土) 11:19:34 ID:???
>>338
それらは保存されるのですよね
電子陽電子が対消滅したときには全角運動量は光子の角運動量になるんですか?
343ご冗談でしょう?名無しさん:2010/03/27(土) 14:13:43 ID:???
>>342
そうだね、反応後の光子系の全角運動量になる
ちなみに
e+ + e- → nγ
においてn=1はありえない
最低でもn=2個の光子が生成される
344ご冗談でしょう?名無しさん:2010/03/27(土) 15:49:05 ID:???
>>343
あれ、非線形光学過程考えれば周波数二倍の光子が出てくるんじゃね?
345ご冗談でしょう?名無しさん:2010/03/27(土) 16:25:17 ID:???
>>343
> e+ + e- → nγ
> においてn=1はありえない
これはエネルギー運動量保存則
346ご冗談でしょう?名無しさん:2010/03/27(土) 17:54:57 ID:???
>>343
ところで全角運動量ってなんだっけ
標的からの距離×運動量?
347ご冗談でしょう?名無しさん:2010/03/27(土) 19:03:48 ID:???
なんか某スレでけんかが起こってるからこっちで聞くけど
大気の高さが10倍になったら気圧も10倍になるの?ならないの?
348ご冗談でしょう?名無しさん:2010/03/27(土) 19:17:56 ID:???
空気を水に置換
349ご冗談でしょう?名無しさん:2010/03/27(土) 19:32:26 ID:???
某スレってどこ?
見たい
350ご冗談でしょう?名無しさん:2010/03/27(土) 19:55:52 ID:???
水は圧力上がっても容積ほとんど変わらないから高さと圧力はほぼ正比例的だろうけど、
気体は事情が違うんじゃないの?
351ご冗談でしょう?名無しさん:2010/03/27(土) 20:35:51 ID:???
そもそも大気の高さって決められるの?
352ご冗談でしょう?名無しさん:2010/03/27(土) 20:40:15 ID:???
統計力学的な系を仮定して実際に計算してみるのが一番なんだろうけど
俺は統計力学が苦手だから誰か他の人頼む

大気中の分子数をNとしてその後どうすりゃいいんだOTL
353ご冗談でしょう?名無しさん:2010/03/27(土) 20:50:03 ID:???
昔そんな微分方程式を見た気がする
高さh[m]の微小部分をdhとするみたいな
354ご冗談でしょう?名無しさん:2010/03/27(土) 20:55:13 ID:???
大気の高さなんて、地上から濃度××とか適当に閾値を定義して議論するしかないな
ハミルトニアン
H = p^2/2m + mgz
でカノニカルアンサンブルで計算したら、まぁ議論は出来ると思うが・・・
何の本か忘れたが、そういう問題はやった事はある
355ご冗談でしょう?名無しさん:2010/03/27(土) 21:28:19 ID:???
真面目に統計力学で計算したら、
空気は全部宇宙にすっとんでいってしまうぞ。
356ご冗談でしょう?名無しさん:2010/03/27(土) 21:40:04 ID:ImFu89ZA
おれは今とても怒っています。

先週、ここで質問をしたのですが
1、宇宙は無限である(端がない)
2、円にも端がない
3、よって宇宙は円である
だとかいう解答を頂きました。

このレスをしてくれた方は、おそらくニコニコと答えてくださったと思うのですが
おれはその顔面にたんを吐きかけてやりたいです。
結局、宇宙の膨張速度が光速を超えることがあるのか?という問いもスルーされてるし。
どうなんでしょうか?
357ご冗談でしょう?名無しさん:2010/03/27(土) 21:45:02 ID:???
フンフン。
それで?
358ご冗談でしょう?名無しさん:2010/03/27(土) 21:49:35 ID:???
>>356
そもそもトポロジカルな円だから貴様のイメージしてる円とは全然違うと思うぞ
359ご冗談でしょう?名無しさん:2010/03/27(土) 21:53:10 ID:???
>>356
どんな質問かを書けよ
一応確認しておくが、円と言われて二次元的な円を思い浮かべてないよな?
360ご冗談でしょう?名無しさん:2010/03/27(土) 21:56:49 ID:ImFu89ZA
トポロジカルな円ってなんなんですか?
円って言ったらまんまるワッカだ。トポロジカルな円はどんなんだ?

ていうか、そんな御幣を生みやすい単語を用いる方がおかしいんだよ・・・・・・
まぎらわしい!

質問は、宇宙の中心は地球じゃないのですか?ってやつです。
180あたり。
361ご冗談でしょう?名無しさん:2010/03/27(土) 22:00:01 ID:???
地球以外も含めて宇宙の中心と言えるような特別な点が存在すると言える根拠は
存在しないというのが現時点での一般見解
362ご冗談でしょう?名無しさん:2010/03/27(土) 22:03:12 ID:???
まどろっこしい日本語になっちまった、まぁ分かると思うが
363ご冗談でしょう?名無しさん:2010/03/27(土) 22:10:18 ID:???
>>360
三次元多様体のことだ。
ぐぐればでてくるはず
364ご冗談でしょう?名無しさん:2010/03/27(土) 22:11:23 ID:ImFu89ZA
ええと、ですね。
もし宇宙の膨張速度が光速を超えるならば、今の地球からの観測可能な範囲外にも星々が広がっていると
納得できるんです。

じゃなくて、光速の制限を受けるならば、ビッグバンの衝撃波前線みたいなもんと、地球からの観測可能半径が一致しますよね。
だったら、それが宇宙の果てと言ってもいいんじゃないですか?
今の地球が属しているビッグバンとは別のビッグバン宇宙があるとか言うのなら知らないですけど。
365ご冗談でしょう?名無しさん:2010/03/27(土) 22:15:33 ID:ImFu89ZA
多様体だとかでぐぐってもさっぱり意味が分からない。

三次元多様体と三次元空間と三次元ユークリッド空間の違いってなんなんですか?
366ご冗談でしょう?名無しさん:2010/03/27(土) 22:27:22 ID:???
>>364
> ええと、ですね。
> もし宇宙の膨張速度が光速を超えるならば、今の地球からの観測可能な範囲外にも星々が広がっていると
> 納得できるんです。
光速より早いです。

> じゃなくて、光速の制限を受けるならば、ビッグバンの衝撃波前線みたいなもんと、地球からの観測可能半径が一致しますよね。
> だったら、それが宇宙の果てと言ってもいいんじゃないですか?
> 今の地球が属しているビッグバンとは別のビッグバン宇宙があるとか言うのなら知らないですけど。
制限を受けません。というか何を言いたいのかわかりません。
367ご冗談でしょう?名無しさん:2010/03/27(土) 22:27:30 ID:???
大分前に回答出てるんだろ。SFはSF板でやれ
漫画とかで宇宙論勉強してきてその次はブルーバックスとか読め
368ご冗談でしょう?名無しさん:2010/03/27(土) 22:35:16 ID:ImFu89ZA
ええっ、制限をうけないのですか?

そういうのってどうやって確認するんですか?
369ご冗談でしょう?名無しさん:2010/03/27(土) 22:50:23 ID:???
宇宙のあちこちにチキュウと呼ばれる天体があり、ニンゲンという知的生命体が存在したとする。
あちこちというからには、天の川銀河系の中心の真反対にもあるだろうし、アンドロメダ銀河の中心付近
にもアンタレスから1000光年離れたところにも、アルデバランから500光年離れた位置にもだ。

彼らは、てんでバラバラに 我々の住むチキュウが宇宙の中心だと考えることは想像に難くないわな。
どのチキュウから観測しても、得られる結果は同じだ。

と考えると、やっぱり宇宙の中心はココでもなければアソコでもないということになる。
370ご冗談でしょう?名無しさん:2010/03/27(土) 22:57:57 ID:ImFu89ZA
そんなクソみたいなレスはしないほうがマシですわ!
なぜなら、アンタレスにはニンゲンなんて住んでいないから。
お前はおれをおちょくっているのか?

例えばさあ、地球は平たいんだと思い込んでいる人間に
地球は丸いんだ、ググレ、啓蒙書読めって言ったって納得しないわな。
納得させるには、宇宙から撮った地球の写真を見せたら一発だ。
おれはそういうのを求めているんだ。見せろ!

だいいち、おれはただ単に啓蒙書読めとだけ答えるやつは
自分なりに答えることのできないアホだと思っている。
本当に頭のいい人物は、おれのような未熟者にもレベルにあった回答をしてくれるはず。
ファインマンさんを見習えよ!
371ご冗談でしょう?名無しさん:2010/03/27(土) 23:05:41 ID:???
自分で調べようともせず他人に教えを乞う人間に能弁垂れられたくないな
372ご冗談でしょう?名無しさん:2010/03/27(土) 23:06:20 ID:???
俺はファインマンの言葉は教える側には意味深いか、同時に罪深い言葉だったとも思っているがな
まさにキミみたいに勉強もせず楽して手取り足取り教えてクレクレ君を助長させてるからね
373ご冗談でしょう?名無しさん:2010/03/27(土) 23:06:46 ID:???
かまってかまって、まで読んだ
374ご冗談でしょう?名無しさん:2010/03/27(土) 23:11:48 ID:ImFu89ZA
なんだこいつらは。
教える教えられる態度について講釈たれはじめた。たまらねえよ・・・・・・
375ご冗談でしょう?名無しさん:2010/03/27(土) 23:13:24 ID:???
>>370
お前は自分がどれだけ支離滅裂な主張をしてるのかわかってるのか?
376ご冗談でしょう?名無しさん:2010/03/27(土) 23:16:43 ID:???
厚顔無恥とはまさにこの事
377ご冗談でしょう?名無しさん:2010/03/27(土) 23:26:15 ID:ImFu89ZA
おれの宇宙観はだ。

水面に、石をぼちゃっと放り込むのよ、ビッグバンだ。
そうしたら波紋が広がっていくでしょう。
地球はその波紋の中心にあって、周囲の波がキラキラ光るのを、見ているわけですよ。
やっぱり中心はシェイクされるのも激しいだろうから、生命が発生したのもそこだけなんだ。
地球人は一人ぼっちだ!!

で、波紋の先端ははどこまで広がっていったんだろうかなって聞いてるんだよ。
誰かさんは、光速超えるよ、第一波の波はもう観測できないよって言うけど
それはどうして分かるのだろうかって思ったんです。
それとも、この主張根本的におかしいですか?
378ご冗談でしょう?名無しさん:2010/03/27(土) 23:28:21 ID:???
>>377
寝言は寝てから言えよ。
379ご冗談でしょう?名無しさん:2010/03/27(土) 23:31:45 ID:???
>>377
哲学板に行った方が早いと思うよ。
ネタとか煽りじゃなくて真面目な話
380ご冗談でしょう?名無しさん:2010/03/27(土) 23:31:45 ID:ImFu89ZA
・・・わかったよ、もう寝る。
あんたはおれのイビキを聞いていりゃいいんだ。
381ご冗談でしょう?名無しさん:2010/03/27(土) 23:45:29 ID:???
スカラーポテンシャルのイメージとしては位置エネルギーで合ってますよね?
じゃあベクトルポテンシャルは何をイメージすればいいんでしょうか。
とりあえず場にはそういう量があってそれがB=rotAの関係を満たしながら振動(?)してるってイメージしてるんですが…
382ご冗談でしょう?名無しさん:2010/03/27(土) 23:54:22 ID:???
>>381
電磁相互作用の入ったハミルトニアンを調べたらいいと思う
383ご冗談でしょう?名無しさん:2010/03/28(日) 00:01:00 ID:???
>>382
場の古典論辺りに載ってますかね。
手元にある電磁気学の本(といっていいのかどうかは微妙だが)はそれだけなんで
384ご冗談でしょう?名無しさん:2010/03/28(日) 00:21:32 ID:???
>>377
ポエムなら



詩・ポエム
http://love6.2ch.net/poem/
385ご冗談でしょう?名無しさん:2010/03/28(日) 12:02:24 ID:???
ハミルトニアンとラグランジアンって、よく似ていますが
どう違うものなのでしょうか?
386ご冗談でしょう?名無しさん:2010/03/28(日) 13:00:54 ID:6AjwyEqB
光源からの光が干渉して打ち消し合わないのはなぜ?
確率的には逆位相の波も同じくらいでてて十分打ち消しあいそうなのに
387ご冗談でしょう?名無しさん:2010/03/28(日) 16:35:17 ID:???
道路に降った雨が流れる時に
波みたいになって流れるのはなんでですか?
388ご冗談でしょう?名無しさん:2010/03/28(日) 20:19:20 ID:???
あ、乱反射するからか
389ご冗談でしょう?名無しさん:2010/03/28(日) 21:04:12 ID:???
夜空を見上げると星が沢山見えますよね。途方も無い距離と、様々な重力動乱を
免れて、肉眼でもひとつひとつの星が分離して見える程の明瞭度で、地表で観測
できている。

電磁波のセパレーションが良好です。普通に考えれば混ざってノイズになって、モヤッ
としか見えないだろうところ、ちゃんと見えます。

光は真空中を進むとき減弱しないんですってね。

一方、電波という電磁波。大気圏外で受信してもザーというノイズだそうですね。
どこからかの意味のある電波が、明瞭度をもって受信できそうなものなのに?

やはり37億光年範囲内には、電波を交信手段とする文明など無いのでしょうか?
390ご冗談でしょう?名無しさん:2010/03/28(日) 22:14:29 ID:RHePVVBV
質問です。
鏡はガラスなどの滑らかな透明板の裏に、銀色の金属を付着させています。
これで光が鏡面反射するから鏡になるのですが、
白い紙など白色物は光を乱反射させるから、鏡にはならないらしいです。

では、ガラスに白ペンキを塗布して滑らかにしても鏡にはならないのでしょうか?
ならないとしたら、白はその色自体が光を乱反射させると言うことでしょうか?

あと、万華鏡を作るとき、ガラスの裏に黒い紙を貼って鏡を作りますが、
黒は光を吸収するのになぜ鏡になるのでしょうか?

あと、万華鏡以外では通常、たとえば窓ガラスの裏に黒い紙を貼っても
鏡のようには映りますが、鏡と違って像が暗いのはなぜでしょうか?

よろしくお願いします。
391ご冗談でしょう?名無しさん:2010/03/28(日) 22:37:21 ID:???
銀が曇ると白くなるよね
392ご冗談でしょう?名無しさん:2010/03/28(日) 22:46:39 ID:???
>>390
ガラスの背面を銀ではなく白にすると、裏面にある白の乱反射で、表のガラス面の反射を邪魔し、鏡のように綺麗には移れない。
一方黒色だと、ガラス面の反射だけとなるので、(反射能は低いが)鏡のように扱える。
鏡は、ガラス背面の、真っ平らになった金属面が主な反射構造だが、黒色ではガラス表面のみの反射となるから。
393ご冗談でしょう?名無しさん:2010/03/28(日) 22:51:22 ID:???
>>390
自分から出た光がそのまま反射すると自分が映ります
ガラスの後ろに白色(乱反射)があると
手前に映っている自分がとても見えにくくなります
394ご冗談でしょう?名無しさん:2010/03/28(日) 22:53:06 ID:luSNC2vE
>>392
かぶったすまん
395ご冗談でしょう?名無しさん:2010/03/28(日) 23:03:44 ID:RHePVVBV
>>392
>>393
ありがとうございます。
なるほど、窓に自分の顔が移るのは、バックの黒のせいというより、ガラス面自体の反射なんですね。
勉強になりました。

あと、つまり、白はどんなに平らに滑らかにしても乱反射してしまうんですか?

それから、万華鏡では外部からの余分な光が少ないから、
黒い紙を貼るだけでも、ガラス面の反射だけで鏡のように綺麗に見える、という理解でいいですか?
396ご冗談でしょう?名無しさん:2010/03/28(日) 23:05:31 ID:luSNC2vE
>>389
今現在観測可能な精度の範囲内では
電波を交信手段とする文明が見つかっていないということで
観測範囲外の精度を必要とする距離
観測されていない過去や未来についてはわかりません
397ご冗談でしょう?名無しさん:2010/03/28(日) 23:17:53 ID:luSNC2vE
>>395
つるつるに磨けるならホワイトボードや
白い食器のようには見えるかもしれません
ただ白く見えている段階では乱反射しているだけです

万華鏡については、反射しているガラスの背面から余計な光がない
(反射したものだけがきれいに見える)
かつ、万華鏡の見たいもの以外に余計なものが映りこむ、反射する余地がない
ならばきれいに見えます
398ご冗談でしょう?名無しさん:2010/03/28(日) 23:26:02 ID:???
>>395
白ってあらゆる光を反射するよね…
399ご冗談でしょう?名無しさん:2010/03/28(日) 23:31:40 ID:RHePVVBV
>>397
ありがとうございます。

実は先日、飲み会で、鏡はなぜ映るの?と聞かれて、万華鏡を作った事を思い出して
ガラスに黒い紙を貼ってあるんだよ、などと言ってしまいましたが、間違っていました。
400ご冗談でしょう?名無しさん:2010/03/28(日) 23:42:25 ID:luSNC2vE
>>399
言いそびれましたが、
万華鏡でガラスに反射した光(見たいもの)がきれいにはっきり見えるのは
ただ反射しているだけでなく全反射しているからです
401ご冗談でしょう?名無しさん:2010/03/29(月) 00:20:04 ID:U/PZayNw
>>400
ちょっと分からなくなってきたんですが、屈折率の大きいほうから小さいほうを見る時に
起こるのが全反射ですよね。たとえば海中から空を浅く斜めに見上げると、
空が一切見えなくて、海底のワカメやサンゴが水面に映るというかんじで。

でも万華鏡の場合は空気中から見るのにどういう理屈で全反射が起こるのですか?
402346:2010/03/29(月) 00:23:58 ID:???
一応ガチ質問なんだ・・・
403ご冗談でしょう?名無しさん:2010/03/29(月) 00:25:22 ID:T/HJo5Kd
ふと思ったことがあるんですが、ドラえもんのお腹についている4次元ポケットの中身は4次元ではないのでは?と疑問に思いました。

0次元→点
1次元→線(点が集まったもの)
2次元→面(線が集まったもの)
3次元→立体(面が集まったもの)
4次元→時間を止めた逆行したり自由自在に操れる世界(
5次元→時間と空間を自在に操れる世界
そう考えると、あのポケットが4次元ならポケットのサイズ以上のものは入らない(空間が広がるわけではないため)けど、5次元なら違う世界の空間も自在に操れるため、ポケットより大きいサイズのものも入れることができます。


404ご冗談でしょう?名無しさん:2010/03/29(月) 00:36:48 ID:???
ランダウ減衰ってどの辺が凄いのか,複素積分が苦手な俺に教えてくれ
そもそも複素積分って物理的な経路を選ばないといかんの?
「場と粒子がエネルギーやり取りして,減衰するのが物理的。
この効果を取り入れる積分路がこれ」 by landau
→実験で確認→物理的直観sugee ってことでいい?
405ご冗談でしょう?名無しさん:2010/03/29(月) 00:40:53 ID:???
>>401
空気 -> (表) ガラス (裏) -> 空気
406ご冗談でしょう?名無しさん:2010/03/29(月) 00:58:40 ID:U/PZayNw
>>405
ありがとうございます。
ガラスの内部で全反射が起きるんですね。納得しました。すっきりしました。
これで飲み会で知ったかぶりできます。
407ご冗談でしょう?名無しさん:2010/03/29(月) 01:01:56 ID:I1o+pJww
コイルに変化しない直流電流を流すと、変化しない磁場が発生しますよね。普通の直流電磁石。

コイルのなかに変化しない磁場(磁石)を置くと、コイルに直流電流が流れないのは何故ですか?

408ご冗談でしょう?名無しさん:2010/03/29(月) 01:11:04 ID:???
>>402
角運動量?
系の空間の回転変換の生成子で、回転対称性がある場合の保存量
一般にはスピンもあるから位置ベクトル×運動量ベクトルと言うのは正しくない
409ご冗談でしょう?名無しさん:2010/03/29(月) 02:42:31 ID:???
>>407
コイルの中に電磁石を置く時、磁場の増加に反する起電力が発生する。
もしコイルを含む回路がありその抵抗が0ならば、起電力を積分した電流が流れる。
しかし回路には普通抵抗が有り、電流は直ぐジュール損失する。
410ご冗談でしょう?名無しさん:2010/03/29(月) 11:47:01 ID:???
スピノルの導入において、ベクトルやテンソルとは違った概念だとと見たのですが、
普通の縦ベクトルではないのでしょうか?何が違うのでしょう?
411ご冗談でしょう?名無しさん:2010/03/29(月) 12:17:16 ID:???
>>403
ドラえもんのポケットは時間は含めないで空間のみで4次元なのだろう。
だから3次元物体は沢山収納出来ます。
とは言っても、ポケットの容量は有限だろうけど
412ご冗談でしょう?名無しさん:2010/03/29(月) 12:33:33 ID:???
上の方にあった質量に関する質問に似ているのですが
先日実験で電子秤を用いて質量を計測し、ある式中のmに代入したことを覚えています
電子秤で測っているものは、重量ではなく質量なのですか?
原理がよくわかりません
413ご冗談でしょう?名無しさん:2010/03/29(月) 12:47:35 ID:???
簡単な原理はググレば出てくる。
原理からして、計っているのは重量。
したがって重力加速度が異なる場所へ移動したら補正が必要。
414ご冗談でしょう?名無しさん:2010/03/29(月) 14:11:10 ID:???
地球にいるかぎりは重量を質量として扱っても良いということですか?
415ご冗談でしょう?名無しさん:2010/03/29(月) 14:28:32 ID:???
>>414
地球上でも場所によって重力加速度は異なる
それを気にしないくらいの測定なら、重量=質量でよい(数値が同じという意味で)
気にするくらいの測定ならだめ

あと、なにを測ってるのかだけど
しかるべく補正かけて表示されるなら、質量を測っているといってよいのではなかろうか
416ご冗談でしょう?名無しさん:2010/03/29(月) 14:40:20 ID:???
測定しているのは「重量」という「力の大きさ」だ。
「質量」の大きさではなく「質量×重力加速度」の大きさを測っている。
それがわかっているなら、あとはどうとでも言葉遊びすればいい。
417ご冗談でしょう?名無しさん:2010/03/29(月) 14:47:01 ID:???
ぶっちゃけ質量と重量って次元が違うわけだけど
質量=重量と扱っていいってなんだよ
それはないわ
418ご冗談でしょう?名無しさん:2010/03/29(月) 15:01:02 ID:DLpoz05u
高校時代文系だった友人に相対論おしえてと言われて困っています。
行列もベクトルも一次変換も知りません。
どうしたらいいでしょうか?
419ご冗談でしょう?名無しさん:2010/03/29(月) 15:08:07 ID:???
ちょうど4月から開講するではないか
420ご冗談でしょう?名無しさん:2010/03/29(月) 15:42:44 ID:???
>>417
>質量=重量と扱っていいってなんだよ
一方をkg単位、他方をkgw単位であらわせば数値が(ほぼ)一致するという程度の意味だな。
それが何か
421ご冗談でしょう?名無しさん:2010/03/29(月) 17:47:53 ID:ooI07ubM
貴方は「いじめられる側にもいじめの原因がある」と思ってませんか
或いは「死刑には抑止力がある」と思ってませんか
或いは「掲示板で不愉快になったらその原因は発言者にある」と思ってませんか

これらの考えの根っこは、実は同じです(※善悪の話ではありません)

これまで当たり前だと思って、深く考えなかったこと・・・
周りに言われるままに、何の疑問も抱かなかったこと・・・
それらが本当に正しいのか、ちょっと立ち止まって考えてみませんか


いじめに関するよくある勘違い
http://mamono.2ch.net/test/read.cgi/youth/1268212962/

死刑制度に関するよくある勘違い
http://society6.2ch.net/test/read.cgi/court/1268212298/

外界は内界を映し出す鏡だって言ってたようちの嫁も
http://academy6.2ch.net/test/read.cgi/philo/1264845494/
422ご冗談でしょう?名無しさん:2010/03/29(月) 19:11:17 ID:???
院試の際の訪問で他大行った時
そこの教授は量子情報専門にしてるんだけど電磁場テンソルの存在を知らなかったんだわ

量子論系の教授って相対論に疎いモンなの?
自分Fランだからよくわからん
423ご冗談でしょう?名無しさん:2010/03/29(月) 20:00:43 ID:???
>>410
座標変換に対する変換性が違う
424ご冗談でしょう?名無しさん:2010/03/29(月) 20:25:20 ID:???
>>422
量子論系というと色々あると思うが、物性あたりは相対論的効果とは
無縁の分野も多いから、相対論に疎い人は居る
また大学に残ってる教官って専門バカな人も少なくないから、
研究分野から外れる事では、こんなことも知らないの?って人は割と居るよ
まぁでも、そんな学部で普通に学ぶ事で全然知らないのはちょっと恥ずかしいとは思うけどね
425ご冗談でしょう?名無しさん:2010/03/29(月) 20:36:27 ID:???
>>424
やっぱりそういうモンなんですね。
ありがとうございます、礼を言うほどでもない気がするが
426ご冗談でしょう?名無しさん:2010/03/29(月) 21:32:38 ID:???
ブラケット表記法の
<1|0>とかの1や0は何を意味しているんでしょうか。
c数でベクトルじゃないみたいだし…それともベクトルなんでしょうか
427ご冗談でしょう?名無しさん:2010/03/29(月) 21:55:47 ID:???
定義してるだろ。どうせ整数で状態をナンバリングしてるだけだろ。
428ご冗談でしょう?名無しさん:2010/03/29(月) 22:33:58 ID:???
>>427
ああなんだ。
c数だと思って真剣に悩んでた自分が馬鹿馬鹿しいわ
429ご冗談でしょう?名無しさん:2010/03/30(火) 00:14:12 ID:???
レーザが出るまで,マイクロ波の技術の方が進んでいたとかいうけど,
結局なんで,可視光って制御しにくかったの?単純にマイクロ波の方が,
大きめの扱いやすい素子で偏光とか位相とか弄れたりしただけ?
430ご冗談でしょう?名無しさん:2010/03/30(火) 00:30:13 ID:???
メーザーって反転分布無しで動くんじゃなかったっけ?
記憶違いじゃなければ
431ご冗談でしょう?名無しさん:2010/03/30(火) 00:31:49 ID:???
単純にって言うけど
数100THzのオシレータってそう簡単には作れないでしょ
432ご冗談でしょう?名無しさん:2010/03/30(火) 01:47:06 ID:???
返答ありがとうございます。黒体輻射や放電蛍光等で,
以前から高周波の可視光が例外的に使われていたせいでよく引っかかる・・
オシレータ→ダイポール等で比較的精度のいいマイクロ波が使われると一時的に遅れをとったけど,
レーザが出ると高周波の可視光の方がやはり発展するといった感じか・・
433ご冗談でしょう?名無しさん:2010/03/30(火) 04:30:59 ID:???
20 :132人目の素数さん:2010/03/28(日) 00:45:57
物理学者はちょっと程度が低いのばかりだから連中のいうことは基本的に気にしなくて良い
最近数ヶ月物理の勉強し始めたんだけど、革命的な理論がすでに5個くらい思いついたよマジで
http://science6.2ch.net/test/read.cgi/math/1269691530/

物理学者って、数学者に比べればかなり頭が悪いの?
434ご冗談でしょう?名無しさん:2010/03/30(火) 11:24:19 ID:???
マジレスすると頭の良さのベクトルが全然違う
物理屋は理論屋も実験屋もいるし一概に比較するのは馬鹿げてる
物理を理解するのに数学が、数学を理解するのに物理が必要なこともある
435ご冗談でしょう?名無しさん:2010/03/30(火) 17:14:45 ID:???
純粋数学と理論物理で比較しても全然違うからなあ。
どっちが上かって言うのはナンセンスもいいとこ

ただ崩れの数で言えば物理学の方が圧倒的だろうな、数学は途中で才能無いと気づいて降りちゃうから
436ご冗談でしょう?名無しさん:2010/03/30(火) 17:25:16 ID:uNYHgLEy
http://beebee2see.appspot.com/i/agpiZWViZWUyc2VlchQLEgxJbWFnZUFuZFRleHQY1LVmDA.jpg

これの8番の問題がわかりません。
接線方向と法線方向のそれぞれに
運動方程式を立てようと思ったとですが、
法線方向の式がわからないです。

437ご冗談でしょう?名無しさん:2010/03/30(火) 17:29:01 ID:???
>>436
写真wwwwwwwwwwwwwwwwwwwwwwwwwwwwwwwwwwwwwwwwwwwwwwwwwwww
438ご冗談でしょう?名無しさん:2010/03/30(火) 17:32:01 ID:???
>>437
あれ?マズかった…?
439ご冗談でしょう?名無しさん:2010/03/30(火) 17:38:46 ID:???
>>436
水平方向と鉛直方向の運動方程式を立てる方が楽そう
440ご冗談でしょう?名無しさん:2010/03/30(火) 18:06:35 ID:???
>>439
運動方程式は立てられましたけど、
うまく合わない…
441ご冗談でしょう?名無しさん:2010/03/30(火) 20:22:20 ID:???
SF等で、例えば1光年離れた天体から光を受け取ったとき
「この光は1年前に放たれたもの」という表現がされますが
これって絶対時間に基づいた考えなんでしょうか?
それとも相対論的に見ても問題のない考えでしょうか?
442ご冗談でしょう?名無しさん:2010/03/30(火) 20:37:48 ID:???
>>441
光が一年で飛ぶ距離が1光年だぞ?
そこに絶対時間も相対論も入り込む余地なんて無いと思うんだが

君は一体何を疑問に思っているんだね
443ご冗談でしょう?名無しさん:2010/03/30(火) 21:16:45 ID:YdhQOG1W
>>440 うまく、合わない、って何が合わなかったの?
444ご冗談でしょう?名無しさん:2010/03/30(火) 22:33:13 ID:LPdfYUDe
>>441
そのSFが何年後の未来を描いてるのかは知らんが
1光年程度の距離じゃたいして変わらんと
何万光年と離れてるのに共同距離や光行距離の概念がないSFなら
作者のレベルがその程度なんだと思ってればいいよ
所詮はフィクション
445ご冗談でしょう?名無しさん:2010/03/31(水) 10:28:09 ID:???
宇宙の全てのエネルギーは、最終的に全て熱になりますか?
446ご冗談でしょう?名無しさん:2010/03/31(水) 11:20:45 ID:YRd5HsBZ
>>439
水平方向と運動方程式
447ご冗談でしょう?名無しさん:2010/03/31(水) 11:22:39 ID:YRd5HsBZ
中級だが、粒子運動とか考えろ。後は、調べていけばわかるよ。
448ご冗談でしょう?名無しさん:2010/03/31(水) 14:56:42 ID:dT/HnwPZ
>>436 landauの力学の問いに接線および法線方向の運動方程式が導出してあったような。
449ご冗談でしょう?名無しさん:2010/03/31(水) 15:46:31 ID:???
>>444
100万光年でも、100万年経っても1パーミルも増えないから気にする必要ナッシング
450ご冗談でしょう?名無しさん:2010/03/31(水) 16:50:54 ID:NSQPNp/n
>>448 そんなに性急に答えを書いていけない、と思う。ちなみに
法線方向は「運動方程式」と呼ぶのには抵抗あるなあ。
451ご冗談でしょう?名無しさん:2010/03/31(水) 17:00:55 ID:???
「○○方向の運動方程式」より「運動方程式の○○方向成分」と言うべき?
452ご冗談でしょう?名無しさん:2010/03/31(水) 17:01:02 ID:dT/HnwPZ
>>450 ごめんなさい。
453ご冗談でしょう?名無しさん:2010/03/31(水) 18:14:18 ID:???
>>440
運動方程式そのまんまだとわからなくて、
Vの微分方程式とφの微分方程式に分けないとだめなんじゃね

とりあえず君のたてた式を書こうぜ
454ご冗談でしょう?名無しさん:2010/04/01(木) 00:38:23 ID:D7KwN9cj
久しぶりに来たら懐かしい話題で・・遠心力mvωと重力法成分の釣り合いね
>>448
ごめんランダウのどこ

質問だが,光子系の自由度を2Vd^3k/(2π)^3と数えて,
自由度にkT/2割り当てると,なんでレーリージーンズの1/2になるんだっけ
455ご冗談でしょう?名無しさん:2010/04/01(木) 00:45:59 ID:???
レーリージーンズの1/2 ってなに?
456ご冗談でしょう?名無しさん:2010/04/01(木) 13:50:23 ID:???
無は無ですよね。
では、真空は「有」ですか?

あと、「有」=有限 「無」=無限 ですよね?
457ご冗談でしょう?名無しさん:2010/04/01(木) 15:24:04 ID:???
>>456
質問の意味がわからん。
458ご冗談でしょう?名無しさん:2010/04/01(木) 16:28:12 ID:???
>>457
多くのもの(こと)は、最終的には消えて無くなりますよね?
そして、大きさや形がありますよね?

しかし、「無」はそのようなことがないじゃないですか。
つまり、「無」は無限だと思うわけです。
それに対して、最終的には消えて無くなったり、形や大きさがある「有」は有限だと思うわけです。

あと、真空はもちろん「有」ですよね?
459ご冗談でしょう?名無しさん:2010/04/01(木) 16:52:06 ID:???
>>458
素粒子に大きさがあるって誰が言った?
460ご冗談でしょう?名無しさん:2010/04/01(木) 17:04:10 ID:???
>>459
ていうか、「有」である時点で、大きさや形は間違いなく存在しますよね?
461ご冗談でしょう?名無しさん:2010/04/01(木) 17:06:18 ID:13p05Ztm
素人質問で申しわけないのですが、

空気冷凍サイクルを利用した冷却で、手動で実用に耐えうる方法はありますか?
もしくは手動のポンプを利用などの人力で、5℃程度温度を下げる方法がありましたら是非ご教授願います。
462ご冗談でしょう?名無しさん:2010/04/01(木) 17:12:42 ID:???
>>460
いいや、そんなもん要らんよ。
463ご冗談でしょう?名無しさん:2010/04/01(木) 17:16:35 ID:???
>>462
でも、「有」である時点で、最終的には消えてなくなることは間違いないですよね?

つまり、有=有限 無=無限

ということになるのでしょうか?
464ご冗談でしょう?名無しさん:2010/04/01(木) 17:22:24 ID:???
>>463
言葉を玩んでいるようだ。
哲学板のほうが適切なんじゃないの。
465ご冗談でしょう?名無しさん:2010/04/01(木) 21:20:07 ID:???
量子光学に手を入れる前に場の理論を学ぶべきでしょうか。
手始めに高橋の場の理論のための解析力学でもやろうかと考えてるんですが

場の理論無しで量子光学に直行できるならラウドン(スペル忘れた)辺りをやろうかと考えています
466ご冗談でしょう?名無しさん:2010/04/01(木) 21:51:37 ID:???
>>465
量子光学は大抵が空間シングルモードの話をしてるので
空間シングルモードに限れば場の理論はあんま関係ないかなあ

ただ一般論としては量子場を学ぶための場の解析力学の内容は頭にあるといいかも
あと電磁気学再入門・QEDへの準備も簡潔によくまとまってる
ラウドンはもちろんいい本
467ご冗談でしょう?名無しさん:2010/04/01(木) 22:15:32 ID:???
>>466
ありがとうございます。
>あと電磁気学再入門・QEDへの準備も簡潔によくまとまってる
考慮に入れておきますね
468ご冗談でしょう?名無しさん:2010/04/01(木) 22:26:37 ID:???
素粒子の大きさを教えてください。
体積は?面積は?長さは?質量は?
469ご冗談でしょう?名無しさん:2010/04/01(木) 22:29:40 ID:???
>>461
用途は?
原理検証?それともなにかを冷やしたいの?
470ご冗談でしょう?名無しさん:2010/04/02(金) 14:29:27 ID:???
>>461
用意するもの

廃品業者から冷蔵庫
スーパーからウナギの蒲焼10本

まず、冷蔵庫を解体して冷凍機を取り出しモーターを取り外す
冷凍機のプーリーに取っ手を付けて人力でひたすら回す、其の内精魂尽きるからスーパーで
買ったウナギの蒲焼を・・・喰っては回し、回しては喰う・・・
471ご冗談でしょう?名無しさん:2010/04/02(金) 16:55:32 ID:EZEkehib
>>451
そうですね。
472ご冗談でしょう?名無しさん:2010/04/03(土) 01:05:43 ID:???
早坂秀雄氏の本(トンデモ?)を読んで思ったんだけど
相対論のトーション=0ってのは実は崩れてたりしてないの?
物理は素人なんで疑った事が無かったんだけど


473ご冗談でしょう?名無しさん:2010/04/03(土) 09:49:04 ID:???
>>472
どんな量でも完全にゼロだとは実証できない。
474ご冗談でしょう?名無しさん:2010/04/04(日) 20:26:18 ID:???
一般向けの本なんかを見てると
フェルミオンのスピンは1/2
ボソンのスピンは1
のように表記されていますが
これは基底状態ってことですか?
整数とか半奇数とかで分けられてないので気になったのですが
475ご冗談でしょう?名無しさん:2010/04/05(月) 02:30:57 ID:???
1粒子状態のスピン
476ご冗談でしょう?名無しさん:2010/04/05(月) 10:32:08 ID:???
基底状態だろうが励起状態だろうが
スピンが整数ならボソン、半奇数ならフェルミオン
477ご冗談でしょう?名無しさん:2010/04/05(月) 11:48:41 ID:???
>>475
残念だなあ。
君は何か勘違いしている様だけど、聞いて揚げ足取りをしたいんじゃなくて、
単純に君の意見が面白そうだから聞いてみたかっただけなんだよ。
もっとまともなレスを期待してたんだけど。ごめんね。
ちなみに君は揚げ足を取られる事を恐れているようだけど、それは違うと思う。
揚げ足を取られるのは、君に隙があるから。
隙だらけの人間ってのはそうやって余計な所でつまらない恥をかく事になるもんだよ。
他人につっこまれる事は、自身の知識をより客観的で多角的なものとする為に
有益な指針となるものだと思うな。
478ご冗談でしょう?名無しさん:2010/04/05(月) 14:06:51 ID:???
とーとつに何の演説おっぱじめたの?>>477
479ご冗談でしょう?名無しさん:2010/04/05(月) 18:44:23 ID:???
電子と陽電子は右巻き、左巻きが違うのでしょうか?
480ご冗談でしょう?名無しさん:2010/04/05(月) 22:00:03 ID:???
>>479
どっちも両方あるが?
481ご冗談でしょう?名無しさん:2010/04/06(火) 10:35:47 ID:???
対生成時の対称性から片方が右ならもう片方は左な気がします
この考えっておかしいのですかね
482ご冗談でしょう?名無しさん:2010/04/06(火) 11:14:16 ID:???
>>481
いろんな意味で間違ってる。

仮に一方が右で他方が左だとしても、電子が右で陽電子が左の場合もあれば
その逆の場合もある。

さらに、重心系で見た場合、初期状態の全角運動量が0の場合、
電子と陽電子はむしろ同じ巻き方でないといけない。
運動量、スピンとも電子と陽電子で逆向きになるから、
運動量の向きに対するスピンの向きは電子と陽電子で同じになる。
右巻き左巻きの定義わかってる?
初期状態の角運動量が0でない場合、単純に右左は決まらない。
どっちにしても対生成時の対称性から一方が右なら他方は左という
結論は導かれない。

そもそも、電子や陽電子のように質量のある粒子の場合、
ある系で左(右)巻きだとしても、それを追い越しながら
眺める系では右(左)巻きになり、そもそも右巻きだ
左巻きだという絶対的な区別はない
483ご冗談でしょう?名無しさん:2010/04/06(火) 11:17:11 ID:IueUgnWU
>>473
−、虚数を使って、実証か、推測できない?
484ご冗談でしょう?名無しさん:2010/04/06(火) 11:19:10 ID:IueUgnWU
過去スレに、酵母を使った不老不死の定期的に食べる食物が書いてあった。
485ご冗談でしょう?名無しさん:2010/04/06(火) 12:02:43 ID:???
エネルギーは相対的なのか絶対的なのかよく分かりません
相対的だとは思うのですが
「絶対零度」がなんか引っかかるんです
486ご冗談でしょう?名無しさん:2010/04/06(火) 13:10:06 ID:IueUgnWU
487ご冗談でしょう?名無しさん:2010/04/06(火) 13:14:14 ID:???
>>485
どこを基準にとっても下限はある
488ご冗談でしょう?名無しさん:2010/04/06(火) 13:16:58 ID:IueUgnWU
489ご冗談でしょう?名無しさん:2010/04/06(火) 15:41:59 ID:???
これって本当ですか?

過去を撮れるカメラの開発を考えるスレ
http://science6.2ch.net/test/read.cgi/future/1264322541/101

101 名前:オーバーテクナナシー[sage] 投稿日:2010/04/01(木) 00:05:41 ID:3vH1dp9x
出来た。遂に過去の光景を画像にする実験に成功した。
論より証拠として画像をうpする。
これは1888年の横浜港の光景である。

 http://mcnc.hp.infoseek.co.jp/cgi-bin/img-box/img20100331235430.gif

(略)
490ご冗談でしょう?名無しさん:2010/04/06(火) 21:34:07 ID:5y71wqNq
初コメです、
凍った物質で割れないで跳ね返るなんてことあるのですか?

凍った死体(ペキンダックみたいな?)が跳ねるのは常識・・・と言ってたのですが、
どういう仕組みなのでしょうか??

もしカテ違いならごめんなさい。
よろしくお願いします。
491ご冗談でしょう?名無しさん:2010/04/06(火) 22:17:42 ID:5y71wqNq
反発係数と物質の中身のこと考えたらわかりました

失礼しました、
本当にご迷惑おかけしました。
492ご冗談でしょう?名無しさん:2010/04/06(火) 22:59:24 ID:???
>>490,491
何を質問して何がわかったのか、さっぱり・・・
493ご冗談でしょう?名無しさん:2010/04/06(火) 23:05:58 ID:???
下記の文章はどの程度本当ですか?

光は電磁波であると同時に光子として粒子との性質も持つ。その粒子性を利用する。
強度の大きい光束内では光子の密度が高く光子相互間に働く力が無視できなくなる。
その状態(光子気相化超臨界密度状態)では光子の集合はあたかも気体のように振舞う(気相光子空間)。気相光子空間では光子密度の粗密が縦波として伝播し、その粗密波の速度は光子そのものの移動速度を超えることができる。
すなわち、気相光子空間における光子粗密波を利用することにより光速を超えて情報を伝達することすなわち情報を過去方向に伝達することが可能である。
ただし因果律を破ることはできないため、未来の情報を現在の時点で入手することはできない。
しかしながら、現在の時点から見て過去の情報をより過去の時点に送ることは因果律を破らないため可能であるので、過去の光景の画像情報をさらに過去の印画紙に焼き付けることができる。
494ご冗談でしょう?名無しさん:2010/04/06(火) 23:11:21 ID:???
最初の1行はまあ、本当
光子-光子相互作用が生じるような光の強度は半端じゃないので2行目はほとんど嘘
3行目以降はただのデタラメ
495ご冗談でしょう?名無しさん:2010/04/06(火) 23:12:56 ID:???
大体間違えてる。
496ご冗談でしょう?名無しさん:2010/04/06(火) 23:14:03 ID:HE9D1Hj9
回転の自由度が3ってどのようなものなのでしょうか?
θとφで自由度は2だと思っていました。
もうひとつはなんなのでしょうか?
497ご冗談でしょう?名無しさん:2010/04/06(火) 23:45:28 ID:???
砲塔みたいなものを考えてみれ

最初大砲は水平で、砲塔は真東に向いてるとする

 ・そのまま大砲を真上に向けた場合と、
 ・砲塔を一旦90度左回転して北に向けてから大砲を真上に向けた場合

この二つの場合が同じになると思うなら自由度は2 (θ=砲塔の向きとφ=大砲の仰角) でいい
498ご冗談でしょう?名無しさん:2010/04/07(水) 00:11:32 ID:???
>>497
なかなか教育的でおもしろいな
単純に
回転軸を指定するのに2個
回転の量を指定するのに1個
で計3個ってのもありかな
499ご冗談でしょう?名無しさん:2010/04/07(水) 00:16:05 ID:???
>>494
> 最初の1行はまあ、本当
> 光子-光子相互作用が生じるような光の強度は半端じゃないので2行目はほとんど嘘
> 3行目以降はただのデタラメ

もし仮に光子-光子の相互作用が生じるような強度であれば、光子の粗密波の伝播速度は光速を超えるというのは本当でしょうか。
500ご冗談でしょう?名無しさん:2010/04/07(水) 00:25:58 ID:???

だから3行目以降はデタラメ
501ご冗談でしょう?名無しさん:2010/04/07(水) 01:50:11 ID:???
自分は数学を、キーポイントシリーズでイメージつかんで、
その後は問題演習。って感じで勉強しています

大学院を卒業できる程度の数学は身に着けたいのですが
理論的に厳密に数学を勉強するっていう姿勢も必要のなるのでしょうか。
502ご冗談でしょう?名無しさん:2010/04/07(水) 01:58:11 ID:???
理論系なら必要な部分もあるんじゃない?
先輩や先生に聞いてみるのが良いかと
ガチガチにやる必要はなくてもあやふやじゃ困る事ってのはあると思うから
503ご冗談でしょう?名無しさん:2010/04/07(水) 02:01:18 ID:???
>>497
なるほど、ありがとう!
504ご冗談でしょう?名無しさん:2010/04/07(水) 04:38:56 ID:???
>>499
少なくとも既存の物理学ではその相互作用とやらを説明出来ん
505ご冗談でしょう?名無しさん:2010/04/07(水) 06:07:56 ID:???
「無」は無限永遠を超越するのでしょうか?

多くのもの(こと)は、いつかは必ず消えて無くなる。
形や大きさもある。「有」である。

しかし、「無」はそのようなことはない。
無限や永遠は所詮、「有」の領域の概念でしかない。

やはり、「無」は無限永遠を超越するのでしょうか?
506ご冗談でしょう?名無しさん:2010/04/07(水) 07:45:24 ID:???
>>496
> 回転の自由度が3ってどのようなものなのでしょうか?

なんで皆まともに答えてやんないんだ。
マジレスすると、砲塔の例で言うと砲塔が回転して方向を変えるのがひとつめ。
砲塔が仰角を上げげ下げするのがふたつめ。(砲塔ごと動くとしたほうが正確)
砲塔の設置の土台工事が手抜き工事で地盤沈下で傾いて砲塔の左右が傾くのがみっつめ。
507ご冗談でしょう?名無しさん:2010/04/07(水) 07:47:32 ID:???
>>496
車でいうと、
・左右に曲がる
・坂道を登る/降りる
・左右に傾く
508ご冗談でしょう?名無しさん:2010/04/07(水) 08:21:38 ID:???
>>504
光子光子散乱の理論計算はある
509ご冗談でしょう?名無しさん:2010/04/07(水) 09:10:56 ID:???
誰か>>505の質問に答えてください。お願いします。
510ご冗談でしょう?名無しさん:2010/04/07(水) 09:14:57 ID:???
511ご冗談でしょう?名無しさん:2010/04/07(水) 10:12:26 ID:BuD3WVv1
圧縮燃料が必要だな。
512ご冗談でしょう?名無しさん:2010/04/07(水) 10:14:13 ID:BuD3WVv1
光子間の波は、光だから、波状によっては、速さが違う 早いのもあると計算しる。
513ご冗談でしょう?名無しさん:2010/04/07(水) 10:34:13 ID:BuD3WVv1
太陽帆
太陽帆とは? 太陽帆の構想図太陽帆(たいようほ)はソーラー帆、ソーラーセイルとも呼ばれ、薄膜鏡を巨大な帆として、
太陽などの恒星から発せられる光やイオンなどを反射することで宇宙船の推力に変える器具のこと。現在
これが、宇宙船には必要。途中で燃料切れる。
514ご冗談でしょう?名無しさん:2010/04/07(水) 11:36:38 ID:???
>>512
光は波だからうねうねと蛇行しながら進む、とか思ってないだろうな
515ご冗談でしょう?名無しさん:2010/04/07(水) 11:56:03 ID:???
>>493は粗密波って書いてる
516ご冗談でしょう?名無しさん:2010/04/07(水) 12:01:12 ID:???
>>515
3行目以降はデタラメだと何度(ry
517ご冗談でしょう?名無しさん:2010/04/07(水) 12:05:16 ID:???
電磁波の振動数って何ですか?
何かが本当に振動している回数ですか?
それとも数式的にプランク定数の係数にしているだけ?
518ご冗談でしょう?名無しさん:2010/04/07(水) 12:34:53 ID:BuD3WVv1
水素の圧縮燃料って、学校とかにある水素の粉なんだけど、それを固形にして圧縮すると、水素の圧縮燃料だ。
519ご冗談でしょう?名無しさん:2010/04/07(水) 12:37:07 ID:BuD3WVv1
>>517
どっちも、電磁波で調べてみるか、プランク係数の文で調べてみるか、要素でググってみればわかる。
520ご冗談でしょう?名無しさん:2010/04/07(水) 12:37:48 ID:???
どっちも。
521ご冗談でしょう?名無しさん:2010/04/07(水) 13:00:13 ID:???
>>520
サンクス
522ご冗談でしょう?名無しさん:2010/04/07(水) 13:13:23 ID:???
>>518

「固形の水素の粉」ってなんだ。どこの学校にあるんだ。木星の中心か?
523ご冗談でしょう?名無しさん:2010/04/07(水) 14:07:57 ID:???
金属水素なんて夢のある話すなぁ〜
524ご冗談でしょう?名無しさん:2010/04/07(水) 15:10:02 ID:???
金属水素の超伝導とかな
525ご冗談でしょう?名無しさん:2010/04/07(水) 20:01:57 ID:RNW9SL/I
放射能計とか、重力計で一日が2日の時間が経っていることがわかる。
526ご冗談でしょう?名無しさん:2010/04/07(水) 20:02:57 ID:RNW9SL/I
宇宙船のタイムスケジュールが狂うと大変だよ。
527ご冗談でしょう?名無しさん:2010/04/07(水) 23:10:03 ID:???
>>482
>運動量、スピンとも電子と陽電子で逆向きになるから、

運動量が逆向きになるのは分かるのですが、スピンが逆とは?
電子と陽電子のスピンって同じですよね
528ご冗談でしょう?名無しさん:2010/04/08(木) 00:41:07 ID:???
ヘリシティって言いたかったのか?
スピンはおかしい気がする
529ご冗談でしょう?名無しさん:2010/04/08(木) 03:04:30 ID:???
数学者ですが何か質問はありますか?
530ご冗談でしょう?名無しさん:2010/04/08(木) 03:04:51 ID:???
ないです
531ご冗談でしょう?名無しさん:2010/04/08(木) 04:12:29 ID:???
関連する話題なのですが、ニュートリノには質量があるはずなのに
ニュートリノが左巻きで反ニュートリノが右巻きだと何故言えるのでしょうか
これって理論的な話ではなく観測結果ですよね?
532ご冗談でしょう?名無しさん:2010/04/08(木) 05:00:10 ID:???
>>531
うぜえ死ね。
533ご冗談でしょう?名無しさん:2010/04/08(木) 10:39:06 ID:XUrfSilu
電子レンジで温めたお湯は早く冷める気がするがどうでしょうか?
電波で水分子の特定の振動を励起しているのでヤカンで対流させながら
沸かしたお湯と同じ温度でもエネルギーが違うのではないですか?
534ご冗談でしょう?名無しさん:2010/04/08(木) 10:44:01 ID:???
>>532
なんでそんな反応?
535ご冗談でしょう?名無しさん:2010/04/08(木) 10:46:28 ID:???
>>533
定義上不可能だろ
536ご冗談でしょう?名無しさん:2010/04/08(木) 11:34:52 ID:???
>>531
ヘリシティとカイラリティの区別はついてる?
537ご冗談でしょう?名無しさん:2010/04/08(木) 11:56:17 ID:???
>>536
はっきりとはついてないです
ただ、ここでいう左右はヘリシティのことだと思っています
538ご冗談でしょう?名無しさん:2010/04/08(木) 13:39:11 ID:WYYWEN7p
光子を追い越す観測系も存在しない
波はなんなんでしょう?
539ご冗談でしょう?名無しさん:2010/04/08(木) 14:52:26 ID:srS/TqkG
ある本を読んでいたらハイゼンベルグの運動方程式と
演算子A(t)を振動数νでフーリエ展開したもので
hν/2π=E-E'
がみちびかれていました。

しかしエネルギーと振動数の式は
hν=E-E'
ですよね?
なぜ2πずれてしまったのでしょうか?
540ご冗談でしょう?名無しさん:2010/04/08(木) 14:55:37 ID:???
プランク定数
ディラック定数
541ご冗談でしょう?名無しさん:2010/04/08(木) 14:57:05 ID:???
>>539
フーリエ展開の式はどうなっている?
542ご冗談でしょう?名無しさん:2010/04/08(木) 15:01:13 ID:???
>>531,537
> ニュートリノが左巻きで反ニュートリノが右巻き
これは弱い相互作用でのカイラリティのこと。
543ご冗談でしょう?名無しさん:2010/04/08(木) 16:45:14 ID:srS/TqkG
>>541
1/2π∫dνexp(-iνt)A(ν)
です

これからHの固有値の|E>を使うとhν/2π=E-E' がでてきます
544ご冗談でしょう?名無しさん:2010/04/08(木) 16:54:57 ID:???
>>543
なら、このνは振動数ではなく角振動数。
545ご冗談でしょう?名無しさん:2010/04/08(木) 16:55:31 ID:srS/TqkG
>>543
あ!!
そうですね・・・ありがとうございました。
546ご冗談でしょう?名無しさん:2010/04/08(木) 18:49:16 ID:xYfiiYJC
剛体の机にXm真上にある質量Mkgの剛体Aを自由落下して
ぶつかったときのAの受ける衝撃力は何Gになるんですか?
547ご冗談でしょう?名無しさん:2010/04/08(木) 18:50:11 ID:???
>>546

 ∞
548ご冗談でしょう?名無しさん:2010/04/08(木) 19:34:05 ID:xYfiiYJC
>>547
ご回答ありがとうございます

衝突前の速度v1、衝突前の時間t1
衝突後の速度v2、衝突後の時間t2
ここでv2 = -v1
として
加速度a=(v2-v1)/(t2-t1)
で、有限数/0 だから∞ですか?
549ご冗談でしょう?名無しさん:2010/04/09(金) 00:06:56 ID:gLFkGhSS
場の量子論について質問です。

量子化された場の運動量の演算子

P=(h(バー)/2i)∫dx{φ^†∇φ-∇φ^†φ}
として
bose粒子の交換関係を使って
[φ、P]=(h(バー)/i)∇φ
を示したいのですが途中で詰まります。

(h(バー)/2i)∫dx{φ'φ^†∇φ-φ'∇φ^†φ-φ^†∇φφ'-∇φ^†φφ'}
の∇が真中にある場合はどのような展開ができるのでしょうか?
φ=φ(x)、φ'=φ(x')としています。
550ご冗談でしょう?名無しさん:2010/04/09(金) 00:07:34 ID:???
海の波を周波数解析すると、どんな感じになってるのでしょうか?
海岸と海底から十分遠い場合です。
周波数分布は正規分布状で、位相はそれぞれの周波数で
ランダムな方向に進んでいる感じ?
551ご冗談でしょう?名無しさん:2010/04/09(金) 00:10:55 ID:???
>bose粒子の交換関係を使って

どっかで使ったの?
552ご冗談でしょう?名無しさん:2010/04/09(金) 06:13:16 ID:???
今年、東大理科V類を合格した者です。
将来は、物理学者か数学者として食っていきたいと思っているのですが、
どっちになった方が良いのでしょうか?
研究機関などはどっちの方が充実しているのでしょうか?
また、収入や世間一般のイメージや地位などはどちらの方が良いのでしょうか?
そして、一般的に、どちらの方が知能が高いのでしょうか?

物理板の皆さんの詳しい回答を待ってます。
553ご冗談でしょう?名無しさん:2010/04/09(金) 07:41:02 ID:???
春だなあ
554ご冗談でしょう?名無しさん:2010/04/09(金) 09:01:00 ID:???
>>549
フーリエ変換して運動量空間で計算したら?
555ご冗談でしょう?名無しさん:2010/04/09(金) 09:45:48 ID:???
入学祝いにパソコン買ってもらって2chデビューしたんだろうなあ。
でも何か微笑ましいというかちょっと嬉しいのは
日本で一番偏差値の高い大学に入る奴は物理学者か数学者になると思ってるところ。
本当にそうだったら日本の将来は明るいんだけどなあ。
556ご冗談でしょう?名無しさん:2010/04/09(金) 10:09:45 ID:???
>>551
最後の式
(h(バー)/2i)∫dx{φ'φ^†∇φ-φ'∇φ^†φ-φ^†∇φφ'-∇φ^†φφ'}
から交換関係を使うのだと思います。
557ご冗談でしょう?名無しさん:2010/04/09(金) 10:21:23 ID:l1UUQEL+
>>549
いいね。
558ご冗談でしょう?名無しさん:2010/04/09(金) 10:29:56 ID:l1UUQEL+
計算しおわったら、方程式で出るか?教授レベルだけど。
559ご冗談でしょう?名無しさん:2010/04/09(金) 10:31:26 ID:l1UUQEL+
>>554
いいね。
560ご冗談でしょう?名無しさん:2010/04/09(金) 10:38:28 ID:???
>>527
直後にも書いてあるけど、スピンの向きの話をしている。
言葉足らずですまん。スピンの向きが逆ってこと
561ご冗談でしょう?名無しさん:2010/04/09(金) 10:42:29 ID:???
>>539 >>546 >>549 >>550 >>552
いろんなレベルの質問が雑多に混在していて2chらしくていいよなぁ
562ご冗談でしょう?名無しさん:2010/04/09(金) 10:50:17 ID:???
>>527
言葉足らずにならないように482を補足すれば

重心系で初期状態の全角運動量が0の場合を見てみる。
運動量が互いに逆向き、とは、電子の運動量の向きをz軸の正の向きにとると、
陽電子の運動量の向きはz軸の負の向きになる、という意味。
スピンの向きが互いに逆向き、とは、上記のz軸を量子化軸にとって各々のスピンの
向きを見ると、電子が上向きなら陽電子は下向き、逆に電子が下向きなら陽電子は
上向きになる、という意味。

したがって、運動量の向きに対するスピンの向き(=ヘリシティ、すなわち右巻きか左巻きか)を
見ると、電子が上向きで陽電子は下向きの場合はどっちも右巻きだし、逆に電子が下向きで
陽電子は上向きどっちも左巻きということになって、どっちの場合でも
電子と陽電子は互いに同じ巻き方になる
563ご冗談でしょう?名無しさん:2010/04/09(金) 11:36:07 ID:???
>>542
確かワイル粒子って言いますよね?
でもその議論ってmasslessでないと成り立たなかったような気がします
ニュートリノが質量を持っていたら、左右の区別がつかないのではないでしょうか
564ご冗談でしょう?名無しさん:2010/04/09(金) 12:32:10 ID:???
>>554
549です。
ありがとうございます。その方針で導くことができました!
565ご冗談でしょう?名無しさん:2010/04/09(金) 13:11:27 ID:???
>>563
だからヘリシティとカイラリティの区別をつけろと
566ご冗談でしょう?名無しさん:2010/04/09(金) 13:55:14 ID:???
>>552
> どっちになった方が良いのでしょうか?
好きにしたらいいが、どちらにもならない方が良いかもしれない

> 研究機関などはどっちの方が充実しているのでしょうか?
たぶん物理

> 収入
おなじ

> 世間一般のイメージ
高卒→頭のいい変人
大卒→世捨て人乙
厨ニ→凄えカッケー

> 地位
しらない

> 一般的に、どちらの方が知能が高いのでしょうか?
そんな一般論はない
567ご冗談でしょう?名無しさん:2010/04/09(金) 13:57:00 ID:???
>>565
ワイル方程式で別れるのはカイラリティですよね
568ご冗談でしょう?名無しさん:2010/04/09(金) 14:18:13 ID:???
宇宙の果てってどうなってるんですか?地獄ですか?
569ご冗談でしょう?名無しさん:2010/04/09(金) 14:19:11 ID:???
弱い相互作用の話が出ているのに、自由粒子を記述するワイル方程式だけを
持ち出しても仕方あるまい

570ご冗談でしょう?名無しさん:2010/04/09(金) 15:29:07 ID:81qZry2R
相対性理論の理解を短く書けって言われたんだけど正解がわからんな、、

つまり空間に対応した感覚の変化として時間がある。のでなく、認識者に対応した光の見
えの違いによって、認識者のいる時空のテンポのようなものがある

なんか変だろうか?
571ご冗談でしょう?名無しさん:2010/04/09(金) 17:54:52 ID:???
自分がどう理解しているかを書けという課題なら、何を書いても間違いにはならんだろ
572ご冗談でしょう?名無しさん:2010/04/09(金) 19:16:26 ID:???
まあハズレって事は無いんだけどさ、、
「テンポ」って表現はなんというか苦しいんだろうかな?とか
点の数に偏りのある数直線みたいな感じを出せればいいんじゃないか。
なんだろ。リズム?
573ご冗談でしょう?名無しさん:2010/04/09(金) 19:19:42 ID:???
やくしまるえつこにきけばいいよ
574ご冗談でしょう?名無しさん:2010/04/09(金) 20:04:44 ID:a1gazhHh
輝け甲子園の星(2010早春号)より
選抜高校野球選手権大会出場選手「好きなタレント」アンケート

1位 北川景子 49票
2位 上戸 彩 24票
3位 香里奈  23票
4位 佐々木希 22票
5位 松本人志 18票
6位 新垣結衣 17票
7位 南 明奈 黒木メイサ 15票
9位 木下優樹奈 12票
10位 市川由衣 11票
11位 EXILE 10票
12位 ダウンタウン 相武紗季 9票
14位 宮崎あおい 8票
15位 柴咲コウ 戸田恵梨香 井上真央 コトリッチ 7票
575ご冗談でしょう?名無しさん:2010/04/09(金) 20:38:11 ID:???
>>569
あ、そこが問題だったんですね
ということは、自由粒子としてニュートリノが振る舞っているときに
ニュートリノに質量がなければワイル粒子として扱え、
ニュートリノに質量があればワイル粒子として扱えない
ということですか?
576ご冗談でしょう?名無しさん:2010/04/09(金) 21:03:23 ID:???
hをディラック定数とします

[q,p]=ih
[q,q]=[p,p]=0
[q^†,p^†]=-ih
から
qp-p^†q^†
はいくらになるか導出できますか?
上の3式だけを使うのが条件です。
577ご冗談でしょう?名無しさん:2010/04/09(金) 21:21:05 ID:???
>>576
> [q,p]=ih
> [q^†,p^†]=-ih
何か変
578ご冗談でしょう?名無しさん:2010/04/09(金) 21:22:24 ID:???
>>575
それはニュートリノに限らずそうだが。
579ご冗談でしょう?名無しさん:2010/04/09(金) 21:33:49 ID:MIUAYGQ5
Pauli-Lubanskiベクトルが以下の様にどうしても0になってしまうのです。
どこが間違っているのでしょうか?

Pauli-Lubanskiベクトルは、
 W^μ=1/2 * ε^μνρσ * P_ν * L_ ρσ
ですが、
 L_ρσ = x_ρ * P_σ - x_σ * P_ρ
を代入して、
 W^μ=1/2 * ε^μνρσ *P_ν *(x_ρ * P_σ - x_σ * P_ρ)
=ε^μνρσ *P_ν *x_ρ * P_σ
交換関係
  [x_ρ,P_ν]= i g_ρν
を使って、xを左に移動させると、(P_ν * x_ρ= -i g_ρν+x_ρ*P_ν)
 P_ν *x_ρ * P_σ=( -i g_ρν+x_ρ*P_ν)* P_σ
 = -i g_ρν* P_σ+x_ρ*P_ν* P_σ
だから、
 W^μ= ε^μνρσ *(-i)g_ρν* P_σ+ε^μνρσ *x_ρ*P_ν* P_σ
右辺第1項のg_ρνがρνについて対称、2項もP_ν* P_σがνσについて対称。
ところが、ε^μνρσは逆にρν、νσについて反対称だから結局すべて相殺されて、
 W^μ=0
、、、。

どこが間違っているのでしょう?
580ご冗談でしょう?名無しさん:2010/04/09(金) 21:52:15 ID:???
>>579
スピン
581ご冗談でしょう?名無しさん:2010/04/09(金) 22:14:12 ID:???
携帯で色んな色に光る部分ありますよね
メールや着信で色を分けられたりするような
あれってどういう原理で光ってるんですか?どうしてカラフルにグラデーションを光れるのですか
582ご冗談でしょう?名無しさん:2010/04/09(金) 22:29:36 ID:vN261YMm
GBRの組み合わせで強弱つけてるだけじゃね?
583ご冗談でしょう?名無しさん:2010/04/09(金) 23:00:22 ID:UueOv56P
オウムに物理卒の人が多いのは何故ですか?
584ご冗談でしょう?名無しさん:2010/04/09(金) 23:41:14 ID:bdiDLUUa
電磁場の量子論では光のエネルギーは振動数に比例することになっていますが、
電磁場の古典論では光のエネルギーは何に比例するのですか?
585ご冗談でしょう?名無しさん:2010/04/09(金) 23:49:09 ID:???
>>582
元々は何で発光させてるんですかね
混ぜるまでの過程が全然分からないです
586ご冗談でしょう?名無しさん:2010/04/09(金) 23:50:12 ID:???
振幅の二乗
587ご冗談でしょう?名無しさん:2010/04/09(金) 23:55:42 ID:???
588ご冗談でしょう?名無しさん:2010/04/09(金) 23:56:44 ID:bdiDLUUa
>>586
電場の振幅と磁場の振幅のどっちですか?
589ご冗談でしょう?名無しさん:2010/04/09(金) 23:59:09 ID:???
>>578
でも質量が0のフェルミオン、つまりワイル粒子の候補って現在のところニュートリノくらいしかないですよね
そのニュートリノが質量を持つとすればワイル粒子のように左右が分離した粒子は存在しないということですか?
590ご冗談でしょう?名無しさん:2010/04/10(土) 00:03:04 ID:???
>>588
どっちもです
ってどの教科書にも書いてると思う
591ご冗談でしょう?名無しさん:2010/04/10(土) 00:10:06 ID:aC1/Nvnp
てことは電場の振幅をA、磁場の振幅をBとすると、電磁場のエネルギーは
A×Bで表せるんですね。
592ご冗談でしょう?名無しさん:2010/04/10(土) 00:26:12 ID:???
それはポインティングベクトルだな
593ご冗談でしょう?名無しさん:2010/04/10(土) 00:38:26 ID:aC1/Nvnp
ポインティングベクトルと電磁場のエネルギーはどう違うのですか?
594ご冗談でしょう?名無しさん:2010/04/10(土) 00:59:47 ID:???
どの教科書にも(ry
595ご冗談でしょう?名無しさん:2010/04/10(土) 01:10:08 ID:aC1/Nvnp
ポインティングベクトルの振幅がエネルギーという理解でよろしいですね?
596ご冗談でしょう?名無しさん:2010/04/10(土) 01:18:05 ID:???
ポインティングベクトルは電磁場のエネルギー流束密度 (運動量密度でもある)
電磁場のエネルギー密度は (>>591の記法に従うなら) A^2 + B^2 (の定数倍) だな
597ご冗談でしょう?名無しさん:2010/04/10(土) 01:26:54 ID:???
古典場の理論では密度でないとエネルギーが定義できないんだな。
密度の概念と無関係なエネルギー量子とは随分違うな。

量子論では空間の概念を離れてエネルギーが定義できるということか。

確かにこれは古典論と全く違うね。
598ご冗談でしょう?名無しさん:2010/04/10(土) 01:56:41 ID:???
>>552
語るに落ちててハゲワロスw

偏差値が一番高いので安直に「理科V類」を選んだだけで、
「理科V類」が医学部へ進学することも知らなかったんだねw
599ご冗談でしょう?名無しさん:2010/04/10(土) 02:17:17 ID:/1wKp6m/
今存在する宇宙の何処かでビッグバンが起きたらどうなるの?
600ご冗談でしょう?名無しさん:2010/04/10(土) 02:22:12 ID:???
>>598
別に理Vは医学部に行かなければならないというルールはないよ。
601ご冗談でしょう?名無しさん:2010/04/10(土) 02:32:20 ID:???
>>597
それは勘違いで、
光子一個の大きさをいずれ空間使って定義しないといけないので
あんま変わんないです
602ご冗談でしょう?名無しさん:2010/04/10(土) 04:20:10 ID:???
いいかげん物理で+1と-1を等価に扱うのはやめたほうがいいと思う
+iと-iは真に等価なのでどちらで考えても結果は同じだけど
+1と-1では累乗根の結果がまったく異なるわけだから
本来-1で考えなければいけない場面を1に置き換えたり+1で考えなければ
ならない場面を-1に置き換えて演算しない方がいいと思う
603ご冗談でしょう?名無しさん:2010/04/10(土) 04:30:42 ID:???
四元数が3次元空間の座標変換に上手く適用出来るのは、3次元空間の座標系が
純粋に虚数だから上手くいくんだと思う
そして時間座標だけが実数なので物体が移動する方向を決めるとその方向が
時間軸となるので時間が逆行できないのは時間軸が実数だからという説明だけで終わる
空間座標は3軸方向がすべて虚数なので等価に扱えるだけ
604ご冗談でしょう?名無しさん:2010/04/10(土) 04:36:14 ID:???
電磁方程式が虚数を使うと計算が簡単になったり、波動方程式に虚数が現れるのも
空間の性質が最初から虚数で時間軸(運動方向)のみが実数だからと考えればつじつまがあう
605ご冗談でしょう?名無しさん:2010/04/10(土) 04:57:25 ID:???
-d(si)^2 = dt^2 + d(xi)^2 + d(yj)^2 + d(zk)^2
606ご冗談でしょう?名無しさん:2010/04/10(土) 05:53:34 ID:???
「無」は無限永遠を超越しますか?
607ご冗談でしょう?名無しさん:2010/04/10(土) 06:02:24 ID:???
するする
608ご冗談でしょう?名無しさん:2010/04/10(土) 07:59:27 ID:???
>>607
理由は?
609ご冗談でしょう?名無しさん:2010/04/10(土) 08:03:34 ID:???
>>580
>スピン
もう少し詳しくお願いします。
610ご冗談でしょう?名無しさん:2010/04/10(土) 11:01:52 ID:vFOey5lm
虚数は、方程式を先に出して、軸やベクトルを出す。昔、言われたことがある。飛びすぎか?
611ご冗談でしょう?名無しさん:2010/04/10(土) 11:28:20 ID:???
>>609
重心系で角運動量=0のときW^μ=0は当然。>>579の計算は正しい。
612ご冗談でしょう?名無しさん:2010/04/10(土) 11:31:59 ID:???
わからないことをここで質問して、今までもいつもたいへん勉強になって来ました。
最近よく耳にする言葉で「ゼロ磁場」というものがありますが。
このゼロ磁場の状態というのは、トンデモ科学上だけでの話しなのでしょうか?
それとも、何か特定の条件を整えれば地球上で可能になる状態なのでしょうか?

よろしくお願いします。
613ご冗談でしょう?名無しさん:2010/04/10(土) 11:37:55 ID:???
>>612
ググれば一瞬でトンデモだと分かるだろ。
http://www.google.co.jp/search?num=50&hl=ja&lr=lang_ja&q=%E3%82%BC%E3%83%AD%E7%A3%81%E5%A0%B4

磁場が0というのは物理的に可能。
そこから導かれる健康にうんたらとかは全部とんでも。
614ご冗談でしょう?名無しさん:2010/04/10(土) 11:41:03 ID:???
>>612
ゼロ磁場は100%トンデモ間違いなし。
その単語を出すこと自体ここではスレ違い。
まじめに聞いているように見えたのでレスしたが本来はスルー。
615ご冗談でしょう?名無しさん:2010/04/10(土) 11:46:08 ID:???
>>612-613
ありがとうございます。
オカルティックなことが好きな友人と話しをしていて、どうやって説き伏せたらw
いいのかな。と思いまして、質問しました。
物理的には可能。という説明を出来るだけでも、わたしにとっては進歩です。
616ご冗談でしょう?名無しさん:2010/04/10(土) 11:47:02 ID:???
あっ、お礼を言うのは、>>613>>614でしたね。失礼しました。
617ご冗談でしょう?名無しさん:2010/04/10(土) 12:08:04 ID:???
超伝導で磁場遮蔽すれば…限りなく小さく出来るんじゃないかな
618ご冗談でしょう?名無しさん:2010/04/10(土) 14:26:18 ID:???
反物質のことで疑問におもったんですが、電荷が反対意外同じの粒子どうしが対消滅するなら
反陽子と正中性子をぶつけても対消滅はしないということでいいんでしょうか?
619ご冗談でしょう?名無しさん:2010/04/10(土) 14:27:33 ID:???
良くない。陽子も中性子も素粒子じゃない。
620ご冗談でしょう?名無しさん:2010/04/10(土) 14:35:12 ID:DVVj+6UW
量子力学を復習していたら変なところではまってしまいました。
いまpを運動量演算子、xを位置演算子として期待値<Ψ|px|Ψ>を計算したいのですが、
この式を<Ψ|pxΨ>と解釈した時と<pΨ|xΨ>と解釈したときでは答えが異なるのです。
どちらが正しいのでしょうか?
621ご冗談でしょう?名無しさん:2010/04/10(土) 14:42:34 ID:???
>>620
意味不明。
622ご冗談でしょう?名無しさん:2010/04/10(土) 14:45:52 ID:???
神学者と物理学者ってどっちの方が頭が良いの?
視野や心が広いのはたぶん、神学者だとは思うけど・・・。
623ご冗談でしょう?名無しさん:2010/04/10(土) 14:48:00 ID:???
>>620
そっちが正しいよ
624ご冗談でしょう?名無しさん:2010/04/10(土) 14:54:49 ID:???
>>622
ニーチェのアンチクリストでも読んでろ。
625ご冗談でしょう?名無しさん:2010/04/10(土) 15:05:16 ID:???
>>620
それぞれ具体例を書いてみて
626ご冗談でしょう?名無しさん:2010/04/10(土) 15:12:50 ID:DVVj+6UW
>>621,>>625
積分∫exp[-ikx]・D・x・exp[ikx]dxを計算するにあたって、
この積分を馬鹿正直に∫exp[-ikx]・{p・x・exp[ikx]}dxと計算するのと、
運動量演算子のエルミート性より∫{p・exp[-ikx]}・{x・exp[ikx]}dxと計算するのとでは、
計算結果が違ってくる。どちらが正しいのかということ。
ここではp運動量演算子はx位置演算子。
というわけでお願いします。
627ご冗談でしょう?名無しさん:2010/04/10(土) 15:13:47 ID:???
>>626
死ね。
628ご冗談でしょう?名無しさん:2010/04/10(土) 15:18:39 ID:???
ゲーテとノイマンってどっちの方が天才なの?
629ご冗談でしょう?名無しさん:2010/04/10(土) 15:33:28 ID:???
>>626
各々をどう計算してどういう結果になったのかを詳しく書いて
630ご冗談でしょう?名無しさん:2010/04/10(土) 16:18:23 ID:MCTd7xA9
スペースシャトルとかの無重力でセックスしたらどうなるの?
631ご冗談でしょう?名無しさん:2010/04/10(土) 16:30:25 ID:???
天才のランキング(トップ30)

1位、アリストテレス 2位、アルキメデス 3位、エウクレイデス 4位、ガウス
5位、オイラー 6位、パスカル 7位、ピタゴラス 8位、フェルマー
9位、デカルト 10位、ゲーテ 11位、ニュートン 12位、ライプニッツ
13位、ニーチェ 14位、アインシュタイン 15位、プラトン 16位、ヒューム
17位、ガリレオ 18位、リーマン 19位、カント 20位、マクスウェル
21位、ノイマン 22位、ダヴィンチ 23位、ラマヌジャン 24位、ガロア
25位、ハイデガー 26位、プランク 27位、ウィトゲンシュタイン 28位、ケプラー
29位、ホッブス 30位、ヒルベルト
632ご冗談でしょう?名無しさん:2010/04/10(土) 16:33:59 ID:???
やばっ、フロイトとダーウィンを忘れてた・・・・!
633ご冗談でしょう?名無しさん:2010/04/10(土) 17:32:27 ID:DVVj+6UW
>>629
∫exp[-ikx]・1/id/dx・x・exp[ikx]dxを計算する。
ただしデラック定数を1とおく。
1)∫exp[-ikx]・{1/id/dx・x・exp[ikx]}dx
=∫exp[-ikx]・{1/i(1+ikx)exp[ikx]}dx
=∫1/i・(1+ikx)dx
=1/i・x-1/2・kx^2
2)∫{1/i・d/dx・exp[-ikx]}・{x・exp[ikx]}dx
=-∫k・exp[-ikx]・{x・exp[ikx]}dx
=-∫kxdx
=-1/2kx^2
(1)と(2)は違う。
困った、困った。
634ご冗談でしょう?名無しさん:2010/04/10(土) 17:42:48 ID:???
∫{p・exp[-ikx]}・{x・exp[ikx]}dx
={1/i・d/dx・exp[-ikx]}・{x・exp[ikx]}dx
ではなくて
∫{p・exp[ikx]}^*・{x・exp[ikx]}dx
=∫{-1/i・d/dx・exp[-ikx]}・{x・exp[ikx]}dx
だろ。(*はエルミート共役。)
635ご冗談でしょう?名無しさん:2010/04/10(土) 17:51:24 ID:DVVj+6UW
>>634
ありがとうございます。
ご指摘の箇所も復習していてあいまいでわからなかったところです。
しかしエルミート共役ならば複素共役をとってかつ転置した結果、
運動量演算子の符号は元に戻りませんか?
よろしくお願いします。

個の符号の問題はあるにせよ、(1)と(2)が一致しないという問題は残りますよね。
636ご冗談でしょう?名無しさん:2010/04/10(土) 19:17:37 ID:DVVj+6UW
xとpが非可換である以上pxはエルミート演算子ではありません。
だから量<px>になにか物理的な意味はありますかと問われれば、
たぶんないだろうと答えることしかできません。
しかし量子力学の規則にしたがっているのに決定できない量があるということは、
あまり気持ちのよいものではありませせん。
だから皆様方よろしくお願いします。
637ご冗談でしょう?名無しさん:2010/04/10(土) 19:24:25 ID:???
平面波は微妙なので、ψ(x)=(規格化定数)exp(ikx-εx^2/2), ε>0 のような波動関数で計算して最後にε→0 とするのがよさそう。
638ご冗談でしょう?名無しさん:2010/04/10(土) 19:44:28 ID:DVVj+6UW
>>637
ありがとうございます。
さっそくやってみます。
639ご冗談でしょう?名無しさん:2010/04/10(土) 20:43:44 ID:sDLrtHP8
>>601
光子一個の大きさなんて定義できるの?
点粒子なんじゃないの?
640ご冗談でしょう?名無しさん:2010/04/10(土) 20:59:04 ID:???
クロネッカーデルタとディラックのデルタ関数

フーリエ変換で
Vを重み関数として

δ(x-x')=(1/V)∫exp[-ik(x-x')]dk
となるのは知っていますが、物理の教科書(量子)に
δx,x'=(1/V)∫exp[-ik(x-x')]dk
となっていました。
前者がdiracのδ関数で後者がクロネッカーのδです。
なぜこのようなちがいがでるのでしょうか?

xについて連続か離散かというちがいなのかな?
と自分は思っているのですが、どうなのでしょうか?
641ご冗談でしょう?名無しさん:2010/04/10(土) 21:08:10 ID:???
物理法則は経験則を元にして定式化したものですよね。
それは一応(近似的に)成り立っているみたいです。

実験をして、とれたデータは定式化する元となった要素なので、
式が正しいかどうかは堂々巡りになってしまうと思うのですが、
本当に正しいかどうかを確かめる方法は他にあるんでしょうか?
642ご冗談でしょう?名無しさん:2010/04/10(土) 21:13:57 ID:???
>>640
その教科書に「xが連続量の場合はδx,x'はδ関数を表す」というような注意はないか?
643ご冗談でしょう?名無しさん:2010/04/10(土) 21:16:02 ID:???
>>642
いえ、普通にないです。
これはクロネッカーのデルタである。
みたいなことしか書いてないです。
やはり連続化離散かで使い分けているということでいいでしょうか?
644ご冗談でしょう?名無しさん:2010/04/10(土) 21:22:24 ID:???
カール・フリードリヒ・ガウス vs サー・アイザック・ニュートン
645ご冗談でしょう?名無しさん:2010/04/10(土) 22:18:24 ID:???
>>641
物理学とは、物理現象を数学でモデル化する学問。
モデル化が目的なので、そのモデルの真偽は問わない。
より多くの物理現象を説明できるモデルを正しいとみなす。
本当に正しいかどうかを確かめる方法は存在しないし、
そもそも物理学はそれを目的としていない。
646641:2010/04/10(土) 22:26:13 ID:???
>>645
レスありがとうございます。
なるほど、モデル化できれば良いんですね。
では、次に、
物理に無限を適用すると様々な不具合が生じませんか?
例えば、プランク定数は物理量の下限ですよね。
限りなく小さくすると、かならず分解できる限界があるはずなのに、
無限を導入するということは必ずしも現実に沿ってないと思うのですが。
647ご冗談でしょう?名無しさん:2010/04/10(土) 22:47:04 ID:???
> 例えば、プランク定数は物理量の下限ですよね。
> 限りなく小さくすると、かならず分解できる限界があるはずなのに、
> 無限を導入するということは

どこに無限の話があるのかもわからないが。
648ご冗談でしょう?名無しさん:2010/04/10(土) 22:57:55 ID:kglKP0Wh
もし無限が現れたら、その場合にはその理論の適用範囲外と考えた方がいいのと違うか。
649ご冗談でしょう?名無しさん:2010/04/10(土) 23:05:29 ID:???
>>641
できあがった物理法則で自明でない予言をして、その成否を実験で確かめる。
650ご冗談でしょう?名無しさん:2010/04/10(土) 23:11:58 ID:???
>>646
何の不具合も生じない。
便利だから使う、それだけ。
651ご冗談でしょう?名無しさん:2010/04/10(土) 23:48:19 ID:???
ヘリウムは軽いので量子効果のせいで絶対固体になりませんが、水素は固体になりますよね?なぜなのでしょうか?水素はヘリウムよりも軽いのに。
652ご冗談でしょう?名無しさん:2010/04/10(土) 23:52:52 ID:???
ヘリウムがボース粒子だから
653ご冗談でしょう?名無しさん:2010/04/10(土) 23:57:25 ID:???
フェルミ粒子はなんで固体になるの?
654ご冗談でしょう?名無しさん:2010/04/11(日) 00:00:12 ID:???
ヘリウム3…
655ご冗談でしょう?名無しさん:2010/04/11(日) 00:00:37 ID:???
>>652
ボーズ粒子はなぜ固体にならないんですか?
656ご冗談でしょう?名無しさん:2010/04/11(日) 00:29:31 ID:???
>>639
ごめん書き方が悪かった
光子自身は素粒子で大きさはゼロだけどその存在しうる場所の広がりってこと
(大きい箱とか共振器の中とかパルスの中とか)

電磁場の量子化は、まず光子の存在しうる場所全体でのエネルギーを古典論と同じように計算して
それが hν の整数倍になるようにとるという意味で、古典論と量子論はそんなに違わない

古典論で自然な変数は場の振幅で、量子論で自然な変数は量子の数だから違うように見えるけど
657ご冗談でしょう?名無しさん:2010/04/11(日) 00:42:49 ID:???
なんと。
そういうことだったのか。
658ご冗談でしょう?名無しさん:2010/04/11(日) 08:23:08 ID:myVYj1XO
>>655
触媒つかって、固体にして、その後、触媒を抜けば、固体。小学生でも知ってるぞ。
659ご冗談でしょう?名無しさん:2010/04/11(日) 10:21:38 ID:???
>>652
水素も原子や分子としてはボーズ粒子だぞ。
660ご冗談でしょう?名無しさん:2010/04/11(日) 10:24:40 ID:myVYj1XO
超高速伝動、超高速通信、超高速通信プロコトル、超高速プログラム、波理論
http://slashdot.jp/science/article.pl?sid=09/07/03/1018210
登記
661ご冗談でしょう?名無しさん:2010/04/11(日) 10:45:11 ID:???
>>651
ヘリウムも加圧すれば固体になるというのはさておき、

ヘリウムは希ガス元素なので単原子として存在し、原子間力も弱い。
対して水素は分子として存在し、分子間力が強いので、熱運動に打ち勝って
結合することができる。
662ご冗談でしょう?名無しさん:2010/04/11(日) 12:58:33 ID:???
ログ読んでて分からなくなってしまったんだが
ニュートリノが左手で反ニュートリノが右手ってのはニュートリノがWeyl粒子かどうかって事と関係ないの…?
色んなHP見てるとWeylとCP対称性の破れが関連付いて書いてある所もあって
それはまさに弱い相互作用なのではないかと
詳しい方、解説お願いします
663ご冗談でしょう?名無しさん:2010/04/11(日) 18:45:55 ID:???
その方面とても疎いおれが解説しよう!まず、ニュートリノが質量を持てば、
右左巻きどっちも可能!多くの啓蒙書にある通り当該ニュートリノを
追い越すような系から見れば巻き方逆になるから。と、言う訳でいつでも
どこでも巻き方が一定であるためにはWeyl粒子でないといけない。
 一方弱い相互作用の話の場合、相互作用ハミルトニアンがいわゆるV-A
の形なので片方の巻き方の粒子しか作用し得ない(電子だって同じ)。
664ご冗談でしょう?名無しさん:2010/04/11(日) 20:36:24 ID:???
Liouville

これを日本語読みにしたらどうなるのですか?
665ご冗談でしょう?名無しさん:2010/04/11(日) 20:38:12 ID:???
リュービル
666ご冗談でしょう?名無しさん:2010/04/11(日) 21:50:17 ID:???
ふつうリュービーユと読むな。ベルサイユの綴りを調べてみな。
667ご冗談でしょう?名無しさん:2010/04/11(日) 22:04:54 ID:???
リュービウ
668ご冗談でしょう?名無しさん:2010/04/11(日) 22:29:09 ID:???
読む人の言語によるんじゃないか。
フランス人ならフランス語読みするし、英米国人なら固有名詞だろうが英語読みする。
日本人なら・・・ローマ字読みするしかないかな? リオウビレ
669ご冗談でしょう?名無しさん:2010/04/11(日) 23:01:00 ID:???
>>668
あんたアルファベットが並んでりゃ何でもローマ字で読めると思ってる?
サマセット・モームの綴りを見てみな。
670ご冗談でしょう?名無しさん:2010/04/11(日) 23:08:21 ID:???
>>669
読めるとは思ってないけど、読むしかないんじゃないか?
日本語で読めって言われたら。
671ご冗談でしょう?名無しさん:2010/04/12(月) 03:46:09 ID:???
Bjarne Stroustrup
ビヨーン・スポッスポッ!
672ご冗談でしょう?名無しさん:2010/04/12(月) 11:50:24 ID:ZIOPrisi
ボース粒子がスッポンポン。
673ご冗談でしょう?名無しさん:2010/04/12(月) 12:17:34 ID:???
>>670
シボレーをチェヴロレットと読むのが日本語だとそう主張したいのか?
674ご冗談でしょう?名無しさん:2010/04/12(月) 12:29:09 ID:???
>>673
別にそんな変な主張をしたいわけではないけど、
予備知識なしでアルファベット綴りだけを見せられて、
「日本語で読め」と言われたら読めるように読むしかないだろ。
そう言っているだけ。
675ご冗談でしょう?名無しさん:2010/04/12(月) 15:59:18 ID:???
読めないなら読めないと言えばいいだけでは
676ご冗談でしょう?名無しさん:2010/04/12(月) 16:17:20 ID:???
>>662
片っ端から「近似的に」ってつければいい。
677ご冗談でしょう?名無しさん:2010/04/12(月) 17:21:32 ID:???
>>675
それを言っちゃえばミもフタも無い
読めない単語を前に皆でああでもないこうでもないと言っているときに「読めない」と一言
678ご冗談でしょう?名無しさん:2010/04/12(月) 17:30:33 ID:???
そもそも、あーでもないこーでもないと言う必要なんてなかった

どう読むんですか?→こう読みます
という話に、ローマ字読みでリオウビレとかわけわからんこと言い出すアホが一人
679ご冗談でしょう?名無しさん:2010/04/12(月) 17:40:02 ID:ZIOPrisi
最初から、一つづつ、論理証明していけば?
680ご冗談でしょう?名無しさん:2010/04/12(月) 17:48:45 ID:???
>>678
じゃ、あれはどう読むんだよ。
何語として発音するのかを抜きにカタカナ化できるのか?あんたは。
681ご冗談でしょう?名無しさん:2010/04/12(月) 17:52:52 ID:???
>>678
現実に>>665-667でああでもないこうでもない、になっている。
それを無視して正論めいたことを書いてもバカ丸出し。
「こう読みます」の誰も異論のない正解をお前が書けるのなら別だが。
682ご冗談でしょう?名無しさん:2010/04/12(月) 17:55:07 ID:???
人名の読み方は地域によってもさまざまだからな。
日本でミハエルで通っている名前も、外国ではマイケルって呼ばれてたり。
683ご冗談でしょう?名無しさん:2010/04/12(月) 17:59:05 ID:???
フランス人だけどテキストなどでは普通はリューヴィル、リュウヴィルと表記する事が多いと思う。
やっぱリューヴィユの方がフランス語的には正しいの?
684ご冗談でしょう?名無しさん:2010/04/12(月) 18:04:06 ID:???
>>682
> 日本でミハエルで通っている名前も、外国ではマイケルって呼ばれてたり。

英語読みを耳できいたらマイコーとしか聞こえないし。
685ご冗談でしょう?名無しさん:2010/04/12(月) 18:15:51 ID:???
少なくともリオウビレがアホであることは誰にも異存がなかろう
686ご冗談でしょう?名無しさん:2010/04/12(月) 18:24:46 ID:ZIOPrisi
空間に、エネルギーをある一定掛けると、空間が歪む。その空間に、何かあったりして。イゼ・・・・・・。
687ご冗談でしょう?名無しさん:2010/04/12(月) 18:29:05 ID:???
>>685
それであんたはなんて読むの?
読めないの?
読めないのだったらなんで発言してるの?
688ご冗談でしょう?名無しさん:2010/04/12(月) 18:35:34 ID:???
やっぱリオウビレだな、日本語読みなら。
689ご冗談でしょう?名無しさん:2010/04/12(月) 19:17:45 ID:Cc8oKck9
じゃあ僕はこれからPoinsotはポインソトって読む
690ご冗談でしょう?名無しさん:2010/04/12(月) 19:27:08 ID:???
フランス人はHamiltonianをなんと呼んでおるのじゃ?
691ご冗談でしょう?名無しさん:2010/04/12(月) 19:51:16 ID:???
>>688
おまいはtheをトヘとでも読んでろ
692ご冗談でしょう?名無しさん:2010/04/12(月) 19:53:28 ID:???
>>691
お前は theme をシームと読んでろ
693ご冗談でしょう?名無しさん:2010/04/12(月) 19:55:31 ID:???
じゃあ、これは日本語読みではどうなるよ

Justine Henin
Kim Clijsters

年ごとに言い方がくるくる変わってる名前だぞ
694ご冗談でしょう?名無しさん:2010/04/12(月) 20:02:14 ID:???
>>693
なにも情報がなければ
 ジュスチネ ヘニン
 キム クリジステルス
とでも読むしかないだろ。
もし予備知識や情報があればそれに従うまでだ。
695ご冗談でしょう?名無しさん:2010/04/12(月) 20:04:24 ID:???
>>690
アミトーニャ?
696ご冗談でしょう?名無しさん:2010/04/12(月) 20:32:22 ID:???
院試の際の研究室訪問で貰ってきた論文があります。まあその院には落ちましたが。
んでその論文でわからないところがあるから質問のメールを出そうと思ってるんだが…

相手がドイツ人なんでこういう場合論文も英語だし最初から英語でメールを出す方が良いのか、
それともイミフな事にならないように日本語混じりでメールする方が良いのか…

どっちが良いと思う? ちなみに英作はかなり下手です、ハイ
697ご冗談でしょう?名無しさん:2010/04/12(月) 20:45:39 ID:???
脈アリだ、押せ
698ご冗談でしょう?名無しさん:2010/04/12(月) 20:46:47 ID:???
        / ○ \
   ○----\○ ○/---

のような豆電球の回路に流れる電流の比を、乾電池と豆電球一個ずつのときと比べて知りたいのですが・・・
図が下手で申ししわけありません。
699ご冗談でしょう?名無しさん:2010/04/12(月) 21:03:49 ID:???
>なにも情報がなければ
だからすでに>>665-667という情報がありながらリオウビレなどと言い出した
668が間抜けってことで終わりのはずでは?
700ご冗談でしょう?名無しさん:2010/04/12(月) 21:07:04 ID:???
>>699
なにを執着してるんだ。冷静になれ。
>>668の奴はローマ字読みするしかない場合はそうなると言ってるだけだ。
別にお前の何かを否定している訳じゃない。
701ご冗談でしょう?名無しさん:2010/04/12(月) 21:08:16 ID:???
>>699
> だからすでに>>665-667という情報がありながらリオウビレなどと言い出した

>>665-667は質問者ではなく他者が憶測で出した情報だから、元の問題に対する情報量はゼロだ。
702ご冗談でしょう?名無しさん:2010/04/12(月) 21:16:41 ID:???
>>698
どういう回路なのかさっぱりわからんぞ
703ご冗談でしょう?名無しさん:2010/04/12(月) 21:29:22 ID:???
>>698

ロボタンのAAにしか見えない・・・
704ご冗談でしょう?名無しさん:2010/04/12(月) 21:35:48 ID:???
>>698
なんか学研の科学の付録についてきそうw

それはそうと誰か>>696を頼む。
どっちでもいいとかそもそもどうでもいいとかは無しで
705ご冗談でしょう?名無しさん:2010/04/12(月) 21:38:56 ID:???
相手の日本語力による
706ご冗談でしょう?名無しさん:2010/04/12(月) 21:40:31 ID:???
>>700
そもそも言語とは何の関係もないのになぜローマ字読みをしなければ
いけないのか何も説明されていない。ローマ字読みするしかない場合、
などというわけのわからん条件指定をいきなり持ち出すこと自体が
じゅうぶん間抜けだと思うぞ。

>>701
どうみたって質問に対する回答の提示だが、それを他者の憶測であり
情報量ゼロだなどと言っていたら質問スレ自体が成り立たないだろ。
わけわからん
707ご冗談でしょう?名無しさん:2010/04/12(月) 21:41:54 ID:???
ああ、変換ミスで意味不明になってしまった。
「そもそも*原語*とは何の関係もないのになぜ…」だorz

708ご冗談でしょう?名無しさん:2010/04/12(月) 21:44:33 ID:???
>>696
英語で書けなくて論文の内容がわかるのか?
709ご冗談でしょう?名無しさん:2010/04/12(月) 21:46:22 ID:???
>Liouville

>これを日本語読みにしたらどうなるのですか?

そもそも日本語ではない文字を日本語読みにしたらどうなるかという
質問がすでに異常なんだから、どうでもいい。
710ご冗談でしょう?名無しさん:2010/04/12(月) 21:48:45 ID:???
外国人名をカタカナで書くのはさほど異常な行為とは思えないが
711ご冗談でしょう?名無しさん:2010/04/12(月) 21:49:37 ID:???
日本語ではない文字を日本語読みしろというのが異常だと言ってるの
意味わからんか?
712ご冗談でしょう?名無しさん:2010/04/12(月) 21:55:44 ID:???
 Liouville

 この読みは日本では一般にどう表記していますか?

などと訊くべきだということ
713ご冗談でしょう?名無しさん:2010/04/12(月) 21:56:23 ID:???
どーでもいいわ もう他所でやれよ
714ご冗談でしょう?名無しさん:2010/04/12(月) 22:08:55 ID:???
確かにどうでもいいなw
だが折角調べたから書く
カタカナで書いてる本はなかなか見つからないが
理化学辞典 リュウヴィル
ランダウ力学 リウヴィル
715ご冗談でしょう?名無しさん:2010/04/12(月) 22:18:15 ID:???
他分野の人間ですが相対論について質問させてください
他のスレだと荒れそうなんでw
光源が動いてるときは光の速度は変わらんのはわかるんですが
観測者が動いてる時も観測速度は変わらないということでいいんですよね?
中性子崩壊の光の速度なんかは凄いわかりやすいんですけど
やっぱりなかなかピンとこないもので・・・
716ご冗談でしょう?名無しさん:2010/04/12(月) 22:21:04 ID:???
> 観測者が動いてる時も観測速度は変わらないということでいいんですよね?

慣性系ならね。
加速系ではかならずしもそうではない。
717ご冗談でしょう?名無しさん:2010/04/12(月) 22:26:55 ID:???
>>716
有難うございました、勉強してみます
718ご冗談でしょう?名無しさん:2010/04/12(月) 23:32:37 ID:???
>>705
相手は日本語で講義してるから日本語で返信して良いよと言ってきています

>>708
まあ、それを言われるとな…読めるけど書けないという典型的な日本人のパターンに陥っていますわ…
719ご冗談でしょう?名無しさん:2010/04/13(火) 07:02:35 ID:f/33wZmJ
言語辞書使えお!!
720ご冗談でしょう?名無しさん:2010/04/13(火) 07:03:24 ID:f/33wZmJ
ドイツ言語辞書が有名だよ。
721ご冗談でしょう?名無しさん:2010/04/13(火) 08:42:05 ID:f/33wZmJ
エネルギー固体って、できない?
722ご冗談でしょう?名無しさん:2010/04/13(火) 08:47:31 ID:f/33wZmJ
過去スレに、あるエネルギー水を、固形化したり、結晶化すると、エネルギー固体、エネルギー結晶体ができる。発電所に使ってもよし、結晶体をエネルギー伝導、増幅に使っても良し、伝導体に使っても良し。
登記
723ご冗談でしょう?名無しさん:2010/04/13(火) 08:51:32 ID:???
孔子とガウスってどっちの方が知能高いんですか?
724ご冗談でしょう?名無しさん:2010/04/13(火) 08:53:22 ID:f/33wZmJ
この前、打ち上げた探査機に使われているから、フィールズでも、ノーベルでも、京都賞でも取ってよ
725ご冗談でしょう?名無しさん:2010/04/13(火) 08:54:29 ID:???
ニュートンとリーマンってどっちの方が知能高いんですか?
726ご冗談でしょう?名無しさん:2010/04/13(火) 09:08:17 ID:1Uxymhqq
今世界の優秀な数学者と物理学者で、素数の謎を解く事で、宇宙の謎、万物の謎が解る様な事テレビで見て衝撃的だったんですけど本当なんですか!
727ご冗談でしょう?名無しさん:2010/04/13(火) 09:25:52 ID:???
>>718
間違いなくお前の英語力より相手の日本語力のほうが高い
おとなしく日本語で書け
その際、一言詫び入れるといいかも
728ご冗談でしょう?名無しさん:2010/04/13(火) 09:41:37 ID:f/33wZmJ
超光速伝導の光を、エネルギー水に時計回りに回転させながら上方に通すと、反重力ができる。カナダとの研究チームとより。
登記
729ご冗談でしょう?名無しさん:2010/04/13(火) 09:59:13 ID:f/33wZmJ
東工大にインテリジェンスデバイスつくれる人いない?
730ご冗談でしょう?名無しさん:2010/04/13(火) 10:11:53 ID:f/33wZmJ
HNのテクノジーで、金属と金属をつなげるにのに成功しているよ ドイツ研究所より。
731ご冗談でしょう?名無しさん:2010/04/13(火) 10:13:40 ID:f/33wZmJ
MHとも言うが。
732ご冗談でしょう?名無しさん:2010/04/13(火) 13:15:07 ID:f/33wZmJ
ナノマシンは、フランスが成功しているが、失敗作だよ、でもどこかで成功しているいうことを聞いたことがある。ドイツかな?
733ご冗談でしょう?名無しさん:2010/04/13(火) 16:02:31 ID:f/33wZmJ
フランス語で、ナノマシン研究しているのパスーツ研究所だって。
734ご冗談でしょう?名無しさん:2010/04/13(火) 17:44:12 ID:???
>>727
こんなくだらない質問に答えてくれてサンクス
735ご冗談でしょう?名無しさん:2010/04/13(火) 18:51:10 ID:???
ドイツでドイツ人が日本語で講義しとるとなすげぇな。
ところでリチャード・キャンベルぐらい日本語流暢なのは理系の学部にも居るのかな。
つかあんんぐらいの人捕まえて日本語流暢って言う言い方は失礼過ぎるなスマソ。
736ご冗談でしょう?名無しさん:2010/04/13(火) 21:11:23 ID:???
ある現代物理学の本に書いてあったことで、
ニュートン力学では鉄を熱すると赤くなる、ということがうまく説明できなくなる
と。
簡単でいいですから教えてください
737ご冗談でしょう?名無しさん:2010/04/13(火) 21:16:33 ID:???
738ご冗談でしょう?名無しさん:2010/04/13(火) 21:18:15 ID:???
簡単でいいから少しは自分で考えてから質問してください
739ご冗談でしょう?名無しさん:2010/04/14(水) 01:51:38 ID:???
>>736
あらゆる物体はその組成に関係なく、温度のみに依存する色を発することが
分かっています。

その温度と色の関係を正確にみるためには測定対象物質自体が色を発していては
いけません。よって自身からは全く光を発しない(反射しない)物体を想定する必要が
あります。これを黒体と言います。
つまり色と温度の関係を調べるには黒体からでる光、即ち黒体輻射を温度に応じて
測ればいいことになります。

黒体輻射は低温では赤く光り、高温では青白く光ります。
古典物理では温度と輻射のエネルギーが関係することが説明できますが、色が変わる
ことまでは説明できません。
なぜなら、古典物理では電磁波のエネルギーはその振幅に比例することになっている
からです。振幅とは即ち光の強さです。

つまり、古典物理の枠内で考えると、黒体の温度を上げると輻射の強さが大きくなるはずであり、
色については何も予言しないのです。

この発見が古典物理の限界であり、即ち量子力学の黎明期でもあります。
740ご冗談でしょう?名無しさん:2010/04/14(水) 03:15:37 ID:???
>>739
この説明ちがうでしょ
741ご冗談でしょう?名無しさん:2010/04/14(水) 03:59:36 ID:???
>>740
違うと思うんなら訂正しなさいよ。
742ご冗談でしょう?名無しさん:2010/04/14(水) 08:15:32 ID:???
そもそも電磁気理論はニュートン力学に含まれるの?
743ご冗談でしょう?名無しさん:2010/04/14(水) 10:07:03 ID:???
>>739
>あらゆる物体はその組成に関係なく、温度のみに依存する色を発することが
>分かっています
そんなことわかってない。黒体についてはそう言えるが、現実の物体が
黒体とみなせるかは別問題。

>自身からは全く光を発しない
として黒体を想定しておきながら、直後に
>黒体からでる光
を測ればいい、とか、説明に自己矛盾がある。

>古典物理では温度と輻射のエネルギーが関係することが説明できますが、色が変わる
>ことまでは説明できません。
古典物理でも黒体輻射の色が変わることは言える。ただ、観測事実と短波長側で
合わないだけ。
744ご冗談でしょう?名無しさん:2010/04/14(水) 11:09:15 ID:???
ふむ。
まずまずだな。
745ご冗談でしょう?名無しさん:2010/04/14(水) 15:32:51 ID:???
古典で考えると波長0から∞までの全ての光が出てて、そのエネルギーの総和も無限大になるからでしょう。
746ご冗談でしょう?名無しさん:2010/04/14(水) 16:25:49 ID:???
747ご冗談でしょう?名無しさん:2010/04/14(水) 18:33:40 ID:aYD971iI
>>620-637

>>637
やってみたけれどだめでした。
そこで、
pxがエルミート演算子ではないから<Ψ|px|Ψ>も実数ではなくなる、
つまり<Ψ|pxΨ>≠<pxΨ|Ψ>。
結果、<Ψ|pxΨ>と<pΨ|xΨ>は等しくなくなる、と考えたのですがどうですか?
748ご冗談でしょう?名無しさん:2010/04/14(水) 19:17:41 ID:???
>>743
>古典物理でも黒体輻射の色が変わることは言える。
これは何で?

古典物理では光のエナジーはその振幅にしか依存しないのでは?
光の振動数の違いはどこから生じるのでつか?
749ご冗談でしょう?名無しさん:2010/04/14(水) 20:03:23 ID:???
>>747
>やってみたけれどだめでした。
詳しく。ところで、<Ψ|px|Ψ>を計算するときの積分は-∞から∞の定積分だというのはいい?
750ご冗談でしょう?名無しさん:2010/04/14(水) 20:18:33 ID:???
計算を全部書け
751ご冗談でしょう?名無しさん:2010/04/14(水) 20:58:39 ID:???
自然単位系で
hbar=c=1と置いて良いのは何でですか?
hbarとcってかなり値が違うと思うんですが
752ご冗談でしょう?名無しさん:2010/04/14(水) 21:07:34 ID:???
>>751
単位を決めなきゃ値は決まらんぞ?
753ご冗談でしょう?名無しさん:2010/04/14(水) 21:10:32 ID:???
>hbarとcってかなり値が違う
それこそまさに単位系によるだろ
754ご冗談でしょう?名無しさん:2010/04/14(水) 21:49:59 ID:???
ああなるほど。
言われてみれば仰る通りで
755ご冗談でしょう?名無しさん:2010/04/15(木) 00:10:42 ID:???
>>747
サクライの演習やれよ
756ご冗談でしょう?名無しさん:2010/04/15(木) 15:21:18 ID:qXHGGxHo
福神付けなど、漬物で、薬草補える。
登記
757ご冗談でしょう?名無しさん:2010/04/15(木) 15:56:38 ID:wc2xEW/A
>>749,>>750
ありがとうございます。
計算します。
ここでディラック定数を1とします。また^(*)はエルミート共役を表わします(>>634,ありがとうございます)。
(1)<Ψ|pxΨ>=∫exp[-ikx-1/2・εx^2]・{px・exp[ikx-1/2・εx^2]}dx
=∫exp[-ikx-1/2・εx^2]・{(1/i+x)・exp[ikx-1/2・εx^2]}dx
=∫exp[-εx^2]・1/i(1+ikx-εx^2・exp[ikx-1/2・εx^2]}dx
=-i√(π/ε)+0+i/2√(π/ε)
=-i/2√(π/ε)
ε→0にて
与式=-∞

(2)<pΨ|xΨ>=∫{p^(*)・exp[-ikx-1/2・εx^2]}・{x・exp[ikx-1/2・εx^2]}dx
   =∫{i・d/dx・exp[-ikx-1/2・εx^2]}・{x・exp[ikx-1/2・εx^2]}dx
   =∫i(-ikx-εx^2)exp[-εx^2]
   =0-i/2√(π/ε)
   ε→0にて
与式=-∞

最初に合わなかったのは計算間違いだったようです。
しかし状態を選んでやっと成立する関係とはあんまりうれしくないですね。
それとも自由電子の波動関数を考えるときには、常にガウス性とでもゆうべき性質、
電子の分布が常に正規分布すること、を考慮しなければならないということの現われなのでしょうか。

>>755
ありがとうぞざいます。
確かに<px>を求める問題がありますね。
ざっと見たところ、確かに噂道理サクライの問題はおもしろそうですね。
問題集だけ買ってしまいました。

以上、よろしくお願いします。
758ご冗談でしょう?名無しさん:2010/04/15(木) 16:39:15 ID:???
>>757
ちゃんと規格化すべし。それから、その計算と部分積分公式を見比べよ。
759ご冗談でしょう?名無しさん:2010/04/15(木) 21:17:22 ID:wc2xEW/A
>>758
規格化して計算しなおしました。
(1)<Ψ|pxΨ>π/ε=∫exp[-ikx-1/2・εx^2]・{px・exp[ikx-1/2・εx^2]}dx
  =∫exp[-ikx-1/2・εx^2]・{(1/i+x)・exp[ikx-1/2・εx^2]}dx
  =∫exp[-εx^2]・1/i(1+ikx-εx^2・exp[ikx-1/2・εx^2]}dx
  =-i√(π/ε)+0+i/2√(π/ε)
=-i/2√(π/ε)
<Ψ|pxΨ>=-i/2√(ε/π)
ε→0にて
         =0

(2)<pΨ|xΨ>π/ε=∫{p^(*)・exp[-ikx-1/2・εx^2]}・{x・exp[ikx-1/2・εx^2]}dx
   =∫{i・d/dx・exp[-ikx-1/2・εx^2]}・{x・exp[ikx-1/2・εx^2]}dx
   =∫(kx-iεx^2)exp[-εx^2]dx
   <pΨ|xΨ>=-i/2√(ε/π)
   ε→0にて
与式=0

ガウス積分は公式を用いました。
こんどこそ計算は合っていると思います。
やっぱり普遍的であらねばならない計算規則が状態を選ぶという点は疑問です。

よろしくお願いします。
760ご冗談でしょう?名無しさん:2010/04/15(木) 21:20:50 ID:???
ここでやらなくていいですよ
761ご冗談でしょう?名無しさん:2010/04/15(木) 21:44:25 ID:???
=∫(kx-iεx^2)exp[-εx^2]dx
=∫kx*exp[-εx^2]dx
=(-k/2ε)∫(-2εx)*exp[-εx^2]dx
=(-k/2ε)[exp(-εx^2)]
=∞
762ご冗談でしょう?名無しさん:2010/04/15(木) 23:09:57 ID:???
>>759
>やっぱり普遍的であらねばならない計算規則が状態を選ぶという点は疑問です。
んじゃ関数形を決めずにΨ(x)でやってみりゃいいんじゃね?
763ご冗談でしょう?名無しさん:2010/04/15(木) 23:44:49 ID:???
今手元に本がないので具体的な式は書けないのですがブラケット表記について質問です

ブラケットをばらして積分に直したり、ブラケットの中にブラケットを含んだ積分を押し込んだりする演算がありますよね。
その計算の根拠はブラケット(内積)の性質に拠るものなんでしょうか。それとも単なる計算規則なんでしょうか

本当は具体的な式を書いて質問するべきなのに申し訳ない
764ご冗談でしょう?名無しさん:2010/04/16(金) 00:11:33 ID:???
>>763
状態|ψ>を|x>展開するには
∫dx|x><x| = 1
をかけて
|ψ>=∫dx|x><x|ψ>=∫ψ(x)|x> ,ψ(x)=<x|ψ>
とする。

完全性
∫dx|x><x| = 1
は、基底{|x>}に対する仮定
765ご冗談でしょう?名無しさん:2010/04/16(金) 00:21:13 ID:???
>>764
質問文が質問文なので答えは得られないと思ってました。
回答していただきありがとうございます

Fランク大なもんで量子力学の授業で
「君たちはブラケットと聞いたらブラジャーを連想するといい」
みたいなふざけた講義しか聴けなかったんですよね。
というよりもその教授自身真面目に説明しても理解できないだろうと思ってたみたいですが
766ご冗談でしょう?名無しさん:2010/04/16(金) 16:35:56 ID:B7wXukX/
∂f(x,t)/∂tって
∂_tf(x,t)という書き方でもいいのでしょうか?
767ご冗談でしょう?名無しさん:2010/04/16(金) 17:14:49 ID:???
いい
768ご冗談でしょう?名無しさん:2010/04/16(金) 18:46:02 ID:kW5XFrSa
>>761
奇関数の積分だから0だよ。
769ご冗談でしょう?名無しさん:2010/04/16(金) 22:53:22 ID:???
絶対零度の反対の
絶対最高温度って何ケルビン?
770ご冗談でしょう?名無しさん:2010/04/16(金) 22:57:32 ID:???
一般的な物理では定義されていない。
771ご冗談でしょう?名無しさん:2010/04/16(金) 23:04:08 ID:???
つか理論的な上限は無い事になってるよね、今のところ。
772ご冗談でしょう?名無しさん:2010/04/16(金) 23:25:31 ID:???
ところでエネルギーってビッグバンからずっと保存されてるの?
773ご冗談でしょう?名無しさん:2010/04/16(金) 23:42:27 ID:???
総量としては保存されてるけど、宇宙がどんどん膨張してるから
エネルギー密度が下がって、平均温度もグンと下がって来たと言う話でそ?

インフレーション中に他の宇宙と分かれたとか言う話なら良く知らない。
774ご冗談でしょう?名無しさん:2010/04/16(金) 23:49:55 ID:???
いや、最高温度ってビッグバン以外ありえないのかなーと思って
775ご冗談でしょう?名無しさん:2010/04/17(土) 00:02:54 ID:???
そもそもビックバンの時に温度というものはあったのか?
776ご冗談でしょう?名無しさん:2010/04/17(土) 00:08:46 ID:???
ビッグバンと言うか、宇宙が始まった瞬間ってもし、それが一点から始まったのなら
密度無限大で温度(エネルギー密度)も無限大なんじゃ無いの?

つまり、特異点。

一点からでは無く、ある有限の大きさを持っていきなり生まれたと言う説もあるみたいだが、
どちらにしろ今のところこれだと言う定説は無いでしょう?
777ご冗談でしょう?名無しさん:2010/04/17(土) 00:11:19 ID:???
>>776
一点から始まったという前提からしておかしい
長くなっちゃうから簡単に言うと、至るところでビックバンが起きたという方が正しい
ビックバンの時には、ここもそこのあれもあそこも全て同じだった
778ご冗談でしょう?名無しさん:2010/04/17(土) 00:27:42 ID:???
>>777
ここもそこもすべて同じになるほど、宇宙のサイズ自体が昔は小さかったと言うのが、ビッグバン宇宙論でしょ?

「至るところで」ってその時も今と同じ大きさの宇宙が有ったと思ってるの?
779ご冗談でしょう?名無しさん:2010/04/17(土) 00:29:06 ID:???
話が噛み合ってないな
お互い揚げ足取りたいだけで本質を見てない
780ご冗談でしょう?名無しさん:2010/04/17(土) 00:30:29 ID:???
>>779
どのように本質を見てないと?
781ご冗談でしょう?名無しさん:2010/04/17(土) 00:38:30 ID:???
もしかしたら、宇宙(空間)自体が正の曲率で曲がって閉じている状態だった(境界は無いが有限の大きさ)と言うのを理解して無い?
782ご冗談でしょう?名無しさん:2010/04/17(土) 01:02:30 ID:???
ビッグバン宇宙論で言う「宇宙は(ほぼ)一点から始まった」と言うのは
現在と同じような広大な空間の「中の」どこか一点で始まったと言う意味じゃ無いよ。

「空間」そのものがほぼ一点にまで縮んでたと言う話。
783ご冗談でしょう?名無しさん:2010/04/17(土) 02:16:39 ID:???
いろいろ書いてたら、ホーキングの虚時間宇宙論に興味が出て来ちゃった。

特異点の消去とか時間軸は元々、空間軸のひとつ(同種のもの)だったとか素晴らしスグル。
784ご冗談でしょう?名無しさん:2010/04/18(日) 01:24:16 ID:???
哲学者ですが何か質問はありますか?
785ご冗談でしょう?名無しさん:2010/04/18(日) 01:53:50 ID:???
哲学者の存在意義は何ですか?
786ご冗談でしょう?名無しさん:2010/04/18(日) 09:01:45 ID:???
プランク長さとかを求める問題でディメンションから求めるのは分かる 
でも、プランク定数を使って計算するときに
プランク定数を2πで割った換算プランク定数(ディラック定数)を使う理由が分からない
ディメンションは変わらないからいいと思うのだけど・・・
不確定性原理とかと関係があるの?
教えてください
787ご冗談でしょう?名無しさん:2010/04/18(日) 09:29:28 ID:dAcAqqXk
そんなに待てなかったみたい。。。アゲ
788ご冗談でしょう?名無しさん:2010/04/18(日) 09:35:11 ID:???
>>782
お前はビックバンが複数あるという論を激しく否定しているわけだが。
789ご冗談でしょう?名無しさん:2010/04/18(日) 12:14:25 ID:T2Ck49TA
解析力学の問題です。
x-y平面上の点P(x,y)を変数(u,v)を用いて、x=coshucosv、y=sinhusinv と表す。

(1) u=一定ならびにv=一定なる曲線の方程式を求めて、曲線座標系 (u,v)が(x,y)平面上でどのような座標を張っているか図示せよ。

u=一定ならびにv=一定の意味が分からないのですが、これってcoshu等が定数になるということでしょうか?
790三流学生:2010/04/18(日) 13:12:16 ID:???
>>789
u=一定→u=const.として、vを動かしたときの軌跡をxy平面に書く。当然coshuは定数になるね。
v=一定のときはv固定。でやることは同じ。
791ご冗談でしょう?名無しさん:2010/04/18(日) 13:32:54 ID:???
u=一定 (でvは変化する)の曲線と v=一定(でuは以下ry)の曲線って意味だろ。
792ご冗談でしょう?名無しさん:2010/04/18(日) 14:08:18 ID:T2Ck49TA
ありがとうございます。
解決しました!
793ご冗談でしょう?名無しさん:2010/04/18(日) 18:15:06 ID:???
固体物理の質問です。

化学ポテンシャルの温度依存が状態密度のように次元で変わるのはなぜですか?
794ご冗談でしょう?名無しさん:2010/04/18(日) 18:20:14 ID:???
音波の質問はOKですか?

音波はなぜ伝搬するのですか?
よろしくお願いします
795ご冗談でしょう?名無しさん:2010/04/18(日) 18:33:48 ID:???
>>793 状態密度が次元で変わるから、だろ。μってどうやって決める?
ある関数のエネルギー積分がある値になる、って形で決めるはずだけど、
その際積分の重みに何がでてくる?
796ご冗談でしょう?名無しさん:2010/04/18(日) 18:43:06 ID:???
>>794
空気圧の振動(粗密波)が伝わるからです…と言う説明じゃダメ?

水中でも伝わるけど。
797ご冗談でしょう?名無しさん:2010/04/18(日) 18:46:01 ID:???
>状態密度が次元で変わるから

確かに状態密度が絡んでくるのはわかるんです。
μは状態密度にFD分布をかけてμを求めます
798ご冗談でしょう?名無しさん:2010/04/18(日) 18:46:34 ID:???
電力の単位である W(watt) は基本単位で表すと kg*m^2/s^3 となる、と
Wikipedia にあるのですが、なんで s^3 とかになるのか考えてもわかりません。
わかりやすい説明をいただけると助かるのですが…
799ご冗談でしょう?名無しさん:2010/04/18(日) 19:07:18 ID:???
W = J/s = N・m/s = kg・m^2/s^3
800ご冗談でしょう?名無しさん:2010/04/18(日) 19:10:08 ID:???
>>798
wattは時間当たりの仕事率の単位だから。

1kgの物体を1m/s^2で加速するのに必要な力が1N(ニュートン)。

1Nの力で物体を1m動かした時の仕事量(エネルギー)が1J(ジュール)。

結局 1J = 1N・m = 1kg m^2 / s^2

1wattとは1J / sの仕事率の単位だから、
1W = 1N・m / s = 1kg m^2 / s^3
801ご冗談でしょう?名無しさん:2010/04/18(日) 19:25:33 ID:???
「実在」と「存在」の違いを教えてください。
802ご冗談でしょう?名無しさん:2010/04/18(日) 19:27:12 ID:???
>>799-800
ありがとうございます。結局のところ私がわかっていないのは

> 1Nの力で物体を1m動かした時の仕事量(エネルギー)が1J(ジュール)。

これのようです。このエネルギーの定義がよくわかりません。

1Nというのは物体を加速させる力の単位ですよね。
でも、物体が一旦加速したら慣性でそのまま動き続けるのですから、
その物体が1m動くのも2m動くのもエネルギーは同じな気がするんですが、
いったい私は何を勘違いしているのでしょうか。
803ご冗談でしょう?名無しさん:2010/04/18(日) 19:47:27 ID:???
>>802
「1Nの力で物体を1m動かす」と言うのは1Nの力を物体に加え続けて、結果的に1m移動させる事。

摩擦が無いとした場合、物体の速度はどんどん上がって行きます。

その結果として物体の速度が何m/sになるのか?やその時の運動エネルギーは?とかは自分で計算してみてくださいw
804ご冗談でしょう?名無しさん:2010/04/18(日) 19:48:10 ID:???
>>802
速度が変化しない=加速度0=力0=何m動かしたところでエネルギー0
805ご冗談でしょう?名無しさん:2010/04/18(日) 20:10:22 ID:???
誰か>>801の質問に答えてください。お願いします。
806ご冗談でしょう?名無しさん:2010/04/18(日) 20:14:43 ID:???
運動量演算子p=-i∇と波動関数φに対して
pφとなっているとき
これのエルミート共役は
(pφ)~=φ~p~
ですよね?すると
p~=i∇
となり、φ~に演算p~が作用しなくなってしまう気がするのですが
何がおかしいのでしょうか?
少なくともp≠p~だと思うのですが…
807ご冗談でしょう?名無しさん:2010/04/18(日) 20:20:30 ID:???
誰か>>801の質問に答えてください。お願いします。
808ご冗談でしょう?名無しさん:2010/04/18(日) 20:26:26 ID:???
相転移はなぜ起きるのですか?
809ご冗談でしょう?名無しさん:2010/04/18(日) 20:26:33 ID:???
>>802
> 1Nの力で物体を1m動かした時の仕事量(エネルギー)が1J(ジュール)。
>これのようです。このエネルギーの定義がよくわかりません。

仕事量を計算するには必要条件が欠けていませんか?

>いったい私は何を勘違いしているのでしょうか。

等速運動と加速運動を混同していませんか?
810ご冗談でしょう?名無しさん:2010/04/18(日) 20:35:44 ID:???
>>801
「存在」には「実在」の全てを含むが「実在」には「存在」の全てをを含まない
811ご冗談でしょう?名無しさん:2010/04/18(日) 20:39:51 ID:???
では、実存とは何ですか?
812ご冗談でしょう?名無しさん:2010/04/18(日) 20:42:45 ID:???
>>795
Nが変化しないようにFD分布と状態密度の積を積分する。
その条件でのμが化学ポテンシャルです。

二次元でμが一定はなんとなくわかるのですが、
三次元ではなぜ温度が上昇するとμが減少するのでしょうか?
813ご冗談でしょう?名無しさん:2010/04/18(日) 20:49:02 ID:???
>>811
読んで字の如し
814ご冗談でしょう?名無しさん:2010/04/18(日) 20:52:38 ID:???
>>812
状態密度
815ご冗談でしょう?名無しさん:2010/04/18(日) 21:08:44 ID:???
>>814
??
816ご冗談でしょう?名無しさん:2010/04/18(日) 21:21:41 ID:???
統計力学において、化学ポテンシャルってのは物理的な考察を諦めた部分。
物理で説明がつかない現象は全部化学ポテンシャルのせいにして逃げる。
817ご冗談でしょう?名無しさん:2010/04/18(日) 21:48:39 ID:???
>>816が化学ポテンシャルを計算したことないのは良く分かった。
818ご冗談でしょう?名無しさん:2010/04/18(日) 21:51:26 ID:???
アルベルト・アインシュタインとベルンハルト・リーマンってどっちの方が天才なんですか?
819ご冗談でしょう?名無しさん:2010/04/18(日) 22:05:42 ID:???
>>806
サクライ読む
820ご冗談でしょう?名無しさん:2010/04/18(日) 22:08:13 ID:???
比べることなんてできないだろ
天才なんていうあいまいな言葉で。
ふたりとも科学や数学におおきな貢献をしてくれた大先輩
821ご冗談でしょう?名無しさん:2010/04/18(日) 22:40:15 ID:???
つか、数学に関してはリーマンの方が優秀だったはずだよね。
アインシュタインは数学そのものはそんなに得意じゃ無かったから、
同僚か誰かに助けてもらいながらリーマン幾何学を学んだとか、どこかで読んだぞ。

でも、アインシュタインが物理学の世界で不世出の大天才なのは変わらないけど。
822ご冗談でしょう?名無しさん:2010/04/18(日) 22:48:57 ID:???
すいませんが宿題でどうしてもわからないところがあるので質問させてください
x軸上を運動する点Pの時刻tでの位置xが、a、b、c、ω、λを定数としてx=ae^(-λt)sinωtで与えられるとき、速さが0になる時刻を求めよ。
という問題です
両辺をtで微分してx´=-λae^(-λt)sinωt+ωae^(-λt)cosωtになって、x´=0としてその方程式をtについて解けばいいというのはわかるのですが、この方程式ではどうしてもt=○、という風な解を出すことができません
レベルの低い質問で申し訳ないですが、どうか教えてください
823ご冗談でしょう?名無しさん:2010/04/18(日) 23:05:45 ID:???
e^(-λt)は0にならないし、a = 0 の場合は解は自明だから a≠0としていいわけで

 -λsinωt+ωcosωt = 0

を求めるだけだな。あとはもう少し自分で考えてみたらいいとおもうぞ。
824ご冗談でしょう?名無しさん:2010/04/18(日) 23:20:41 ID:???
>>818

数学(者)>>物理学(者)

なので、

必然的に、

ベルンハルト・リーマン(数学者)>>>アルベルト・アインシュタイン(物理学者)

ということになる。
825822:2010/04/18(日) 23:26:54 ID:???
>>823
ありがとうございます
 -λsinωt+ωcosωt = 0を変形するとω/λ=tanωtでしょうか・・・
でもこれでは問題に対する答えになっていませんよね・・・
826ご冗談でしょう?名無しさん:2010/04/18(日) 23:32:46 ID:???
>>822
数式いじるのもいいけど、
どういう運動してるのかわかる?
827822:2010/04/18(日) 23:46:10 ID:???
>>826
単振動だと思いますが・・・
828ご冗談でしょう?名無しさん:2010/04/18(日) 23:49:36 ID:???
vをグラフに書いてみよ
829ご冗談でしょう?名無しさん:2010/04/18(日) 23:54:46 ID:???
arctanという関数を知っているだろうか。知らないか。
830822:2010/04/19(月) 00:20:26 ID:???
>>828
vの式をサインに合成してグラフを書いてみましたが、これだとv=(係数)sin(ωt+α)という風に合成したときのαがわからないです・・・
>>828
知らないです、すいません
831ご冗談でしょう?名無しさん:2010/04/19(月) 00:22:21 ID:???
>>829
なんとトンチンカンな・・・
832ご冗談でしょう?名無しさん:2010/04/19(月) 00:26:02 ID:???
>>819
一応手元にあります
どの辺でしょうか・・・
833ご冗談でしょう?名無しさん:2010/04/19(月) 00:27:30 ID:???
>>831
え?、結局 arctanは要るでそ?
834ご冗談でしょう?名無しさん:2010/04/19(月) 00:29:19 ID:???
>>832
ブラケットの辺りを隅々までと演習やれば判ると思うよ
835822:2010/04/19(月) 00:31:57 ID:???
ごめんなさい、>>830の下の安価は>>829です
arctanというのがわからないと解けないのでしょうか?
明日指導教員に質問してみます、ありがとうございました
ながながと付き合わせてしまってすいませんでした
836ご冗談でしょう?名無しさん:2010/04/19(月) 00:39:10 ID:???
>>835
arctanはただのtanの逆関数だよ。
多価関数だけど。
837ご冗談でしょう?名無しさん:2010/04/19(月) 00:43:23 ID:???
>>835
その問題はtan-1なんか使わなくても解けるし、もし使うとしても最後に値を求めるときだけ。
問題理解の本質には何の関係もない。
838ご冗談でしょう?名無しさん:2010/04/19(月) 00:43:41 ID:???
数物に関する質問です
線形代数では、ある行列Aの上にバーを書けば複素共役を、A^*のように書けば共役転置を表しますが
物理の世界では物理量の複素共役を表現するときには*を使い、共役転置を表現するときには†を使いますよね?
これらの明確な区別ってあるのでしょうか?
私は現在数学科で、大学院試験では物理を受験するのですが、こういった定義は自分で記述するものなのでしょうか?
839ご冗談でしょう?名無しさん:2010/04/19(月) 00:54:41 ID:???
>>838
記号の用い方なんて分野毎の慣習だから、明確な区別などない
例えば、U(S,V,N) = U(T,V,N) なんて書き方は数学畑から見たら狂気の沙汰かもしれんが、
物理ではそのようなことは気にせずこういう書き方をするわけ

まあ、共役と随伴に関しては、
採点する教授が物理系だと分かっているなら、郷にしたがって後者を使うか、
どうしても前者を使いたければ答案中に一言断るべきだろう
840ご冗談でしょう?名無しさん:2010/04/19(月) 00:59:35 ID:???
>>837
求めるのは位置が0になる時刻じゃ無くて、速さ(微分)が0になる時刻だよ。

何か勘違いしていないか?
841ご冗談でしょう?名無しさん:2010/04/19(月) 01:00:30 ID:???
>>839
アドバイスありがとうございます
アスタリスクを見るとたまに混同してしまうのですが
書き方に関しては物理側を受け入れようと思います
842ご冗談でしょう?名無しさん:2010/04/19(月) 01:49:11 ID:???
>>840
いや、その点は勘違いしていない。
arctanを使ったら使わないよりスムースに解けるというんなら部分的でいいから示してくれ。
俺が勘違いしてたんなら謝るから。
843ご冗談でしょう?名無しさん:2010/04/19(月) 02:02:13 ID:???
>>842
いや、スムーズになると言うよりarctanを使わないと解けないと思うけど。
だって、sin ωtの波形がe^(-λt)で歪まされてるから波形がピーク(微分が0)になる位相がπ/2から微妙にずれるだろう?

そのズレ分を求めるのに必要だと思う。
844ご冗談でしょう?名無しさん:2010/04/19(月) 02:04:36 ID:???
>>842
最終的に sin(ωt + φ) = 0 に帰着するから、φを求めれば解は出たも同然だが、
φは tanφ=… とか sinφ=… (とかcosφ=…) な形で出てくるから、最終的に arctan
なり arcsin なりを使うことになると思うけど。
845ご冗談でしょう?名無しさん:2010/04/19(月) 02:26:38 ID:???
>>844
死ねゴミ
846ご冗談でしょう?名無しさん:2010/04/19(月) 02:37:41 ID:???
arctanを(arcsinやarccosも)つかわないで解けるというのならそれを示してくれ。
そのうえでならゴミ呼ばわりされてもかまわんぞ
847ご冗談でしょう?名無しさん:2010/04/19(月) 12:56:52 ID:???
>>846
(ところで俺は>>845じゃないよ)
数値を出すところでarctanが要るかもしれないけど、
もし俺が採点者なら「tanφ=・・・ を満たすφ」まで書いてあれば90点ぐらいやる。
せいぜい関数電卓を叩くかどうかの違いだから。
でももし arctan を使うことでエレガントに解けるんであれば是非教えてくれ。
俺には思いつかない。
848ご冗談でしょう?名無しさん:2010/04/19(月) 13:42:27 ID:5knP+G1d
日本物理学会の学会誌の投稿論文査読ってどのくらいかかる?
849ご冗談でしょう?名無しさん:2010/04/19(月) 13:43:09 ID:5knP+G1d
あ、時間の話ね
850ご冗談でしょう?名無しさん:2010/04/19(月) 13:46:03 ID:???
t=… の形で与えない限り、関数電卓たたけないと思うんだけど
それとも関数電卓って方程式といてくれるんだっけ?
851ご冗談でしょう?名無しさん:2010/04/19(月) 13:52:38 ID:???
> 速さが0になる時刻を求めよ。
という問題の答として
(0) x'(t)=0 を満たす t
(1) -λsin(ωt)+ωcos(ωt)=0 を満たす t
(2) tan(ωt)=ω/λ を満たす t
(3) t=(1/ω)(arctan(ω/λ)+nπ)
等のどこまでやるのが適切か、ということね。>>825は(2)では答になっていない
という解釈だが、逆三角関数を既知としないのであれば(3)にはできない。
852ご冗談でしょう?名無しさん:2010/04/19(月) 14:54:57 ID:???
ミハエル・シューマッハ並のF1ドライバーになるのと、
アイザック・ニュートン並の物理学者になるのは、
どっちの方が難しいのでしょうか?
853その1:2010/04/19(月) 17:36:09 ID:???
>>802
そこは結構ひっかかるとこだと思う。
基本は
1運動する物体はエネルギーを持ってて、静止させるとそのエネルギーは失うって前提がまずある。
 また静止した物体をその速さにするのに必要なエネルギーは、それを静止させるのに必要なのと同じである。
2また同じ大きさの力を同じ時間かけつづければ、物体の受けるエネルギーは同じであり、
 異なる大きさの力でも、速さが同じになるような時間をかけるなら、物体の受けるエネルギーは同じになる。
まぁこれは式で表された力積の前提としてにある、運動の基本的な定理(かな?)を述べたものだけど文字にすると長い。
3(運動量の方は)質量と速さの積が同じなら、運動する物体の持つエネルギーは同じという意味。
異なる質量であれば、速さがそれに応じて違ってくるからね。
1・2・3を合わせたのがmv=Ftで、それがこの式の中に含まれてる。

で、先に結論を言うと運動する物体のエネルギーの、2つの表し方で少しだけ理解が変わってくる。
結局は等価的、同値な表現なんだけどもさ。ただ基本は上記の運動量表現だから、そこを考えて見る。
854その2:2010/04/19(月) 17:37:40 ID:???
質量mの物体が速度v0で等速運動してて、これに力Fを時間tの間働かせて静止させるとする。
そのとき力をかけ始めた地点から静止するまでの距離をxとする。
このときの関係式は2つ表現できて、1つが運動量と力積が対応するもの。
mv0=Ft(公式v=v0+atだから0=v0+at=v0+(F/m)t=mv0+Ft∴−mv0=Ft。符号は向きが逆の意だけだから除外)
これは力│F│をどれだけの時間t加えれば静止させれるかによって、物体のエネルギーを判断するもの。
これは1で書いたように、静止する同質量の物体を同じ速さにするために必要な力積でもある。
で、この運動量表現をより一般化したのがmv0+Ft=mv1ってやつ。
静止した状態は、v0=0の場合になる。
これは物体の運動が変化した際の運動量の変化とそのエネルギーのやり取りを示す式。
で、これはmv=Ftが示すように、静止した状態を基準に考えた運動量又は力積の和でも表せる。
物体が速度v0で動いてて、これに力Fが働いて速度v1になるとき、速度の増加分をΔvすると
運動量表現だと、(mv0)+(mΔv)=(mv1)になる。
これは同時に力積の和でも書ける。(F0・t0)+(Ft)=(F1・t1)
(F0とt0は静止した状態から物体Aを速度v0にするための力と時間で
同様にF1とt1も静止した状態から物体Aを速度v1にするための力と時間)
これは静止を基準にとって分解したものの和で表せるってこと。
これは結局、運動量又は力積の単純な和で運動の変化とエネルギーのやり取りを表現できるってこと。
つまりE0+ΔE=E1って形で表現できるということ。
855その3:2010/04/19(月) 17:39:54 ID:???
で、もう1つの関係式が運動エネルギーと仕事が対応するもの。
これはFの力を加え続け、静止するまでに要した距離と力で運動する物体のエネルギーを判断するもの。
距離xは、速度v=f(t)=v0+atとしてx=∫f(t)dt=v0t+1/2at^2+C
静止するまでの距離xはx=∫[0,t]f(t)dt=v0t+1/2at^2
で、静止させるときは運動量のmv0=−Ftが成立するので、そこからt=−mv0/Fを代入してtを消去。
そうするとx=v0(−mv0/F)+1/2(F/m)(−mv0/F)^2=1/2(mv0^2)/−F ∴Fx=1/2mv0^2(Fは向きが逆の意だけなので除外)
この式の意味だけど、質量と速ささえ決まっていれば運動量でも運動する物体のエネルギーを一意に定められるけど
同時に運動量に一対一に対応する1/2mv^2という値が必ず存在する。
つまりmv⇔1/2mv^2という一対一の対応関係ができることから、これを用いても支障がないと思われる。

では、これを用いた際には上述した運動量や力積のような関係式はどうなるか。
速度v0からv1に変化した際、その速度変化の始点から終点までの距離をxとする。
力F,質量m、時間t、速度の増分をΔv、運動エネルギーの増分をΔEとする。
エネルギーのこの表現では、速度v0のときは1/2mv0^2、v1のときは1/2mv1^2となる。
問題は、1/2mv0^2+ΔE=1/2mv1^2が成立するかどうか。
定義より、ΔE=Fx=F(v0t+1/2at^2)=F(v0t+1/2Δvt)=Ft(v0+1/2Δv)=mΔv(v0+1/2Δv)
(左辺)=1/2mv0^2+ΔE=1/2mv0^2+mΔv(v0+1/2Δv)=1/2m(v0^2+2Δvv0+Δv^2)=1/2m(v0+Δv)^2
(右辺)=1/2mv1^2=1/2m(v+Δv)^2
左右が等しいので、これを運動する物体のエネルギーとして用いることに支障がないことがわかる。
ただ、この表現では運動量のときのように静止時を基準にした単純な和は成立しない。
856その4:2010/04/19(月) 17:42:41 ID:???
こうすると、一定の重力下なんかで物を上下させる際に時間を使わず
距離と力さえわかれば、簡単にそのエネルギーの変化分を知ることもできる。
また運動量はベクトル表示をさせることにすれば、より端的に微積分も表現できる。
こうして、運動量に代わって運動エネルギーが実用上用いられるに至ったと僕は理解した。

これ数年前に「中3 運動とエネルギー」ってスレがあってそこでも議論がされてたんだ。
自分もそこで指摘を受け、その時はうまく総括できなかったんだけど、今日時間かけてまとめてみた。
ここは結構紛らわしいとこで、力学の教科書にもその経緯とか書いてないし
物理学史にもあんまり詳しく意味とか書いてないんだよね。
で自分も悩んで、ようやく今一応このぐらいにまとめれてた。参考になるかは断言できないけど。
ホントは近代にライプニッツ派やデカルト派などによる論争があったらしいんだけど
そういう経緯が説明から排除されてるから、後世でも再びそこで同じように悩む人が出て
全く以って無駄なんだけどねぇ、同じことで悩むのはさ。

>でも、物体が一旦加速したら慣性でそのまま動き続けるのですから、
>その物体が1m動くのも2m動くのもエネルギーは同じな気がするんですが、
>いったい私は何を勘違いしているのでしょうか。
というのは、Fxをそのまま式通り受け取ってるからで
運動する物体のエネルギーの本来的な意味は運動量の方だから、
その表現形式が上述のように使い分けされてるだけとみなせばいいと思うよ。
857その5:2010/04/19(月) 17:53:39 ID:???
この仕事の概念は本来は滑車なんかの機械学で培われてきたものなんだよね。
地球上で物体をある高さに上げるときに、滑車を複数使うときなんかで
結局力は小さくてもその分長さが変わらず、その積が一定になるというもの。
これはてこの原理でも同じだけど、仕事の原理とか言ってたような。
こういう特殊な状況下での問題を考えるのに有効でもあるから、運動エネルギーが採用されてるんだと思う。

かなり長くなったので、適当に読む気が起きたら読んでください。
858ご冗談でしょう?名無しさん:2010/04/19(月) 18:13:34 ID:???
質問です。
例えば、60Wの電球を直接見ても目に異常はないが、60mWのレーザー光線を直視すると失明すらしてしまうのはどうしてでしょう?
859ご冗談でしょう?名無しさん:2010/04/19(月) 18:15:14 ID:???
「時空」に関して詳しく勉強をしたいのですが、どのような分野を勉強すれば良いのでしょうか?
860859:2010/04/19(月) 18:17:34 ID:???
特に、「時間」に関して詳しく勉強をしたいです。
861ご冗談でしょう?名無しさん:2010/04/19(月) 18:41:57 ID:???
>>859
まずはマクロスからだな
862ご冗談でしょう?名無しさん:2010/04/19(月) 19:52:32 ID:???
>>859
相対論から
>>860
現代の物理学では無理
来世で研究汁
現代の物理学では時間・空間が離散的である可能性すら否定できない
宇宙の形は全く分かってない(天体の大規模構造の事じゃない)
863ご冗談でしょう?名無しさん:2010/04/19(月) 19:57:23 ID:???
>>858
電球の光を集光したらどうかな
864ご冗談でしょう?名無しさん:2010/04/19(月) 23:44:53 ID:???
>>860
時間を研究しようと思うと意外に対象が広くなるんだよな。

例えば統計力学(エントロピー論)や非線形力学系、量子力学、素粒子物理学や宇宙論、生物学etc.

「これが時間だ」ってものがまだ分かっていない分、
いろんな事を研究やら勉強しないと「時間」の全体像がさっぱり見えて来ない。

一体どれが本質なのかって? 知らないw
865ご冗談でしょう?名無しさん:2010/04/20(火) 00:19:17 ID:???
>>858
レーザーは、そのエネルギーが網膜のほぼ1細胞に集光するから。
866ご冗談でしょう?名無しさん:2010/04/20(火) 02:29:15 ID:???
ウルトラコンピュータって作れますかね?
867ご冗談でしょう?名無しさん:2010/04/20(火) 03:13:26 ID:???
>>866
おまいさんが作ったそのウルトラコンピュータとやらの用語をどう定義するかによるだろう
例えば、既存の全てのスーパーコンピュータより処理速度が早いコンピュータと言うならその答えは確実にイエスだ
868ご冗談でしょう?名無しさん:2010/04/20(火) 19:57:58 ID:???
ウルトラマン・コンピューターなら昔有ったぞ。
869ご冗談でしょう?名無しさん:2010/04/20(火) 20:42:25 ID:Uc5iswxN
野菜からつくるHigh フォトジェニック水晶は、船を動かす力がある。エネルギー結晶体のこと。東大、京都大開発済み。
870ご冗談でしょう?名無しさん:2010/04/20(火) 21:14:14 ID:???
スピノザとアインシュタインはどちらの方が想像力があるのでしょうか?
871ご冗談でしょう?名無しさん:2010/04/20(火) 22:06:35 ID:uiWvdhbq
大学の前期試験の筆記にも、そろそろ慣れる準備しておくこと。

物質振動数って知っているかい?それが、答えだよ。質問者君。

次は、計算と、部分積分公式、部分積分積方程式、方程式ね。見比べられる?

>>759
場の理論で、方程式だしてみ。日本語でもいいから。

男は昔からいた。女は、戦闘兵器として、作り出された生物兵器で失敗作。古文書解析の考古学者は知っているよ。古文書解析考古学者で、年代はわかるだろ。
http://tsushima.2ch.net/test/read.cgi/newsplus/1271362760/

なぜ地域差があるのに銀行泥棒するのか?

細胞再生2回くらい じゃがいものでんぷん パセリ 八百年生きる薬(酵母でつくった)牛乳でつくったやつ(一日熟成させたもの) 日本人の肉体再生細胞2回くらい
登記

永遠超えて生きる常食(牛乳でつくったやつを4日間熟成させたものと
、牛乳でつくったやつを一日熟成させたものを混ぜた常食)で、鬱が微小に治る
登記

こばと。の最終回を3回半見て、夏影を1回見て、休み、睡眠、休日を充分取って。

永遠超えて生きる常食(牛乳でつくったやつを4日間熟成させたものと
、牛乳でつくったやつを一日熟成させたものを混ぜた常食)で、鬱が微小に治る
登記


872ご冗談でしょう?名無しさん:2010/04/20(火) 22:09:01 ID:???
温度の関数として化学ポテンシャルが三次元で減少し、二次元では一定である理由を
言葉で述べるとどうなりますか?

状態密度を使うのはわかっていますが、言葉ではどのようにあわらせばよいのでしょうか?
873三流学生:2010/04/20(火) 22:15:10 ID:???
>>872
状態密度をどうやって使うの?説明してみて。
874ご冗談でしょう?名無しさん:2010/04/20(火) 22:17:23 ID:???
>>872
状態密度、次元ということばを使って答えろ
という問題なので・・・
875ご冗談でしょう?名無しさん:2010/04/20(火) 22:20:50 ID:???
>>872
こういう質問は難しいので、答えられる奴はいないだろう、ここにはな。
876ご冗談でしょう?名無しさん:2010/04/20(火) 22:28:46 ID:BLLhfzDI
sinとcosって難しい?
877ご冗談でしょう?名無しさん:2010/04/20(火) 22:41:33 ID:???
誰か>>870の質問に答えてくれる方は居ないでしょうか?
878三流学生:2010/04/20(火) 23:06:26 ID:???
>>872
ちなみにどんな系なの?授業の内容だと勝手に理解しちゃうけど、
フェルミ系とか、ボゾン系とか、ポテンシャルがどうとか、ちゃんと指定してあると思うよ。
879ご冗談でしょう?名無しさん:2010/04/20(火) 23:07:57 ID:???
なんで物理という学問が存在するのですか?
880ご冗談でしょう?名無しさん:2010/04/20(火) 23:08:30 ID:???
>>878
固体物理の授業なのですが、
フェルミ系の話です。
できるだけ式を使わずにということなので、ゾンマーフェルト展開も使わないで
評価したいのです
881三流学生:2010/04/20(火) 23:14:58 ID:???
>>880
まあ、日本語でっていうのは「機械的に∂μ/∂Tを求めるのではなく」という意味くらいだと思うけど・・・。
FD分布関数と状態密度関数つかって粒子数を評価する式与えて被積分のグラフ形から説明するくらいでいいんじゃない?
882三流学生:2010/04/20(火) 23:15:58 ID:???
被積分→被積分関数
わかると思いますが。
883ご冗談でしょう?名無しさん:2010/04/20(火) 23:18:31 ID:???
正規分布になったテスト結果の中からいわゆる偏差値60以上の精鋭を集めて
テストさせたらその中でまた正規分布が出来上がるのはなぜですか?

正規分布は多重構造になっているということですか?
884ご冗談でしょう?名無しさん:2010/04/20(火) 23:25:37 ID:???
テスト問題の作り方が上手いんだろう
885ご冗談でしょう?名無しさん:2010/04/20(火) 23:26:14 ID:???
被積分のグラフ形から説明する
これがむずかしい
886ご冗談でしょう?名無しさん:2010/04/20(火) 23:41:41 ID:o/jhlocY
>>883
最初と抽出後では、平均が違うから

正規分布は、平均から外れると指数関数的に減っていくという分布だから、
平均の異なる統計しなおしなら、また指数関数的になる。

現実的な受験者数だったら、
同じ正規分布じゃなくて、やや扁平になると思うけど
887ご冗談でしょう?名無しさん:2010/04/20(火) 23:53:54 ID:???
でも最初の分布を偏差値60で切ったら抽出後の標本は偏差値60付近に
偏ってるよね?
888ご冗談でしょう?名無しさん:2010/04/20(火) 23:56:27 ID:???
>>883
ほんとにそうなるの?
889ご冗談でしょう?名無しさん:2010/04/20(火) 23:59:40 ID:???
集団の中から落ちこぼれを排除して精鋭を集めても、その精鋭の
中からも落ちこぼれが出てくるから、そうなんじゃない?
890ご冗談でしょう?名無しさん:2010/04/21(水) 00:03:15 ID:???
「××なのはなぜですか?」の答が「××とは限らない」はありがち。
891ご冗談でしょう?名無しさん:2010/04/21(水) 00:08:46 ID:???
>>887
偏差値ってもの自体がね、平均と分散から計算されるから、

抽出した段階で話が違って来るんだよ
892三流学生:2010/04/21(水) 00:15:29 ID:???
>>883
正規分布を成す母集団からランダムに標本を持ってきて統計をとると
やっぱり正規分布になるっていうのは知ってるけど・・・
一般に標本分布の標準偏差は母集団のそれより小さくなる。

ちなみにそれってどこに載ってたの?
893ご冗談でしょう?名無しさん:2010/04/21(水) 00:21:57 ID:???
>>891
あーなるほど。
偏差値50と51との間の得点差と偏差値60と61との得点差は、両方とも
偏差値の差は1で同じだけど、得点差は違うということだね。

偏差値60以上の標本でそのまま統計をとったらやっぱり正規分布になるんですね?
894ご冗談でしょう?名無しさん:2010/04/21(水) 00:26:05 ID:+qS4PX84
>>893
そう。
理想的な分布なら、形(標準偏差)は元とは変わるが、やっぱり正規分布になる。

現実的には、片側を切るとハズレ値の影響が大きくなったり、χ2分布に近くなったりと
「高得点者」という性質が効いて来てきて理想的な分布にはならないんだけど
895三流学生:2010/04/21(水) 00:32:28 ID:???
>>894
ランダム抽出じゃ無いのにどうしてχ2が出てくるのか理解できない。
896ご冗談でしょう?名無しさん:2010/04/21(水) 00:37:31 ID:???
>>895
F分布の間違い
897ご冗談でしょう?名無しさん:2010/04/21(水) 01:31:36 ID:???
ポアソン分布のことですね?
898ご冗談でしょう?名無しさん:2010/04/21(水) 01:39:39 ID:BC7cqPlN
細胞再生2回くらい じゃがいものでんぷん パセリ 八百年生きる薬(酵母でつくった)
牛乳でつくったやつ(一日熟成させたもの) 日本人の肉体再生細胞2回くらい
登記

永遠超えて生きる常食(牛乳でつくったやつを4日間熟成させたものと
、牛乳でつくったやつを一日熟成させたものを混ぜた常食)で、鬱が微小に治る
登記

こばと。の最終回を3回半見て、夏影を1回見て、休み、睡眠、休日を充分取って。
永遠超えて生きる常食(牛乳でつくったやつを4日間熟成させたものと
、牛乳でつくったやつを一日熟成させたものを混ぜた常食)を常食すると、アディクションが多少治る
登記

こばと。の最終回を3回見て、夏影を1回見て、休み、睡眠、休日を充分取って。
http://say-move.org/comeplay.php?comeid=59569
夏影は自分で探せ。
永遠超えて生きる常食(牛乳でつくったやつを4日間熟成させたものと
、牛乳でつくったやつを一日熟成させたものを混ぜた常食)を常食すると、アディクションが多少治る
登記
899ご冗談でしょう?名無しさん:2010/04/21(水) 01:40:37 ID:BC7cqPlN
900ご冗談でしょう?名無しさん:2010/04/21(水) 01:55:18 ID:???
900
901ご冗談でしょう?名無しさん:2010/04/21(水) 01:59:10 ID:???
超ド素人の質問ですいません。

光というのは、可視光線のことですか?それとも電磁波のことですか?

光速よりも速いものは宇宙には存在しないと聞いたのですが、それはなぜ言い切れるのでしょうか?
何か決定的な根拠があるのでしょうか?
902ご冗談でしょう?名無しさん:2010/04/21(水) 02:39:30 ID:BC7cqPlN
会社、企業、財閥を探せ まーちゃんの
169億社ある。日本人枠を探せ。発表してください。
903ご冗談でしょう?名無しさん:2010/04/21(水) 03:03:33 ID:???
>>883
正規分布にはならない
極端な話、偏差値25と75のやつを抽出して再テストしたら
二つ山の分布になる
904ご冗談でしょう?名無しさん:2010/04/21(水) 03:10:35 ID:???
>>901
・一般的な定義だと、紫外光や赤外光も光に含まれるから。
 可視光⊂光⊂電磁波
・歴史的には、まず光速度が一定である事が実験事実として示され、(マックスウェル方程式からの帰結も有ったが)
その事を原理の一つとして特殊相対論が構築され、これが(それ以前矛盾と考えられていた点を含め)観測事実とよく合致し、ニュートン力学を包含する物理理論として認められ、
一般相対論もあわせて、光速は光の速さというだけでなく、この時空の一般的性質として受け入れられた。
だから物理学者は光速を絶対のものとして断定したりはしない。
観測事実として光速を上回る現象が発見・検証されれば、それに合致する理論を構築する事になるだろう。
905ご冗談でしょう?名無しさん:2010/04/21(水) 11:05:13 ID:VAzRSf6K
エネルギー水としての甘露水の作り方 大麦茶とビール(スーパードライというのが良い)で光水、ピッカのキレイト、砂糖少々の順で かき混ぜる
登記
906ご冗談でしょう?名無しさん:2010/04/21(水) 11:20:27 ID:FHUerObm
現在大学2年で理論物理を志している人間が
いまのうちにやっておいたほうがいいことを教えてください
907ご冗談でしょう?名無しさん:2010/04/21(水) 11:34:40 ID:???
一生食える財源確保
908ご冗談でしょう?名無しさん:2010/04/21(水) 11:36:17 ID:???
彼女を作る
909ご冗談でしょう?名無しさん:2010/04/21(水) 12:59:44 ID:???
>一生食える財源確保
これは胆に銘じるべきことだろうなぁ。
910ご冗談でしょう?名無しさん:2010/04/21(水) 13:34:06 ID:???
>>906
先輩の進路・就職率を調べる
百人の博士のなんとかというサイトを見る
911ご冗談でしょう?名無しさん:2010/04/21(水) 15:10:16 ID:???
アリストテレスとアインシュタインはどっちの方が天才なんですか?
912ご冗談でしょう?名無しさん:2010/04/21(水) 17:29:08 ID:???
>>853-857
ありがとうございます。表現形式の違いについてはよくわかりました。
運動量と仕事量についても納得しました。

実は理屈や意味として納得はしたものの、単位が W と「kg * m^2 / s^3」で
可換なのには違和感ありまくりなんですが、これは何かしら根源的な疑問なので
解消のしようもないのでしょうね。
913ご冗談でしょう?名無しさん:2010/04/21(水) 17:37:43 ID:???
>>912
ワットの定義が kg*m^2/s^3 なんだが、可換とは?
914ご冗談でしょう?名無しさん:2010/04/21(水) 17:59:40 ID:???
>>913
「wattって元々、V * Aの事でしょ」と言う風に考えてるのでは?

だから、それが kg * m^2 / s^3 に一致するのは何で?と疑問なのでしょう。

実は話が逆なのは定義にさかのぼっていろいろ調べてみないと気づかない。
915ご冗談でしょう?名無しさん:2010/04/21(水) 21:48:31 ID:???
アリスとテレスはどっちが頭いいんですか?
916ご冗談でしょう?名無しさん:2010/04/21(水) 22:26:50 ID:???
しまいにゃアリとキリギリスはどっちが偉いのですか?とか聞いて来そう。
917三流学生:2010/04/21(水) 22:40:03 ID:???
>>912
W=V*A=(電場*距離)*(電荷/時間)=(電場*電荷*距離)/時間=エネルギー/時間=質量*速さ^{2}/時間=kg m^{2}/ s^{3}でそ。

>>901
光速よりもはやい「もの」というのが良くわからないけど、その「もの」が「質量のある物質」ということなら、
実験事実から光速を超えるように加速することはできないことは論理の帰結です。

したがって、「決定的な根拠」を挙げろというのなら、それは「光速度が慣性系によらず一定である」という
過去より何度も繰り返されてきた精密な実験の結果である、と答えることになります
918ご冗談でしょう?名無しさん:2010/04/21(水) 22:45:08 ID:???
ニュートンとガウスはどっちの方が天才なんですか?
919ご冗談でしょう?名無しさん:2010/04/21(水) 23:05:43 ID:???
>>917
1voltとは1C(クーロン)の電荷をそれに逆らって移動させると1J(ジュール)の仕事をした事になる電位差(電気ポテンシャル差)

つまり、V=J/Cと定義されている

と言うのがスッキリした説明なんじゃ無いか?
920ご冗談でしょう?名無しさん:2010/04/21(水) 23:25:33 ID:???
どこから質量とか速さでてきたん
単位はm、kgでええの
921ご冗談でしょう?名無しさん:2010/04/21(水) 23:31:01 ID:???
>>920
>>919の説明の事を聞いてるの?
それなら1J(ジュール)と言う仕事(エネルギー)の単位が元々 m kg sから組み立てられてる複合単位だから。
922ご冗談でしょう?名無しさん:2010/04/21(水) 23:34:01 ID:???
エバネッセント場は界面に対して垂直方向には伝播せずに減衰するけど
界面と平行な方向には伝播するってどういう感じですか?

細いレーザー光をプリズムで全反射させると、エバネッセント場は
反射光と同じ方向にプリズムの表面を反射光の界面方向への写像という感じで
伝わっていくと考えて良いんですか?
923ご冗談でしょう?名無しさん:2010/04/21(水) 23:39:02 ID:???
定義を出して「説明」なら
ありがとう
924ご冗談でしょう?名無しさん:2010/04/21(水) 23:57:07 ID:???
>>923
なぜエネルギー(仕事量)の単位が m kg s のあのような組み合わせで表現できるのか?と言う疑問ですか?

それは熱量と仕事とエネルギーの概念を発明あるいは発見した当時の歴史的経緯と様々な実験を知らないと納得行かないかも。

つまり端的に言えば実験の結果、見出した関係なのです。
925ご冗談でしょう?名無しさん:2010/04/22(木) 00:25:26 ID:93poDFPN
熱膨張率についてなんですけど
http://ja.wikipedia.org/wiki/%E7%86%B1%E8%86%A8%E5%BC%B5%E7%8E%87
ここに「線膨張率をα、体積膨張率をβとすると β=3α の関係がある。」
となってて、そうなる計算式も書いてあって確かに式に間違いは無いとは思うのですが、
感覚的にはαの3乗がβになりそうな気がするのですが、どうして3乗にはならないのですか?

例えばαが1.2で長さが1.2倍になるなら縦と奥行きも1.2倍になってβは1.728になりそうなんですが、
実際は1.6ですよね。
一体私はどこを勘違いしてるんですか?
926ご冗談でしょう?名無しさん:2010/04/22(木) 00:42:10 ID:???
>>925
膨張率というのは大抵はかなり小さいんだよ
927ご冗談でしょう?名無しさん:2010/04/22(木) 00:46:39 ID:???
>>925
立方体において一辺の長さ1が線膨張でαだけ伸びたとする。
伸びた後の体積は(1+α)の3乗で 1+3α+3α^2+α^3
α<<1なのでα^2やα^3の項は無視出来て、体積=1+3α

線膨張で伸びた長さを直線近似とするところですでに近似が入っている。
928ご冗談でしょう?名無しさん:2010/04/22(木) 00:57:42 ID:???
あのWikipediaの式はちょっと分かりにくい形で線膨張率が小さいので、
二乗以上の項が無視できると言う前提が入り込んでるね。

差分の形できちんと書き直せば分かる。
929ご冗談でしょう?名無しさん:2010/04/22(木) 00:59:07 ID:???
「存在」とはどういうことなのでしょうか?
930ご冗談でしょう?名無しさん:2010/04/22(木) 01:16:17 ID:???
>>923
「なぜそういう定義なのか」は既に散々説明されてるだろ。すこしは恥を知れ
931ご冗談でしょう?名無しさん:2010/04/22(木) 01:24:07 ID:???
>>922
一回自分で波動方程式解いてみるか
複素のkベクトル考えてみれば分かるよ
932ご冗談でしょう?名無しさん:2010/04/22(木) 01:40:12 ID:???
「存在」とはどういうことなのでしょうか?
933ご冗談でしょう?名無しさん:2010/04/22(木) 01:44:50 ID:???
>>932
そんなものはあんたの脳の中にしかないよ。
934ご冗談でしょう?名無しさん:2010/04/22(木) 01:56:19 ID:???
太陽の熱が地球に伝わるのは熱放射とかありえない
935ご冗談でしょう?名無しさん:2010/04/22(木) 01:59:09 ID:???
>>934 なんで?
相手が6000℃くらいと言うものすごい高温だから、日常の常識では計れないよ。
936ご冗談でしょう?名無しさん:2010/04/22(木) 02:02:25 ID:???
答えられないなら黙ってればいいのに
937ご冗談でしょう?名無しさん:2010/04/22(木) 02:04:34 ID:???
いや、むしろ何が疑問なのかが分からない。
938ご冗談でしょう?名無しさん:2010/04/22(木) 02:04:44 ID:???
>>936
誰に対して言ってるの?
939ご冗談でしょう?名無しさん:2010/04/22(木) 02:04:53 ID:???
>>932
他のもの(観察者)と何らかの相互作用をしうる状態、とでもいうのかなあ?
940ご冗談でしょう?名無しさん:2010/04/22(木) 02:17:42 ID:???
存在=認識=脳
941ご冗談でしょう?名無しさん:2010/04/22(木) 03:15:14 ID:0MFLKKpn
物理の質問ですがお願いします

極座標系における点P(r,θ,φ)を考え、原点0から点Pに向かうベクトルを
r↑とする。ベクトル場の以下の量を計算せよ。ただしaは0でない

(1) grad(1/ar)

(2) div(ar↑)


直交座標系なら解けるのですが、極座標系になるとわからなくなりました。簡単かもしれませんがよろしくお願いします
942ご冗談でしょう?名無しさん:2010/04/22(木) 03:43:41 ID:???
(x,y)のデータから最小二乗法でy=ax+bのaとbを求めた時のaとbの誤差はどうやって求めればいいのでしょうか・・・
943ご冗談でしょう?名無しさん:2010/04/22(木) 03:46:28 ID:???
>>941 なんで極座標だとわからないの?公式教科書に載ってない?
その公式に従って粛々と計算するだけ。公式自体が疑問なら、その導出法を
勉強すればよい。直交座標系の場合の導出法は多くの教科書に載ってるよ。
 なお、君の問題においては(1)はrにしか依存しないのでr微分だけがでて
くるし(2)もr成分しかないのでdivもr成分のみの微分だけが問題。(2)の場合、
divの意味からしてar↑のr成分に何かをかけてからr微分してそれから・・・
ってやらないといけない。詳しくは教科書みてね。
944ご冗談でしょう?名無しさん:2010/04/22(木) 04:22:43 ID:???
脳内ではなく、現実に「無限」は存在するのでしょうか?
というか、「無限」はあり得るのでしょうか?
945ご冗談でしょう?名無しさん:2010/04/22(木) 12:51:49 ID:???
>>944
全ての存在は脳内でしか認識されない。
従って無限は存在するとしても脳内にしか存在しない。
946ご冗談でしょう?名無しさん:2010/04/22(木) 14:47:59 ID:w4ROaeho
>>492
図を書いてみ
947ご冗談でしょう?名無しさん:2010/04/22(木) 16:51:42 ID:???
>>946
凍ったペキンダックが跳ねる図・・・ シュールななー
948ご冗談でしょう?名無しさん:2010/04/22(木) 17:28:56 ID:???
真空ポンプを使って容器内の圧力を下げた後に何もせず放置した場合、どんなに性能の良い容器でも、内部の圧力は上昇してしまうのでしょうか?
また、その際の圧力の上がる仕組みについて教えてください。
949ご冗談でしょう?名無しさん:2010/04/22(木) 17:46:28 ID:???
>>948
内部の圧力は上昇してしまう。

容器内壁を構成する分子が染み出したり、容器自体を透過して分子が乱入してくる。
950ご冗談でしょう?名無しさん:2010/04/22(木) 18:06:23 ID:???
継ぎ手やフランジなどの隙間からの漏れとかもな
951ご冗談でしょう?名無しさん:2010/04/22(木) 18:24:36 ID:???
真空容器をさらに真空容器の中に入れれば大分性能が良いものになるよ
952ご冗談でしょう?名無しさん:2010/04/22(木) 20:36:33 ID:???
>>949
容器自体を透過したりもするんですね・・・

>>950
考えうる全ての隙間からは漏れがあるかもしれないということですね。

>>951
そんな手もあるんですね。参考になります。

皆様親切に回答ありがとうございました。
953ご冗談でしょう?名無しさん:2010/04/22(木) 20:46:15 ID:???
>>951
これそうなの?
普通の極高真空容器って漏れはほとんどないでしょ
954ご冗談でしょう?名無しさん:2010/04/22(木) 20:58:07 ID:???
どの程度のオーダーの真空が欲しいのかによる。
955ご冗談でしょう?名無しさん:2010/04/22(木) 22:06:58 ID:tHSGpMr9
>>952
壁面吸着ガスのデガスはゼロにならないしね
(ベーキングして実用上かなり改善できる)
逆を利用して、ゲッタ材で真空改善できる

逆に、チャンバー内に余計なモンを入れると真空度は落ちる。
個人的には、入れるチャンバーはきちんとケアしないと、>>951は逆効果にもなり得るなぁ

RP/DP→TMPのマトリョーシカは個人的にあんまりやりたくない。
もう、原子レベルのことをやる人か?

普通の製膜くらいだったら、
TMPメインチャンバーとRPロードロックチャンバーで対処するかなぁ
956ご冗談でしょう?名無しさん:2010/04/22(木) 22:21:40 ID:???
生成消滅演算子について質問です

a†|0>やa†a†|1>
の|0>や|1>はそれぞれの演算子に対応した固有ベクトルと考えて間違いないのでしょうか

また光子数ないし電子数の基底を表していると言うことはヒルベルト空間上で
|0>や|1>や|2>は直交していると考えて間違いないのでしょうか
957ご冗談でしょう?名無しさん:2010/04/22(木) 22:26:05 ID:???
>>956
定義次第。
958ご冗談でしょう?名無しさん:2010/04/22(木) 22:30:12 ID:???
最初>>957はあんまりだとおもったけど
>>956は教科書100回読んだほうがいい
959ご冗談でしょう?名無しさん:2010/04/22(木) 22:31:59 ID:???
>>945
回答ありがとうございます。

では、脳内以外(外界)に「無限」は存在するのでしょうか?
960ご冗談でしょう?名無しさん:2010/04/22(木) 22:35:55 ID:???
>>959
存在しても認識できない
961ご冗談でしょう?名無しさん:2010/04/22(木) 22:50:42 ID:???
>>958
確かに教科書に書いてあることですよね…シッフにでも舞い戻りますわ
962ご冗談でしょう?名無しさん:2010/04/22(木) 22:52:55 ID:???
>>959
「無限」って密度が無限大とかの特異点のような話じゃ無いよね。
例えば宇宙空間のサイズは無限だとか、時間は無限だとか言う話?

だとすると理論的には可能性を否定できないが、実際にはその可能性は低いと考えてる人が多いような気がする。

現実に無限があると考えると、いろいろ理論的に都合が悪い事も起こり得そうで。

飽くまで、私の私見と言うか印象だけど。
963ご冗談でしょう?名無しさん:2010/04/22(木) 22:56:55 ID:???
>>945 で脳内にしか存在しないっていってるのに
>>959
>では、脳内以外(外界)に「無限」は存在するのでしょうか?
ってなんなの
964ご冗談でしょう?名無しさん:2010/04/22(木) 23:02:26 ID:???
>>963
>>959>>945のような意見は想定済みなので無視すると言う事でしょ?

つまり、彼が聞きたいのはそんな話じゃ無い(ピント外れだ)と言う意思表示だろう。

やんわりとした表現でね。
965ご冗談でしょう?名無しさん:2010/04/22(木) 23:04:40 ID:???
量子力学で
波動関数をフーリエ変換することで距離空間から波数空間や周波数空間に写すことが出来ますが
これはフーリエ変換のどのような性質に拠っているのでしょうか

フーリエ変換って単に関数を三角関数の和であらわすってだけじゃないの?
966ご冗談でしょう?名無しさん:2010/04/22(木) 23:13:49 ID:???
>>965
フーリエ変換は量子力学だけじゃ無くて、時間域の信号(例えば音声信号)を周波数領域で表すのに良く使われるけど。

つまり、いろんな信号を三角関数の和(周波数スペクトル)で表せます。
967ご冗談でしょう?名無しさん:2010/04/22(木) 23:16:03 ID:???
>>965
フーリエ変換とフーリエ級数の区別はついてる?
968ご冗談でしょう?名無しさん:2010/04/22(木) 23:25:13 ID:???
>>966-967
ああ、なるほど。
そういうことね…フーリエ変換とフーリエ級数ごっちゃにしてたわ
969ご冗談でしょう?名無しさん:2010/04/22(木) 23:36:52 ID:???
まあいいや。物理板は>>962のような文系スイーツ(笑)思考の馬鹿しかいないから、数学板で聞いてくるわ。
970ご冗談でしょう?名無しさん:2010/04/22(木) 23:44:19 ID:???
>>964
じゃどんな話を聞きたいっていうんだろ。
>>969を読む限りはよくいる逆ギレ厨房のようだし。
数学板へ行ってどんな答えが得られると思ってるんだろうね。
971ご冗談でしょう?名無しさん:2010/04/22(木) 23:45:24 ID:???
フーリエ級数
f(x)=Σ{a(k)cos(kx)+b(k)sin(kx)}
でも、元の関数f(x)から係数a(k),b(k)への対応だと考えれば同様の話になる
972ご冗談でしょう?名無しさん:2010/04/22(木) 23:55:52 ID:???
>>970
確かに不思議だね。
数学の話なら無限なんかいくらでも存在して珍しくも無いけど、
それと物理的に本物の無限があり得るかは全然ちがう話だし。
973ご冗談でしょう?名無しさん:2010/04/23(金) 00:19:44 ID:???
>>971
そういう見方もあるのか。
勉強になります
974ご冗談でしょう?名無しさん:2010/04/23(金) 00:30:40 ID:???
位置と運動量は互いに相補的なのに、何で量子論では波数空間でしか
話をしないんでしょうね。
ファイマンダイアグラムといい。
975ご冗談でしょう?名無しさん:2010/04/23(金) 00:41:01 ID:???
輪ゴムがあります。
輪ゴムに指をつけてゴムをなぞっていって下さい。
終わりはありますか?
ありません。
はい「無限」です。
976ご冗談でしょう?名無しさん:2010/04/23(金) 00:44:45 ID:???
時間が有限なら有限だろ。
977ご冗談でしょう?名無しさん:2010/04/23(金) 01:00:01 ID:???
>>959
数直線上の0〜1までの区間には無限個の点が含まれる件
978ご冗談でしょう?名無しさん:2010/04/23(金) 01:07:56 ID:???
ありがとうございます。
では、
979ご冗談でしょう?名無しさん:2010/04/23(金) 01:09:56 ID:???
>>975
だっせー
980ご冗談でしょう?名無しさん:2010/04/23(金) 01:11:08 ID:???
>>977
無限個あることを公理を立てないで証明できるかという件
981ご冗談でしょう?名無しさん:2010/04/23(金) 01:13:26 ID:???
>>977
それは数学上、つまり理論上の無限であって、空間や物質が本当に無限に細かく分けられるかはかなり疑問だ。

つか、物質は素粒子が存在すると思われてるから、時間も空間も実は最小単位が存在するのでは無いか?
982三流学生:2010/04/23(金) 02:44:11 ID:???
>>974
たぶん相補的と言う言葉の意味を間違えている。

それはいいとして、kを量子数にとるのは、当然それが時間変化しない「良い量子数」に
なるから。例えば自由粒子を考えればわかるはず。

>>981
時空が離散的って仮定しないと説明できない実験事実でもあるの?
そうじゃない限り、それこそ実在しない妄想の世界になるでそ。
983ご冗談でしょう?名無しさん:2010/04/23(金) 02:49:43 ID:???
量子力学を真っ向から否定する気かよ
984ご冗談でしょう?名無しさん:2010/04/23(金) 05:23:05 ID:Ow+sCCk5
大学の参考書読む
985ご冗談でしょう?名無しさん:2010/04/23(金) 06:21:35 ID:???
時空が離散的って可能性はある。その場合は従来微分方程式で解いてたものは全て差分方程式でやらざるを
えなくなる。時空の最小単位が無限小の極限においては従来の物理学が成り立つ。
986DCCLXXIV ◆u4TdTqYF3n5x :2010/04/23(金) 07:44:56 ID:???
>>985
意地悪だが、
(・∀・)つ「カオス理論」
987ご冗談でしょう?名無しさん:2010/04/23(金) 08:48:48 ID:???
誰か次スレ立てて
988ご冗談でしょう?名無しさん:2010/04/23(金) 11:10:54 ID:???
>>986
離散的でもカオスは起きると思うけど?
でなければコンピューターシミュレーションが出来ない。
989ご冗談でしょう?名無しさん:2010/04/23(金) 11:56:11 ID:???
990三流学生:2010/04/23(金) 12:30:51 ID:???
指摘の意味が通じていない気がする…。
理論化=モデル化という立場を貫く限り、いまんところ離散的な時空を考えるのは自己満足でしかない。と思うんです。

確かに「可能性がある」、と言うのは勝手だし、否定はしないですけど。その理論が説得力を持つまで
洗練されたものかは、まだ誰もわからないわけですから。
991ご冗談でしょう?名無しさん:2010/04/23(金) 13:08:37 ID:???
>>982
時空が連続と仮定しないと説明出来ない実験事実でもあるの?
992ご冗談でしょう?名無しさん:2010/04/23(金) 13:29:19 ID:???
そう、時空が不連続だと積極的に主張してるわけでは無い。
逆に連続で無ければならないと言う理論的、実験的証拠が無いなら両方の可能性を考慮しておかなければならないと言うだけ。

そう考えると、数直線を持ち出して「ほら、ここに無限がある」と言う主張は特に根拠があるとは言えない。
993ご冗談でしょう?名無しさん:2010/04/23(金) 13:31:25 ID:???
素粒子の表面も無限
994ご冗談でしょう?名無しさん:2010/04/23(金) 13:38:16 ID:???
素粒子に表面があるのだろうか?
995ご冗談でしょう?名無しさん:2010/04/23(金) 14:16:22 ID:???
スレ立て乙です
996ご冗談でしょう?名無しさん:2010/04/23(金) 15:56:19 ID:???
>>994
というか素粒子って大きさ持ってるんだっけ?
997ご冗談でしょう?名無しさん:2010/04/23(金) 16:14:50 ID:???
>>991
いずれ人類が超ひものサイズより小さいスケールの大きさを実験的にも扱うようになってくると問題が生じるんじゃね?
素粒子論も無限に続く入れ子構造に終止符が打たれて真の基本粒子が見つかるようになるかもしれない。
確か益川氏って、素粒子の入れ子構造が無限に続いてくことを期待してるとか書いてた気がする(論文レベルじゃなく
て期待してるってレベルだろうと思うけど。東洋的な輪廻の思想に関係してるから西欧の研究者には理解されない的な
事も書いてた気がする。うろ覚えだけど。
998ご冗談でしょう?名無しさん:2010/04/23(金) 16:17:13 ID:???
>>997
> >>991
> 確か益川氏って、素粒子の入れ子構造が無限に続いてくことを期待してるとか書いてた気がする(論文レベルじゃなく

実際に入れ子構造が続いていくんじゃなくて人間の認識理解能力では実世界をそういう風にしか捉えることができない、ということじゃないかな。
999ご冗談でしょう?名無しさん:2010/04/23(金) 16:23:16 ID:???
>>998
いやもっと具体的な表現だった気がするけど補足すると、基本粒子が見つかっても必ずもっと小さい構造が
見つかる→素粒子論に終点が無いみたいな感じだった。認識の問題とかは面白そうだけどその時の文脈は
もっと単純に素粒子の構造についてだけ言ってた気がする。
1000ご冗談でしょう?名無しさん:2010/04/23(金) 17:19:28 ID:???
もうここは哲学のスレですね
10011001
このスレッドは1000を超えました。
もう書けないので、新しいスレッドを立ててくださいです。。。